Вы находитесь на странице: 1из 146

– Faraday’s Law and AC circuits – yeazell – (58010) 1

This print-out should have 34 questions. first withdrawn upward from the loop of
Multiple-choice questions may continue on wire, then moved downward toward the
the next column or page – find all choices loop of wire, the current in the loop
before answering. is first clockwise, then counter-clockwise , as
viewed from above.
001 10.0 points
In the figure shown, the magnet is first with- 002 (part 1 of 3) 10.0 points
drawn upward from the loop of wire, then
moved downward toward the loop of wire. A rectangular loop of copper wire of resis-
tance R has width a and length b. The loop
S up is stationary in a uniform magnetic field. The
then magnetic field B at time t = 0 seconds is di-
down rected into the page as shown below. The
N uniform magnetic field varies with time t ac-
Counter- cording to the relationship B = B0 cos ω t,
Clockwise clockwise
where ω and B0 are positive constants and B
induced I I induced is positive when the field is directed into the
current current
page.
B b B
As viewed from above, the induced current
in the loop is

a
1. first counter-clockwise, then clockwise.
n turns
2. for both cases counterclockwise with in-
creasing magnitude. B B

3. for both cases counterclockwise with de- The direction ofπthe induced current in the
creasing magnitude. loop when ω t = , after the magnetic field
2
begins to oscillate is
4. first clockwise, then counter-clockwise.
correct 1. undetermined, since the current is zero.

5. for both cases clockwise with increasing 2. counter-clockwise.


magnitude.
3. clockwise. correct
6. for both cases clockwise with decreasing
magnitude. Explanation:
Explanation:
From Ohm’s law and Faraday’s law, the
V 1 dΦ
current in magnitude is I = = − ,
R R dt
where Φ is the magnetic flux through the loop.
We know the sign of the rate of change of the
magnetic flux is changed when the magnet is
withdrawn upward, as is the current direction
according to the above eqaution. π π
Using the right-hand-rule and from Lenz’s When ω t = , B = B0 cos = 0; i.e.,
2 2
law, we know that when the magnet is the field has been decreasing, and is about to
– Faraday’s Law and AC circuits – yeazell – (58010) 2
change direction. The induced current will
be in a direction to oppose this change; i.e., I
clockwise.

2. ωt
003 (part 2 of 3) 10.0 points
What is the expression for the magnitude of 0 π π 3π 2π 5π 3π
the induced current in the loop as a function 2 2 2
of time in terms of a, b, B0 , ω, R, t, and
fundamental constants.
R ω B0 I
1. I = | sin ω t|
ab
a b B0
2. I = | sin ω t| 3. ωt
ωR
0 π π 3π 2π 5π 3π
a b ω B0
3. I = | sin ω t| correct 2 2 2
R
4. I = a b ω B0 R | sin ω t|
a b B0 I
5. I = | sin ω t|
R
R B0
6. I = | sin ω t| 4. ωt
ab
Explanation: 0 π π 3π 2π 5π 3π
Calculating the flux, 2 2 2
Φ = a b B0 cos ω t .
Calculating the emf,
I

E=− (negative sign not required)
dt
= a b ω B0 sin ω t . 5. ωt
0 π π 3π 2π 5π 3π
Using Ohm’s Law, the magnitude of the cur-
rent is 2 2 2
|E| a b ω B0
I= = | sin ω t| .
R R
I
004 (part 3 of 3) 10.0 points
Select a graph of the induced current I vs ω t,
taking clockwise current to be positive. 6. ωt
0 π π 3π 2π 5π 3π
I 2 2 2

1. ωt
0 π π 3π 2π 5π 3π
2 2 2
Explanation:
correct The graph is a sine wave with period 2 π .
– Faraday’s Law and AC circuits – yeazell – (58010) 3

I To maintain the motion of the bar, a force


must be applied on the bar to balance the
magnetic force
ωt
0 π π 3π 2π 5π 3π F = FB = 32 N .
2 2 2

006 (part 2 of 2) 10.0 points


At what rate is energy dissipated in the resis-
005 (part 1 of 2) 10.0 points
tor?
In the arrangement shown in the figure,
Correct answer: 256 W.
the resistor is 4 Ω and a 4 T magnetic field
is directed into the paper. The separation Explanation:
between the rails is 1 m . An applied force The power dissipated in the resistor is
moves the bar to the left at a constant speed
of 8 m/s .
P = I 2 R = (8 A)2 (4 Ω)

4T = 256 W .

8 m/s
m≪1 g
1m

4Ω

007 (part 1 of 3) 10.0 points


I

A pendulum consists of a supporting rod


4T and a metal plate (see figure). The rod is piv-
oted at O. The metal plate swings through
a region of magnetic field (directed into the
Calculate the applied force required to paper). Consider the case where the pendu-
move the bar to the left at a constant speed of lum’s metallic plate enters the magnetic field
8 m/s. Assume the bar and rails have negligi- region from left to right.
ble resistance and friction. Neglect the mass O
of the bar.

Correct answer: 32 N.
Explanation:
The motional emf induced in the circuit is eld
entering fi
E = B ℓ v = (4 T) (1 m) (8 m/s)
= 32 V .
From Ohm’s law, the current flowing through B B
the resistor is
E 32 V
I= = = 8 A, B B
R 4Ω
so the magnitude of the force exerted on the
bar due to the magnetic field is
FB = I ℓ B = (8 A)(1 m)(4 T) The direction of the circulating eddy cur-
= 32 N . rent in the plate is
– Faraday’s Law and AC circuits – yeazell – (58010) O 4

1. clockwise.

2. cannot be determined.

3. counter-clockwise. correct
eld
Fw entering fi

B B
i

Explanation: B B
A conduction electron in the pendulum will
experience a magnetic force opposite the di-
~ so using the right hand rule,
rection of ~v × B,
we can determine that the motion of the elec- Alternative Solution: Use the result of
trons will be in the clockwise direction, which Part 1, and the right hand rule on the current
produces a counter-clockwise current. flow to determine that the induced magnetic
field must be directed out of the paper.
008 (part 2 of 3) 10.0 points
The direction of the induced magnetic field at 009 (part 3 of 3) 10.0 points
the center of the circulating eddy current The direction of the force which the magnetic
field exerts
1. is along the rod toward the pivot point.
1. is into the plane.
2. is out of the plane. correct
2. is along the rod away from the pivot
3. is along the rod away from the pivot point.
point.
3. is along the direction of swing.
4. is into the plane.
4. is along the rod toward the pivot point.
5. cannot be determined.
5. is opposite to the direction of swing. cor-
6. is opposite to the direction of swing. rect

7. is along the direction of swing. 6. is out of the plane.

7. cannot be determined.

Explanation:
Because the magnetic field only exerts a
force on the current segment already in the
Explanation: magnetic field region, the net magnetic force
Because the magnetic field is pointing into is opposite to the direction of swing.
the paper, the magnetic flux through the pen-
dulum is increasing, so by Lenz’s law we know 010 10.0 points
that the induced magnetic field is in the op- A circular coil is made of N turns of copper
posite direction, or out of the paper. wire as shown in the figure. When viewed
– Faraday’s Law and AC circuits – yeazell – (58010) 5
from the right the coil is wound counter- 011 (part 1 of 2) 10.0 points
clockwise. A resistor R is inserted in the The current in a 79 mH inductor changes with
copper wire. Initially, a uniform magnetic time as I = b t2 − a t.
field of magnitude Bi points horizontally from With a = 10 A/s and b = 2 A/s2 , find
left-to-right through the perpendicular plane the magnitude of the induced emf, |E|, at
of the coil. t = 0.7 s.

Correct answer: 0.5688 V.


Explanation:

Magnetic
Field B(t) Let : L = 79 mH = 0.079 H ,
R b = 2 A/s2 ,
a = 10 A/s , and
During a time interval t the field uniformly t = 0.7 s .
changes at a constant rate, until a reversed
field is reached equal in magnitude to the From Faraday’s Law, the induced emf E
initial field. is proportional to the rate of change of the
The current in the resistor R magnetic flux, which in turn is proportional
to the rate of change of the current. This is
1. is zero. expressed as

2. flows from left to right. correct dI


E =L
dt
3. flows from right to left. d
=L (b t2 − a t)
dt
4. flows in a direction that cannot be deter- = L (2 b t − a)
mined from the information given.
At 0.7 s ,the magnitude of the induced emf is
Explanation:
As the left-to-right magnetic field decreases |E| = (0.079 H) 2(2 A/s2 )(0.7 s) − 10 A/s

(and eventually flipping sign and increasing
in magnitude) it follows from Lenz’s law (op- = 0.5688 V .
position to the change in magnetic field will
tend to keep the current constant and flow-
ing in the same direction) that the induced 012 (part 2 of 2) 10.0 points
emf will produce a left-to-right magnetic field At what time is the emf zero?
arising from induced currents in the coil.
By the right hand rule, the induced current Correct answer: 2.5 s.
flows counter-clockwise when viewed from Explanation:
the right and the coils are wound counter- From Part 1, the induced emf is zero when
clockwise as the wire goes from the right to
left terminals. The current must enter the dI
loop from the right terminal and exit at the =0
dt
left terminal. 2 b t0 − a = 0
Since the current is continuous, the current (10 A/s)
must flow through the resistor in the left-to- t0 =
2 (2 A/s2 )
right direction.
= 2.5 s .
– Faraday’s Law and AC circuits – yeazell – (58010) 6
LE
2. UL =
013 (part 1 of 2) 10.0 points 2R
LE
Consider the circuit shown. 3. UL =
3R
R L R2
4. UL =
2 E2
LE
R L 5. UL =
P 32 R
L E2
E 6. UL = correct
2 R2
S
LE
7. UL =
16 R
What is the instantaneous current through E 2 R2
the upper resistor and what is the instanta- 8. UL =
4L
neous current at point P immediately after LE
the switch is closed? 9. UL =
4R
E LE
1. IR (0) = ; IP (0) = 0 correct 10. UL =
R 8R
E E Explanation:
2. IR (0) = ; IP (0) = After the switch has been closed for a long
R RL
E E time, the current in L does not change any
3. IR (0) = ; IP (0) = more, so there is no voltage increase nor de-
R 2R
crease across L. Therefore, the current going
E E E
4. IR (0) = ; IP (0) = through L is I = , which gives the energy
R R R
E stored in L as
5. IR (0) = 0 ; IP (0) =
R 1 L E2
UL = L I 2 = .
E 2 2 R2
6. IR (0) = 0 ; IP (0) =
2R
015 (part 1 of 4) 10.0 points
7. IR (0) = 0 ; IP (0) = 0
E Consider the circuit. The switch is closed
8. IR (0) = 0 ; IP (0) = at t = 0. The current I through the inductor
RL
takes the form
E  
Explanation: I= 1 − e−t/τx
Rx
The current in L has to change gradually, so
immediately after the switch is closed, there where Rx and τx are to be determined.
is no current going through point P.
I L
014 (part 2 of 2) 10.0 points
When the switch has been closed for a long R2
I2
time, what is the energy stored in the induc-
tor?
E
S R1
L E2
1. UL =
4 R2 I1
– Faraday’s Law and AC circuits – yeazell – (58010) 7
Find I immediately after the circuit is Explanation:
closed. See previous explanation.

1. I = 0 correct 017 (part 3 of 4) 10.0 points


Find I after the circuit is closed for a long
E time.
2. I =
R1
E 1. I = 0
3. I =
R2
E E
4. I = 2. I =
R1 + R2 R1 + R2
Explanation: E
3. I =
Before the circuit is closed, no current is R2
flowing. When we have just closed the circuit E
4. I = correct
we are at “t = 0+ ”, a mathematical nota- R1
tion meaning a very short time ǫ after t = 0. Explanation:
(Nothing happens in the circuit at t = 0, only When we wait a long time, the current I
immediately after when the switch is, indeed, dI
closed. However, this is just a mathematical though the inductor levels out; i.e., → 0.
dt
detail.) There are two loops in the prob- The voltage over the inductor is
lem, one with E, R1 , R2 and one with E, R1 ,
L. So at t = 0+ , the battery “wants to” dI
VL = L ,
drive a current through both loops. The first dt
loop presents no problem; since there is no in-
ductance “working against us,” a current will at all times, so VL → 0 as t → ∞, and we can
immediately be set up. The second loop, how- replace the inductor in the figure by a straight
ever, has an inductor which tries to prevent wire. When the current now comes from R1
any change in the current going through it, to the junction where I1 splits into I2 and I,
and so goes up smoothly from I = 0, as can there is no resistance in the “I” path but a
be seen in the given solution (just put t = 0 nonzero resistance R2 in the other. Naturally,
to find I = 0). Therefore, at this instant, the the current takes the path with no resistance.
inductor L carries no current, and we can ne- Since we do pass through R1 in any case, the
glect it when we find the current through R2 . equivalent resistance is now R1 . At t = ∞
The equivalent resistance is Req = R1 + R2
so, from E = Req I we find E
I= and I2 = 0
E R1
I2 =
R1 + R2
Alternate Solution: An equation of the
016 (part 2 of 4) 10.0 points form
Find I2 immediately after the circuit is closed. dI
L −RI −E = 0
dt
1. I2 = 0 has a solution which is composed of a ho-
mogeneous solution and a particular solution.
E
2. I2 = The particular solution corresponds to the
R1 first term in the I in the problem statement;
E the homogeneous corresponds to the second
3. I2 = correct
R1 + R2 term. The homogeneous solution is therefore
E the only one of relevance to the time con-
4. I2 =
R2 stant τx , and we can disregard any batteries
– Faraday’s Law and AC circuits – yeazell – (58010) 8
E. This provides a simple solution to Part 3:
Just ignore the battery and find the equiva- Correct answer: 0.000284605 A.
lent resistance connected to L, in this case Explanation:
1 1 1
= + ,
Req R1 R2 Let : C = 27 pF = 2.7 × 10−11 F ,
so the time constant is L = 12 mH = 0.012 H , and
E = 6 V.
 
L 1 1
τ= =L +
Req R1 R2 The maximum charge on the capacitor is
as before. Nobody seems to have proven that Qmax = C V . The maximum current is
ignoring the batteries always works, so the Imax = ω Qm = ω C V ,
previous solution is more reliable.
where ω is the oscillatory frequency given by
018 (part 4 of 4) 10.0 points 1
Find I2 after the circuit is closed for a long ω=√ , so
LC
time.
E
r r
1. I2 = C 2.7 × 10−11 F
R1 Imax = V = (6 V)
L 0.012 H
E
2. I2 = = 0.000284605 A .
R1 + R2
3. I2 = 0 correct
020 (part 2 of 2) 10.0 points
E What is the maximum energy stored in the
4. I2 = magnetic field of the inductor?
R2
Explanation:
See previous explanation. Correct answer: 4.86 × 10−10 J.
Explanation:
019 (part 1 of 2) 10.0 points
Q2max 1
An LC circuit is shown in the figure below. Umax,ind = Umax,cap = = CV2
2C 2
The 27 pF capacitor is initially charged by the 1
6 V battery when S is at position a. Then S = (2.7 × 10−11 F) (6 V)2
2
is thrown to position b so that the capacitor
is shorted across the 12 mH inductor. = 4.86 × 10−10 J .

12 mH
021 10.0 points
27 pF
S b The emf E drives the circuit shown below
at angular frequency ω.
a
6V
L

R C
What is the maximum value for the oscil- E
lating current assuming no resistance in the
circuit?
– Faraday’s Law and AC circuits – yeazell – (58010) 9
Under which of the following conditions The capacitive reactance is
does the largest current flow through this cir-
cuit? 1
XC =
ωC
1 1
1. both higher frequencies, √ < ω < ∞, =
LC (636.62 rad/s) (1.62 × 10−5 F)
1 = 96.9627 Ω ,
or lower frequencies, 0 < ω < √
LC
2. steady DC voltage, ω = 0 and the inductive reactance is

1 XL = ω L
3. only at higher frequencies, √ <ω<
LC = (636.62 rad/s) (0.0216 H)

= 13.751 Ω .
4. the resonant angular frequency, ω = The maximum current is
1
√ correct
LC Vmax
Imax =
1 Z
5. only at lower frequencies, 0 < ω < √ Vmax
LC =p
Explanation: R2 + (XL − XC )2
The current through R, C, and L is 156 V
=p
E (27.8 Ω)2 + (13.751 Ω − 96.9627 Ω)2
I=s 2 .

1 = 1.77813 A .
R2 + ω L −
ωC
1 023 (part 2 of 2) 10.0 points
When ω is on resonance (ω0 = √ ) the What is the power factor for the circuit?
LC
reactance will be zero and the largest current
will flow. Correct answer: 0.316872.
Explanation:
022 (part 1 of 2) 10.0 points The phase angle between the voltage and
In a series RLC ac circuit, the resistance is the current in the circuit is
27.8 Ω, the inductance is 21.6 mH, and the  
capacitance is 16.2 µF. The maximum po- −1 XL − XC
φ = tan .
tential is 156 V, and the angular frequency is R
636.62 rad/s.
Calculate the maximum current in the cir- Since
cuit.
XL − XC 13.751 Ω − 96.9627 Ω
Correct answer: 1.77813 A. =
R 27.8 Ω
Explanation: = −2.99323 ,

the power factor is


Let : ω = 636.62 rad/s ,   
C = 16.2 µF = 1.62 × 10−5 F , −1 XL − XC
cos φ = cos tan
L = 21.6 mH = 0.0216 H , R
 −1 
R = 27.8 Ω , and = cos tan (−2.99323)
Vmax = 156 V . = 0.316872 .
– Faraday’s Law and AC circuits – yeazell – (58010) 10
1
CKUT = 2
024 (part 1 of 3) 10.0 points L ωKUT
1
You are designing a radio receiver based on =
(1.2 × 10−6 H) (6.00044 × 108 Hz)2
the idea of a series LRC circuit as shown be-
1012 pC
low. The two major radio stations in town are ×
KUT and KMFA, which broadcast at frequen- 1C
cies 95.5 MHz and 94.5 MHz, respectively. = 2.31447 pF .
Electromagnetic signals from these ra-
dio stations simultaneously supply sinusoidal
emfs 025 (part 2 of 3) 10.0 points
At what value of the variable capacitance C
EKUT = V0 sin(ωKUT t) and will the signal be maximum for KMFA?

EKMFA = V0 sin(ωKMFA t) Correct answer: 2.36372 pF.


to the circuit via an antenna. The receiv- Explanation:
ing circuit includes an inductor of 1.2 µH, a
variable capacitor C, and a speaker with a
Let : νKMFA = 94.5 MHz = 9.45 × 107 Hz
resistance.
1.2 µH The frequency of KMFA is
KUT
ωKMFA = 2 π νKMFA
Speaker

V0 sin(ω t) R = 2 π (9.45 × 107 Hz)


C = 5.93761 × 108 Hz .
Thus
KMFA
At what value of the variable capacitance 1
CKMFA = 2
C will the signal in the speaker be maximum L ωKMFA
for KUT? 1
=
(1.2 × 10−6 H) (5.93761 × 108 Hz)2
Correct answer: 2.31447 pF. 1012 pC
×
Explanation: 1C
= 2.36372 pF .
Let : ν = 95.5 MHz ,
= 9.55 × 107 Hz , and 026 (part 3 of 3) 10.0 points
L = 1.2 µH , If KUT and KMFA are broadcasting the “Star
= 1.2 × 10−6 H . Spangled Banner” simultaneously at equal
power, which graph represents the sound vol-
The frequency of KUT is ume as the capacitor is tuned?

ωKUT = 2 π νKUT
sound volume

= 2 π (9.55 × 107 Hz)


= 6.00044 × 108 Hz .
1.
At resonance
1
ωKUT = √ , so CKU T CKM F A C
L CKUT
– Faraday’s Law and AC circuits – yeazell – (58010) 11
.
~ is same over the
In the special case where B

sound volume
entire loop

2. ~ ·A
Φ=B ~

and if there are N loops in a coil the flux is


CKU T CKM F A C just multiplied by N , that is

~ · A]
Φtotal = N Φ = N [B ~
sound volume

.
Faraday’s law give the emf induced by the
3. magnetic field in terms of the time derivative
of the flux,

CKU T CKM F A C d Φtotal


EM F = −
correct dt
.
Lenz’s law is often used to determine the
sound volume

direction of the induced currents. A simple


statement of this law is ”The induced current
4. creates an induced magnetic field. Whatever
caused the change in magnetic flux is opposed
by the induced magnetic field.” For example,
CKU T CKM F A C if a magnet is pushed into a coil, the induced
current creates an induced magnetic field that
Explanation: tries to push the magnet back out of the coil.
There will be two resonance frequencies of
equal magnitude, since the amplitude of each
signal is the same.
Consider these definitions and laws when solv-
027 (part 1 of 8) 10.0 points ing the following problems. A rectangular coil
with resistance R has N loops, each loop has
Faraday observed that when a magnetic field length L and width W as shown in the fig-
changed within a coil of wire that a current ure. The coil is moved at constant speed
was induced in that coil. For example, if v throughout all these problem. So it will
a magnet is pushed into or pulled out of a move into and through the region of uniform
coil the resulting change in the magnetic field magnetic field B, with the direction of B as
causes a current in the loop. In order to quan- shown.
tify this effect, he defined a quantity known
as the magnetic flux which gives a measure of B B a
the overlap of the magnetic field and the area L c
of the coil or loop.
The magnetic flux is given by the integral v
W

of the magnetic field B ~ over the area S of a


single loop, b d
Z B B R
Φ= B~ · dA
~
0 x0
S
– Faraday’s Law and AC circuits – yeazell – (58010) 12
A = W (v t) initially, so
B a B c Φtotal = N B W v t
d Φtotal
= NBW v.
v dt

b d 028 (part 2 of 8) 10.0 points


B B What is the magnitude of the current I in
0 x0 the loop when the coil is only partly in the
d Φtotal region of the magnetic field, as shown above?
What is (the time derivative of the
dt d Φtotal dΦ
flux for all turns of the loop) when the coil is Remember that E = − = −N .
dt dt
only partly in the region of the magnetic field, N BW R
as shown above? 1. I =
v
d Φtotal N BR
1. =N BW 2. I =
dt Wv
d Φtotal N BW Lv
2. = N BLv 3. I =
dt R2
N BW v
d Φtotal 4. I = correct
3. =N BL R
dt
N W Rv
d Φtotal 5. I =
4. = N BW Lv B
dt N BW
d Φtotal 6. I =
5. = B Lv R
dt N BW L
7. I =
d Φtotal R
6. =N BW L
dt N BW
8. I =
d Φtotal Rv
7. = N B W v correct
dt 9. I = zero
d Φtotal
8. = B W Lv N BW Lv
dt 10. I =
R
d Φtotal Explanation:
9. =BW v
dt
d Φtotal d Φtotal
10. = zero E =− = −N B W v
dt dt
Explanation: E N BW v
I= =−
R R
~ ·A
Φ=B ~ N Bwv
so the current I = is in the counter-
~ · A]
Φtotal = N Φ = N [B ~ R
clockwise direction (from b up to a) in order
The magnetic flux is given by that flux is created in the loop which opposes
the increase of flux in the loop of the uniform
Φtotal = N (B · A) , magnetic field (into the page) as the loop
moves into this magnetic field.
where N and B are constant, but the area is Note: The minus sign merely indicates that
changing. the direction of the current will be set up
– Faraday’s Law and AC circuits – yeazell – (58010) 13
in such a way so as to resist the increasing N 2 B2 W 2 v
magnetic flux. NIBW = as the forces act on
R
the current within the field, and the horizontal
029 (part 3 of 8) 10.0 points currents have equal and opposite forces. Thus
What is the direction of the current I in the only the left hand vertical loops have a force
coil when the coil is only partly in the region acting on them. This force acts to oppose the
of the magnetic field, as shown above? movement of the coils, and must point right.

1. No current 031 (part 5 of 8) 10.0 points


What is the direction of the force F on the
2. clockwise loop when the coil is only partly in the region
of the magnetic field, as shown above?
3. counter-clockwise correct
1. towards the top of the page
Explanation:
The direction is counter-clockwise. 2. towards the bottom of the page

030 (part 4 of 8) 10.0 points 3. right correct


Use the current I you found in Part 2. What
is the magnitude of the force F on the coil 4. indeterminate, since the force is zero
that results from the interaction of this cur-
rent with the external magnetic field for the 5. left
situation when the coil is only partly in the
region of the magnetic field, as above? Explanation:
~ = W [~I × B]
Since F ~ ,

~ N 2 B2 W 2 v F̂ = Î × B̂ .
1. kF k = correct
R
2 2 Using the right hand rule of cross product,
~k = N B W Lv
2. kF the direction of the force points toward the
R right.
2 2
~k = N B W
3. kF
Rv 032 (part 6 of 8) 10.0 points
2 2
~k = N W Rv
4. kF
B2 aB Bc
2 2 2
~k = N B W Lv
5. kF
R v
2 2
~k = N B R
6. kF
W2 v b d
2 2 2
B B
~k = N B W R
7. kF 0 x0
v
What is the magnitude of the force F on
~ N B W2 Lv
2 2
the coil due to the external magnetic field as
8. kF k =
R2 it moves completely within the magnetic field,
2 2
~k = N B W v
9. kF
as shown above?
R
~k= N 2 B2 W L v
~ k = zero
10. kF 1. kF
R
Explanation: N B W2 v
2 2
~
2. kF k =
The force on the loop is given by F = R
– Faraday’s Law and AC circuits – yeazell – (58010) 14

2 2 2
~k = N B W R
3. kF B B
v a c
~ k = 0 correct
4. kF
v
~k = N 2 B2 W
5. kF d
Rv b
B B
~k = N 2 B2 R x0
6. kF 0
W2 v
What is the direction of the force on the
~k = N 2 B2 W v loop as it moves out of the magnetic field
7. kF
R (edge cd is in the field while edge ab is out of
~k = N B2 W 2 L v
2 the field)?
8. kF
R2
1. towards the bottom of the page
~k = N2 W2 R v
9. kF
B2 2. right correct
~k = N 2 B2 W 2 L v
10. kF
R 3. indeterminate, since the force is zero

4. left

Explanation: 5. towards the top of the page


Within the field, the magnetic flux is con- Explanation:
d Φtotal The force on the loop is given by F =
stant, so − = 0. Thus, E = 0, I = 0,
dt N 2 B2 W 2 v
and no force opposes the motion. NIBW = as the forces act on
R
the current within the field, and the horizontal
033 (part 7 of 8) 10.0 points currents have equal and opposite forces. Thus
What is the direction of the force on the loop only the right hand vertical loops have a force
as it moves completely within the magnetic acting on them. This force acts to oppose the
field? movement of the coils, and must point right.
1. left

2. towards the bottom of the page

3. indeterminate, since the force is zero cor-


rect

4. towards the top of the page

5. right

Explanation:

034 (part 8 of 8) 10.0 points


chester (crc2876) – HW-10 – rodenborn – (92691) 1
This print-out should have 26 questions. Motional E
Multiple-choice questions may continue on
the next column or page – find all choices E = Bℓv.
before answering.
From Ampere’s law, the strength of the
001 10.0 points magnetic field created by the long current-
A car with a 1.34 m long radio antenna travels carrying wire at a distance a from the wire
at 116 km/h in a place where the Earth’s is
µ0 I
magnetic field is 6.46 × 10−5 T. B= .
2πa
What is the maximum possible induced emf
in the antenna as it moves through the Earth’s Hence the potential difference is
magnetic field?
∆V = 
BLv 
µ0 I
Correct answer: 0.00278928 V. = (L v)
2πa
Explanation:
= 68.0483 mV .

Let : ℓ = 1.34 m ,
v = 116 km/h = 32.2222 m/s , and 003 (part 1 of 2) 10.0 points
B = 6.46 × 10−5 T .
In the arrangement shown in the figure, the
If the antenna moves perpendicular to the resistor is 7 Ω and a 7 T magnetic field is
Earth’s field, the induced emf between the directed out of the paper. The separation
ends of the antenna becomes the maximum: between the rails is 8 m . An applied force
moves the bar to the left at a constant speed
E = Bℓv of 3 m/s .
= (6.46 × 10−5 T) (1.34 m) (32.2222 m/s)
= 0.00278928 V . 7T

3 m/s

m≪1 g
002 10.0 points
8m

7Ω
I

A conducting bar moves as shown near a long


wire carrying a constant I = 130 A current.
v
7T
A B
L Calculate the applied force required to
a move the bar to the left at a constant speed of
3 m/s. Assume the bar and rails have negligi-
ble resistance and friction. Neglect the mass
of the bar.
I
If a = 8.7 mm, L = 69 cm, and v = 33 m/s, Correct answer: 1344 N.
what is the potential difference, ∆V ≡ VA − Explanation:
VB ? The motional emf induced in the circuit is
Correct answer: 68.0483 mV. E = B ℓ v = (7 T) (8 m) (3 m/s)
Explanation: = 168 V .
chester (crc2876) – HW-10 – rodenborn – (92691) 2
From Ohm’s law, the current flowing through
the resistor is
E 168 V
I= = = 24 A , 0.8 T
R 7Ω
so the magnitude of the force exerted on the v
bar due to the magnetic field is
30◦
FB = I ℓ B = (24 A)(8 m)(7 T)
= 1344 N . Viewed from the side
What will be the terminal speed of the
To maintain the motion of the bar, a force rod? The acceleration due to gravitation is
must be applied on the bar to balance the 9.81 m/s2 .
magnetic force
Correct answer: 6.95153 cm/s.
F = FB = 1344 N .
Explanation:

004 (part 2 of 2) 10.0 points m
At what rate is energy dissipated in the resis- sliding rod
tor?
v
Correct answer: 4032 W.
Explanation:
The power dissipated in the resistor is R
B
P = I 2 R = (24 A)2 (7 Ω)
= 4032 W . Viewed from above

005 10.0 points Let : B = 0.8 T ,


R = 5 Ω,
A rod starts from rest at the top of an
inclined plane at t = 0. The rails have negligi- g = 9.81 m/s2 ,
ble resistance. There is a constant, vertically ℓ = 21 m , and
directed magnetic field of magnitude 0.8 T. θ = 30◦ .
21 m
0.6 kg
sliding rod
B
v v
θ
5Ω Viewed from the side
0.8 T
The induced emf is
Viewed from above E = B ℓ v cos θ .
chester (crc2876) – HW-10 – rodenborn – (92691) 3
Using Ohm’s law, the current in the circuit Ohm’s law
due to the induced emf is V
I=
R
E B ℓ v cos θ
I= = . Solution: With Ohm’s law, the emf induced
R R in one turn of coil is
Then the force due to the current is IR
E1 =
B 2 ℓ2 v cos θ n
Fm = I ℓ B = . (0.09 A) (11.1 Ω)
R =
(43 turns)
Applying Newton’s 2nd law to the rod, = 0.0232326 V ,
dv while with Faraday’s law, we get
m g sin θ − Fm cos θ = m
dt
d ΦB
Fm dv E1 =
g sin θ − cos θ = . dt
m dt d (A · B)
dv =
As the rod reaches the terminal speed, = dt
dt dB
0, and =A .
Fm dt
g sin θ = cos θ
m So, the rate at which magnetic field changes
B 2 ℓ2 v cos θ will be
g sin θ = cos θ .
mR dB E1
=
Thus dt A
(0.0232326 V)
m g R sin θ =
v= (0.004 m2 )
B 2 ℓ2 cos2 θ
(0.6 kg) (9.81 m/s2 ) (5 Ω) sin 30◦ = 5.80814 T/s .
=
(0.8 T)2 (21 m)2 cos2 30◦
100 cm 007 10.0 points
×
1m A circular conducting loop is held fixed in
= 6.95153 cm/s . a uniform magnetic field that varies in time
according to B(t) = B0 exp(−a t) where t is
in s, a is in s−1 and B is the field strength in
006 10.0 points T at t = 0. At t = 0, the emf induced in the
The plane of a rectangular coil of dimension loop is 0.0567 V . At t = 2.67 s, the emf is
5 cm by 8 cm is perpendicular to the direction 0.0161 V.
of a magnetic field B. The coil has 43 turns Find a.
and a total resistance of 11.1 Ω.
At what rate must the magnitude of B Correct answer: 0.471519 s−1 .
change in order to induce a current of 0.09 A Explanation:
in the windings of the coil? Basic concept Faraday’s Law
Correct answer: 5.80814 T/s. dB
E =A·
Explanation: dt
Basic Concepts: Faraday’s Law of Induc- Solution: Since the emf is
tion
d ΦB dB
E =− E =A ,
dt dt
chester (crc2876) – HW-10 – rodenborn – (92691) 4
since only the magnetic field is changing and So, from Faraday’s law, the magnitude of E
can be calculated directly
dB
= −a B exp (−at) , d ΦB
dt E=
dt
we have 2 equations from the 2 different times. ~ · B)
d (A ~
They are =n
dt
dB
at t = 0 , = n (π r 2 )
dt
−a A B = 0.0567 V , and = (21.2 turns) (0.0314159 m2 ) (1.34339 T/s)
at t = 1.5 s , = 0.89472 V .
−a A B exp[−a (2.67 s)] = 0.0161 V .

Dividing the second equation by the first 009 10.0 points


and then taking the natural logarithm, we A 39.5 turns circular coil with radius 1.93 cm
have and resistance 7.57 Ω is placed in a magnetic
  field directed perpendicular to the plane of
(0.0161 V) the coil. The magnitude of the magnetic field
− ln
(0.0567 V) varies in time according to the expression
a= = 0.471519 s−1 .
(2.67 s)
B = a 1 t + a 2 t2 ,

008 10.0 points where a1 = 0.0129 T/s, a2 = 0.0655 T/s2 are


Given: A tightly wound circular coil has constants, time t is in seconds and field B is
21.2 turns , each of radius 0.1 m . The uniform in Tesla.
magnetic field is in a direction perpendicular Find the magnitude of the induced emf in
to the plane of the coil. The field increases the coil at t = 7.96 s.
linearly from 0 T to 0.579 T in a time of
0.431 s . Correct answer: 0.0487962 V.
What is the magnitude of E induced in the Explanation:
windings of the coil?

Correct answer: 0.89472 V. Let : n = 39.5 turns ,


Explanation: r = 1.93 cm = 0.0193 m ,
Basic Concepts: Faraday’s Law of Induc- R = 7.57 Ω ,
tion a1 = 0.0129 T/s ,
Solution: a2 = 0.0655 T/s2 , and
d ΦB
E =− t = 7.96 s .
dt
B is uniform and linearly increased, so The area of the circular coil is

dB ∆B A = π r2
= = π (0.0193 m)2
dt ∆t
B2 − B1 = 0.00117021 m2 ,
=
t
(0 T) − (0.579 T) so from Faraday’s law,
=
(0.431 s) d ΦB
= 1.34339 T/s . E = −n
dt
chester (crc2876) – HW-10 – rodenborn – (92691) 5
dAB dr
= −n
dt
dB
= −n A
dt
= −n A (a1 + 2 a2 t)
= −(39.5 turns) (0.00117021 m2 ) I r b
× [0.0129 T/s + 2 (0.0655 T/s2 ) (7.96 s)]
= −0.0487962 V ,

c a

From Ampère’s law, the strength of the


magnetic field created by the current-carrying
which has a magnitude of 0.0487962 V . wire at a distance r from the wire is (see
figure.)
010 10.0 points
A rectangular loop located a distance from a µ0 I
B= ,
long wire carrying a current is shown in the 2πr
figure. The wire is parallel to the longest side so the field varies over the loop and is directed
perpendicular to the page. Since B ~ is parallel
of the loop.
~
to dA, the magnetic flux through an area
element dA is
Z
Φ ≡ B dA
26.4 cm
0.019 A

µ0 I
Z
= dA .
2πr
Note: B ~ is not uniform but rather depends on
6.04 cm

r, so it cannot be removed from the integral.


3.61 cm In order to integrate, we express the area
element shaded in the figure as dA = b dr.
Find the total magnetic flux through the Since r is the only variable that now appears
loop. in the integral, we obtain for the magnetic
flux
Z a+c
Correct answer: 4.70053 × 10−10 Wb. µ0 I dr
ΦB = b
2π c r
µ0 I b a+c
= ln r

Explanation: 2π c 
µ0 I b a+c
= ln
2π c
 
µ0 (0.019 A)(0.264 m) a+c
= ln
2π c
µ0 (0.019 A)(0.264 m)
= (0.468554)
Let : c = 6.04 cm , 2π
a = 3.61 cm , = 4.70053 × 10−10 Wb .
b = 26.4 cm , and
I = 0.019 A . 011 10.0 points
chester (crc2876) – HW-10 – rodenborn – (92691) 6
A magnetic field that is decreasing with time
is directed out of the page and passes through
a circular loop of wire in the plane of the page. A
Which of the following is true of the induced
current in the wire loop? v
B

L
1. It is directed into the page. ω
+q C

2. It is zero in magnitude.
O
3. It is counterclockwise in direction. cor-
rect B B

4. It is clockwise in direction.

5. It is directed out of the page.

The direction of the magnetic force on the


positive charge +q at a fixed point C due
to the magnetic field, and the relationship
between VO and VA are respectively given by

1. radially outward, VO > VA .

2. in the plane, VO = VA .

3. radially outward, VA > VO .

4. radially inward, VO = VA .
Explanation:
5. radially outward, VO = VA .
Lenz’s law can be used to judge the direc-
tion of the induced current, which states that
6. radially inward, VA > VO .
the polarity of the induced emf is such that
it tends to produce a current that will cre-
ate a magnetic flux to oppose the change in 7. opposite to the direction of ~v , VO = VA .
magnetic flux through the loop.
8. radially inward, VO > VA . correct
In this problem, since the magnetic field is
decreasing, the induced magnetic field should
be in the same direction, namely pointing out 9. out of the plane, VO = VA .
of the plane of the page, so the direction of the
10. in the direction of ~v , VO = VA .
induced current must be counterclockwise.

012 (part 1 of 2) 10.0 points

A metal bar with length OA = L is rotating


in a clockwise manner about the point O with
a constant angular velocity ω. There is a
constant magnetic field B directed out of the
paper. Explanation:
chester (crc2876) – HW-10 – rodenborn – (92691) 7

A 3255
8. |VO − VA | = V
4
465
B 9. |VO − VA | = V
4
16275
+q 10. |VO − VA | = V
2
FB v Explanation:

Let : OC = R = 8 m ,
L = m,
O
v = 93 m/s , and
From the figure above we can see that the
B = 7 T.
force on +q is directed radially inward . The
sign of the charge can reverse the direction of The induced electric field at a point r is
the force FB . This may be obtained from the given by
equation ~ = vB = rωB,
Eind = |~v × B|
~ = +q ~v × B
F ~.
where the angular velocity is
Because of this magnetic force, the positive
charges begin to accumulate at O (or nor- v (93 m/s) 93
ω= = = rad/s ,
mally, the negative charges begin to accumu- R (8 m) 8
late at A), producing an electric field that so the magnitude of the induced E is
points from O to A. Hence, VO > VA . Z R
1
Third of eight versions. Eind = r ω B dr = ω B R2
0 2

1 93
013 (part 2 of 2) 10.0 points = rad/s (7 T) (10 m)2
The length of the bar is 10 m and the magni- 2 8
tude of the magnetic field is 7 T . 16275
= V .
The speed of the bar at point C is 93 m/s , 4
and the length of OC = 8 m .
Determine the magnitude of the potential 014 (part 1 of 4) 10.0 points
difference |VO − VA | . An air-core solenoid consists of 200 turns of
wire wound on a form that is 76 cm long and
1302
1. |VO − VA | = V has a inner diameter of 4 cm.
5 The permeability of free space is µ0 = 4π ×
3255 10−7 T·m/A.
2. |VO − VA | = V
8 If a current of 18 A is established in this
16275 solenoid, what is its self-inductance?
3. |VO − VA | = V correct
4
7812 Correct answer: 0.0831125 mH.
4. |VO − VA | = V
5 Explanation:
2604
5. |VO − VA | = V
5 Let : N = 200 ,
6. |VO − VA | = 16275 V ℓ = 76 cm = 0.76 m ,
3255 d = 4 cm = 0.04 m , and
7. |VO − VA | = V µ0 = 4π × 10−7 T · m/A.
2
chester (crc2876) – HW-10 – rodenborn – (92691) 8
The self-inductance of a solenoid is
Correct answer: 14.098 J/m3 .
µ0 N 2 µ0 N 2 π d 2
L= A=
ℓ ℓ 4 Explanation:
(1.25664 × 10 N/A2 ) (200)2
−6
=
(0.76 m)
Let : B = 0.00595249 T .
(0.04 m)2
×π
4 The energy density in the solenoid is
= 8.31125 × 10−5 H
= 0.0831125 mH . B2
u=
2 µ0
(0.00595249 T)2
015 (part 2 of 4) 10.0 points =
What is the magnetic field at its center? 2 (1.25664 × 10−6 N/A2 )
= 14.098 J/m3 .
Correct answer: 0.00595249 T.
Explanation:
018 10.0 points
A transformer consists of two coils of wire
Let : I = 18 A . wound on a common toroidal iron core. The
The magnetic field of a solenoid is mutual inductance of the pair is 370 mH and
the current in the first coil decreases from
B = µ0 n I 32 A to 0 in 0.7 s.
= (1.25664 × 10−6 N/A2 ) What is the induced emf in the second coil?
 
200
× (18 A) Correct answer: 16.9143 V.
0.76 m
Explanation:
= 0.00595249 T .

016 (part 3 of 4) 10.0 points Let : M = 370 mH = 0.37 H ,


How much energy is stored in it? I = 32 A , and
∆t = 0.7 s .
Correct answer: 0.0134642 J.
Explanation: EMF due to mutual induction is
∆I
E = −M
Let : L = 8.31125 × 10 −5
H. ∆t
32 A
The energy stored in a solenoid is = (0.37 H)
0.7 s
1 = 16.9143 V .
U= L I2
2
1
= (8.31125 × 10−5 H) (18 A)2
2 019 10.0 points
= 0.0134642 J .
A long solenoid carries a current 30 A.
Another coil (of larger diameter than the
017 (part 4 of 4) 10.0 points solenoid) is coaxial with the center of the
What is the energy density in the solenoid? solenoid, as in the figure.
chester (crc2876) – HW-10 – rodenborn – (92691) 9

4m and the flux Φ12 through coil 1 due to coil 2 is


3m Φ12 = B2 A2 , so the mutual inductance is
N2 Φ12 N1 Φ21
M= =
19.5 cm I1 I2
N1 N2 A2 I2 N1 N2 A2
= µ0 = µ0
9.5 cm ℓ 2 I2 ℓ2
(110)(4000)
= (4 π × 10−7 T · m/A)
4m
2
× (0.0283529 m )
Find the mutual inductance of the system.
The outside solenoid has 110 turns and the = 0.00391922 H .
inside solenoid has 4000 turns. The perme-
ability of free space is 4 π × 10−7 T · m/A.
020 10.0 points
An inductor of 140 turns has a radius of 5 cm
Correct answer: 0.00391922 H.
and a length of 28 cm.
Explanation: The permeability of free space is
1.25664 × 10−6 N/A2.
Find the energy stored in it when the cur-
Let : r1 = 19.5 cm = 0.195 m , rent is 0.4 A.
r2 = 9.5 cm = 0.095 m ,
N1 = 110 , Correct answer: 5.52698 × 10−5 J.
N2 = 4000 , Explanation:
ℓ1 = 3 m,
ℓ2 = 4 m, Let : r = 5 cm = 0.05 m ,
I2 = 30 A , and ℓ = 28 cm = 0.28 m ,
µ0 = 4 π × 10−7 T · m/A . N = 140 turns ,
I = 0.4 A , and
ℓ2 µ0 = 1.25664 × 10−6 N/A2 .
ℓ1
The inductance is
A π r2
L = µ0 N 2 = µ0 N 2
A1 A2 ℓ ℓ
= (1.25664 × 10 N/A2 )
−6

π (0.05 m)2
× (140 turns)2
0.28 m
= 0.000690872 H .
For the outside solenoid,
The energy stored in an inductor is
A1 = π r12 = π(19.5 cm)2 = 0.119459 m2
1
U= L I2
and for the inside solenoid, 2
1
= (0.000690872 H)(0.4 A)2
A2 = π r12 = π(9.5 cm)2 = 0.0283529 m2 . 2
= 5.52698 × 10−5 J .
N2 I2
B 2 = µ0
ℓ2 021 10.0 points
chester (crc2876) – HW-10 – rodenborn – (92691) 10
A long cylindrical wire of radius 20 cm carries µ0 I 2 a 3
Z
a current of 78 A uniformly distributed over = r dr
4π a4 0
its cross-sectional area. µ0 I 2 a 4 µ0 I 2
The permeability of free space is 4π × = =
4π a4 4 4π 4
10−7 T · m/A.
(78 A)2
Find the magnetic energy per unit length = (1 × 10−7 T · m/A)
within the wire. 4
= 0.0001521 J/m .
Correct answer: 0.0001521 J/m.
Explanation: The magnetic energy per unit length within
the wire is independent of the radius of the
cylinder a and depends only on the total cur-
a rent.
r
dr 022 (part 1 of 2) 10.0 points
An inductor that has an inductance of 20 H
and a resistance of 32 Ω is connected across a
70 V battery.
Let : a = 20 cm , What is the rate of increase of the current
at 0 s.
I = 78 A , and
µ0 = 4π × 10−7 T · m/A . Correct answer: 3.5 A/s.
Use Ampere’s law, the magnetic field inside
Explanation:
the wire at a distance r < a from its center is
µ0
B= IC Let : L = 20 H ,
2πr
µ0 πr 2 R = 32 Ω ,
= I
2 π r πa2 ε = 70 V , and
µ0 r I t = 0 s.
= .
2 πa2
The magnetic energy within the cylindrical The current in an RL circuit is
annulus is ε  
B2 I= 1 − e−R t / L .
d Um = Vannulus R
2 µ0
B2 The rate of change of the current is
= 2 π r ℓ dr
2 µ0
dI d ε ε 
B2 = − e−R t / L
= π r ℓ dr dt dt R R 
µ0 ε −R t / L −R


µ0 r I
2
π r ℓ dr =− e
= R L
2πa 2 µ0 ε −R t / L
2 3
= e .
µ0 I r ℓ dr L
= .
4π a4
At t = 0 s , the rate is
Thus the magnetic energy per unit length
within the wire is
Z a dI 70 V −(32 Ω) (0 s) / (20 H)
= e
d Um dt 20 H
Um 0
= = 3.5 A/s .
ℓ ℓ
chester (crc2876) – HW-10 – rodenborn – (92691) 11
which can be solved for t as
023 (part 2 of 2) 10.0 points  
What is the rate of increase of the current at I0
t = τ ln
0.7 s. If
 
16 A
Correct answer: 1.14198 A/s. = (0.0005 s) ln
5.76 A
Explanation: = 0.000510826 s .
Using the same formula derived in Part 1, the
rate of change of the current at t = 0.7 s is
dI 70 V −(32 Ω) (0.7 s) / (20 H) 025 (part 1 of 2) 10.0 points
= e A 26 V battery is connected in series with a
dt 20 H
resistance of 6.5 Ω and an inductance of 2.4 H.
= 1.14198 A/s . Find the energy stored in the inductor when
the current reaches its maximum value.
024 10.0 points
Correct answer: 19.2 J.
A certain circuit consists of an inductor of
50 mH in series with a resistor of 100 Ω. At Explanation:
one moment, the current in the circuit is 16 A,
and decreasing.
How long will it take for the current to fall Let : V = 26 V ,
to 5.76 A? R = 6.5 Ω , and
L = 2.4 H .
Correct answer: 0.000510826 s.
Explanation: The maximum current is

Let : L = 50 mH , V
Imax = ,
R
R = 100 Ω ,
I0 = 16 A , and so
If = 5.76 A . 1 LV2
U= L Imax2 =
The time constant τ of a circuit gives the 2 2 R2
time required for the current to fall to 0.37 (2.4 H) (26 V)2
=
times its initial value. For an RL-circuit, 2 (6.5 Ω)2
L = 19.2 J .
τ= .
R
If 026 (part 2 of 2) 10.0 points
Here you can easily check that = 0.37, so
I0 Find the energy stored in the inductor one
that the decay time to If is given by time constant after the switch is closed.
0.05 H
τ= Correct answer: 7.67187 J.
100 Ω
= 0.0005 s . Explanation:
If you did not notice this, then note that in
general the current at time t in such a circuit Let : V = 26 V ,
is given by
R = 6.5 Ω , and
−t R/L
I(t) = I0 e = I0 e
−t/τ
, L = 2.4 H .
chester (crc2876) – HW-10 – rodenborn – (92691) 12
At t = τ ,
   
1 e−1 V
I = 1− Imax = ,
e e R

and
1 L (e − 1)2 V 2
W = L I2 =
2 2 e2 R2
(2.4 H) (e − 1)2 (26 V)2
=
2 e2 (6.5 Ω)2
= 7.67187 J .
Version One – Homework 39 – Ashley Smith 1

This print-out should have 246 questions,


check that it is complete. Multiple-choice
questions may continue on the next column
a b R c
or page: find all choices before making your
selection. The due time is Central time. R
Alternating Voltage R
33:02, trigonometry, numeric, > 1 min. The light bulb glows most brightly at:
001 (part 1 of 2) 10 points
The alternating voltage of a generator is rep- 1. very low frequencies
resented by the equation
1
E = E0 sin ω t , 2. the frequency ω = √
LC
where E is in volts, E0 = 240 V, ω = 500 rad/s, 3. very high frequencies correct
and t is in seconds.
Explanation:
Find the frequency of the electric potential
Since the brightness of the bulb is propor-
E.
tional to the power dissipated in it
Correct answer: 79.5775 Hz.
Explanation: 2
P = Irms R,
We compare the given equation
in order to get maximum brightness we should
E = (240 V) sin(500 rad/s) t
adjust frequency so that the rms current Irms
to the general form for such an equation, through the bulb is the largest. Remember-
1
ing that the capacitive reactance XC =
E = Emax sin ω t. ωC
is inversely proportional to ω, and the induc-
By comparison, we see that tive reactance XL = ωL is proportional to
it, we conclude that the rms current through
ω = 2 π f = 500 rad/s . the bulb is the largest when our inductive
reactance is very large, and our capacitive
Therefore
reactance is very small, i.e. at very high fre-
500 rad/s quencies.
f= = 79.5775 Hz .

Electric Stove
002 (part 2 of 2) 10 points 33:02, trigonometry, numeric, > 1 min.
Find the voltage output of the source. 004 (part 1 of 2) 10 points
Correct answer: 169.706 V. An electric stove is connected to an AC source
Explanation: with rms voltage of 240 V.
The rms voltage is Find the maximum voltage across one of
the stove’s elements when it is operating.
Correct answer: 339.411 V.
Emax 240 V Explanation:
Erms = √ = √ = 169.706 V .
2 2
1
Vrms = √ Vmax
Bulb in an AC Circuit 2
33:02, calculus, multiple choice, > 1 min. √
Vmax = 2 Vrms = 339.411 V
003 (part 1 of 1) 10 points
The emf drives the circuit below at frequency
ω. 005 (part 2 of 2) 10 points
Version One – Homework 39 – Ashley Smith 2

The resistance of the operating element is Looking at the figure, we see that the next
11 Ω. value of t for which
What rms current flows through it?
Correct answer: 21.8182 A. v = (0.25) Vmax
Explanation:
Vrms occurs at
Irms = = 21.8182 A T
R t2 = − t1 ,
2
Generator Output
where T is the period. The frequency is given
33:02, calculus, numeric, > 1 min.
by
006 (part 1 of 2) 10 points
The output of a generator is given by v = ω
Vmax sin ωt. f=

If after 0.01 s, the output v is 0.25 times 25.268 rad/s
Vmax , what is the angular velocity ω of the =

generator? = 4.02153 Hz .
Correct answer: 25.268 rad/s.
Explanation:
We are given that The period is given by

v = (0.25) Vmax 1
T =
at t = 0.01 s . Therefore f
= 0.248662 s .
(0.25) Vmax = Vmax sin[ω (0.01 s)]
ω (0.01 s) = sin−1 (0.25) Therefore, the time required is
= 0.25268 rad .
T
So t2 = − t1
2
(0.25268 rad) = 0.114331 s .
ω= = 25.268 rad/s .
(0.01 s)

007 (part 2 of 2) 10 points


Generator Output 02
What is the next value of t for which the
33:02, calculus, numeric, > 1 min.
output v is 0.25 times Vmax ?
008 (part 1 of 1) 10 points
Correct answer: 0.114331 s.
The output of a generator is given by
Explanation:
Vmax
v(t) = a sin(ω t) ,

where a = 200 V.
Find the rms current in the circuit when
θ this generator is connected to a 100 Ω resis-
0 tor.
t1 Correct answer: 1.41421 A.
t1 Explanation:
t2 Comparing this expression for the voltage
T output with the general form,
2
v(t) = Vmax sin(ω t) ,
Version One – Homework 39 – Ashley Smith 3

we see that Vmax = 200 V . Thus, the rms Basic Concept:


voltage is √
2 ∆Imax
∆Irms = ∆Imax = √
Vmax 2 2
Vrms = √
2 Solution:
200 V √
= √ ∆Imax = ∆Irms 2
2 √
= 141.421 V . = (4.8 A) 2
= 6.78823 A
The calculated rms voltage can be used with
Ohm’s law to find the rms current in the
circuit: 011 (part 3 of 3) 10 points
c) What is the maximum value for potential
Vrms difference?
Irms =
R Correct answer: 169.706 V.
141.421 V Explanation:
=
100 Ω Basic Concept:
= 1.41421 A . √
2 ∆Vmax
∆Vrms = ∆Vmax = √
2 2
Holt SF 22C 01 Given:
33:02, highSchool, numeric, > 1 min. ∆Vrms = 120 V
009 (part 1 of 3) 10 points
Solution:
An rms potential difference of 120 V is placed √
across a light bulb with a resistance of 25 Ω. ∆Vmax = ∆Vrms 2
a) What is the rms current in the light √
= (120 V) 2
bulb?
= 169.706 V
Correct answer: 4.8 A.
Explanation:
Basic Concept: Holt SF 22C 02
33:02, highSchool, numeric, < 1 min.
∆Vrms = ∆Irms R 012 (part 1 of 1) 10 points
The current in an ac circuit is measured with
Given: an ammeter. The meter gives a reading of 5.5
∆Vrms = 120 V A.
R = 25 Ω Calculate the maximum ac current.
Correct answer: 7.77817 A.
Solution: Explanation:
Basic Concept:
∆Vrms √
∆Irms =
R 2 Imax
120 V Irms = Imax = √
= 2 2
25 Ω
= 4.8 A Given:
Irms = 5.5 A
Solution:
010 (part 2 of 3) 10 points √
b) What is the maximum value for current? Imax = Irms 2

Correct answer: 6.78823 A. = (5.5 A) 2
Explanation: = 7.77817 A
Version One – Homework 39 – Ashley Smith 4

33:02, highSchool, numeric, > 1 min.


Holt SF 22C 03 015 (part 1 of 3) 10 points
33:02, highSchool, numeric, > 1 min. An audio amplifier provides an alternating
013 (part 1 of 2) 10 points rms potential difference of 15.0 V. A loud-
A toaster is plugged into a source of alternat- speaker connected to the amplifier has a re-
ing potential difference with an rms value of sistance of 10.4 Ω.
110 V. The heating element is designed to a) What is the rms current in the speaker?
convey a current with a maximum value of Correct answer: 1.44231 A.
10.5 A. Explanation:
a) Find the rms current in the heating ele- Basic Concept:
ment.
Correct answer: 7.42462 A. ∆Vrms = ∆Irms R
Explanation:
Basic Concept: Given:
∆Vrms = 15.0 V

2 Imax R = 10.4 Ω
Irms = Imax = √
2 2 Solution:
Given: ∆Vrms
Imax = 10.5 A ∆Irms =
R
Solution: 15 V
=
10.4 Ω
Imax = 1.44231 A
Irms = √
2
10.5 A
= √ 016 (part 2 of 3) 10 points
2 b) What is the maximum value for current?
= 7.42462 A
Correct answer: 2.03973 A.
Explanation:
Basic Concept:
014 (part 2 of 2) 10 points
b) Find the resistance of the heating element. √
2 ∆Imax
Correct answer: 14.8156 Ω. ∆Irms = ∆Imax = √
2 2
Explanation:
Basic Concept: Solution:

∆Vrms = Irms R ∆Imax = ∆Irms 2

= (1.44231 A) 2
Given:
= 2.03973 A
Vrms = 110 V
Solution:
017 (part 3 of 3) 10 points
∆Vrms c) What is the maximum value for potential
R=
Irms difference?
110 V Correct answer: 21.2132 V.
=
7.42462 A Explanation:
= 14.8156 Ω Basic Concept:

2 ∆Vmax
∆Vrms = ∆Vmax = √
Holt SF 22C 04 2 2
Version One – Homework 39 – Ashley Smith 5

Solution:
√ Holt SF 22C 06
∆Vmax = ∆Vrms 2 33:02, highSchool, numeric, < 1 min.

= (15 V) 2 020 (part 1 of 1) 10 points
= 21.2132 V The largest potential difference that can be
placed across a certain capacitor at any in-
stant is 451 V.
Holt SF 22C 05 a) What is the largest rms potential differ-
33:02, highSchool, numeric, > 1 min. ence that can be placed across the capacitor
018 (part 1 of 2) 10 points without damaging it?
An ac generator has a maximum potential Correct answer: 318.905 V.
difference output of 155 V. Explanation:
a) Find the rms potential difference out- Basic Concept:
put. √
Correct answer: 109.602 V. 2 ∆Vmax
∆Vrms = ∆Vmax = √
Explanation: 2 2
Basic Concept: Given:
√ ∆Vmax = 451 V
2 ∆Vmax
∆Vrms = ∆Vmax = √
2 2 Solution:
∆Vmax
Given: ∆Vrms = √
∆Vmax = 155 V 2
451 V
Solution: = √
2
∆Vmax = 318.905 V
∆Vrms = √
2
155 V
= √ Holt SF 22Rev 25
2 33:02, highSchool, numeric, < 1 min.
= 109.602 V 021 (part 1 of 1) 10 points
The rms potential difference across high-
voltage transmission lines in Great Britain
019 (part 2 of 2) 10 points
is 220000 V.
b) Find the rms current in the circuit when
What is the maximum potential differ-
the generator is connected to a 53 Ω resistor.
ence?
Correct answer: 2.06795 A.
Correct answer: 311127 V.
Explanation:
Explanation:
Basic Concept:
Basic Concept:
∆Vrms = ∆Irms R √
2 ∆Vmax
∆Vrms = ∆Vmax = √
Given: 2 2
R = 53 Ω
Given:
Solution: Vrms = 220000 V
∆Vrms Solution:
∆Irms = √
R
109.602 V ∆Vmax = ∆Vrms 2

= = 2 (220000 V)
53 Ω
= 2.06795 A = 311127 V
Version One – Homework 39 – Ashley Smith 6
0.909 A
= √
Holt SF 22Rev 26 2
33:02, highSchool, numeric, > 1 min. = 0.64276 A
022 (part 1 of 2) 10 points
The maximum potential difference across cer-
tain heavy-duty appliances is 340 V. The 025 (part 2 of 3) 10 points
total resistance of an appliance is 120 Ω. b) What is the rms potential difference across
a) Find the rms potential difference across the bulb’s filament?
the appliance. Correct answer: 116.982 V.
Correct answer: 240.416 V. Explanation:
Explanation: Basic Concept:

023 (part 2 of 2) 10 points ∆Vrms = ∆Irms R


b) Find the rms current in the appliance.
Correct answer: 2.00347 A. Given:
Explanation: R = 182 Ω
Basic Concept:
Solution:
∆Vrms = ∆Irms R
∆Vrms = (0.64276 A)(182 Ω)
Given: = 116.982 V
R = 120 Ω
Solution:
∆Vrms 026 (part 3 of 3) 10 points
∆Irms = c) How much power does the light bulb use?
R
240.416 V Correct answer: 75.1916 W.
= Explanation:
120 Ω
= 2.00347 A Basic Concept:

P = Irms 2 R
Holt SF 22Rev 27
33:02, highSchool, numeric, > 1 min. Solution:
024 (part 1 of 3) 10 points
The maximum current that can pass through P = (0.64276 A)2 (182 Ω)
a light bulb filament is 0.909 A when its resis- = 75.1916 W
tance is 182 Ω.
a) What is the rms current conducted by
the filament of the bulb? Holt SF 22Rev 28
Correct answer: 0.64276 A. 33:02, highSchool, numeric, > 1 min.
Explanation: 027 (part 1 of 2) 10 points
Basic Concept: A 996 W hair dryer is designed to carry a
√ maximum current of 11.8 A.
2 Imax a) How large is the rms current in the hair
Irms = Imax = √
2 2 dryer?
Given: Correct answer: 8.34386 A.
∆Imax = 0.909 A Explanation:
Basic Concept:
Solution:

∆Imax 2 Imax
∆Irms = √ Irms = Imax = √
2 2 2
Version One – Homework 39 – Ashley Smith 7

Given: so the resistance is:


Imax = 11.8 A 2
Vrms V2
R= = max = 192.667 Ω
Solution: Pav 2 Pav
Imax
Irms = √ Resistive AC Circuit
2 33:02, trigonometry, numeric, > 1 min.
11.8 A 030 (part 1 of 2) 10 points
= √
2 An ac voltage of rms value 120 V and fre-
= 8.34386 A quency 50 Hz is connected across a light bulb
with a resistance of 100 Ω.
What is the rms current?
028 (part 2 of 2) 10 points Correct answer: 1.2 A.
b) What is the rms potential difference across Explanation:
the hair dryer? The rms current is:
Correct answer: 119.369 V. V
Explanation: I= = 1.2 A
R
Basic Concept:
the maximum current is:
P = Irms ∆Vrms √
Imax = 2 I = 1.69706 A
Given:
P = 996 W 031 (part 2 of 2) 10 points
Solution: What is the maximum current?
Correct answer: 1.69706 A.
P Explanation:
∆Vrms =
Irms
996 W RMS Current
=
(8.34386 A) 33:02, trigonometry, numeric, > 1 min.
= 119.369 V 032 (part 1 of 1) 10 points
A circuit contains only a 45 Ω resistor. An
AC voltage of peak value 320 V and frequency
Resistance of Light Bulb 55 Hz is applied to this circuit.
33:02, calculus, numeric, > 1 min. What is the rms current?
029 (part 1 of 1) 10 points Correct answer: 5.02831 A.
A lightbulb is connected to a 60 Hz power Explanation:
source having a maximum voltage of 170 V. Given:
What is the resistance of the light bulb that I = I0 sin(ωt)
uses an average power of 75 W? The rms current is
Correct answer: 192.667 Ω. p
Explanation: Irms = I2
The rms voltage is:
s
1 T
Z
= (I0 sin ωt)2 dt
Vmax T 0
Vrms = √
2 I0
=√
2
the average power is: 320 V
=√
2
Vrms 2 (45 Ω)
Pav = = 5.02831 A .
R
Version One – Homework 39 – Ashley Smith 8

Serway CP 21 01 036 (part 4 of 4) 10 points


33:02, trigonometry, numeric, < 1 min. d) Find the maximum current supplied.
033 (part 1 of 4) 10 points Correct answer: 28.2843 W.
An rms potential difference of 100 V is placed Explanation:
across a light bulb with a resistance of 5 Ω. The rms current is
a) Find the maximum voltage applied. √
Correct answer: 141.421 V. 2 ∆Imax
Explanation: ∆Irms = ∆Imax = √
2 2

Given : ∆Vrms = 100 V and


R = 5 Ω. √
∆Imax = ∆Irms 2

The rms voltage is = (20 A) 2

2 ∆Vmax = 28.2843 W
∆Vrms = ∆Vmax = √
2 2

√ Serway CP 21 02
∆Vmax = ∆Vrms 2 33:02, trigonometry, numeric, > 1 min.

= (100 V) 2 037 (part 1 of 2) 10 points
= 141.421 V What is the resistance of a lightbulb that uses
an average power of 75 W when connected to
a 60 Hz power source with an peak voltage of
034 (part 2 of 4) 10 points 170 V?
b) Find the rms current supplied.
Correct answer: 20 A. Correct answer: 192.667 Ω.
Explanation: Explanation:
The rms voltage is
∆Vrms = ∆Irms R Given : P = 75 W ,
f = 60 Hz , and :
Vmax = 170 V .
∆Vrms
∆Irms =
R The rms current is
100 V
=
5Ω Vmax
= 20 A Vrms = √
2

035 (part 3 of 4) 10 points The Power is


c) Find the power dissipated. 2
Correct answer: 2000 A. Vrms V2
P = = max
Explanation: R 2R
V 2
The power is
R = max
2P
P = ∆Irms ∆Vrms
(170 V)2
= = 192.667 Ω.
2 · 75 W
P = (20 A) (100 V)
= 2000 A 038 (part 2 of 2) 10 points
Version One – Homework 39 – Ashley Smith 9

What is the resistance of a 100 W bulb? What is the reading of the ammeter?
Correct answer: 144.5 Ω. Correct answer: 2.94628 A.
Explanation: Explanation:
The rms current is given by
Given : P = 100 W . Vrms
Irms =
The resistance is R
70.7107 V
(170 V)2 =
R= = 144.5 Ω. 24 Ω
2 · 100 W = 2.94628 A .

Serway CP 21 03
33:02, trigonometry, numeric, > 1 min. Serway CP 21 04
039 (part 1 of 2) 10 points 33:02, trigonometry, numeric, < 1 min.
An AC power supply produces a maximum 041 (part 1 of 3) 10 points
voltage of 100 V. This power supply is con- The figure shows three lamps connected to
nected to a 24 Ω resistor, and the current and a 120 V AC (rms) household supply voltage.
resistor voltage are measured with an ideal Find the rms current and resistance of each
AC ammeter and an ideal AC voltmeter, as in bulb.

100 V

A 150 Ω

150 Ω

100 Ω
24 Ω

V
120 V
What does the voltmeter read? Recall that
an ideal ammeter has zero resistance and an What is rms current?
ideal voltmeter has infinite resistance. Correct answer: 3.33333 Ω.
Explanation:
Correct answer: 70.7107 V.
Explanation:

Given : Vmax = 100 V and


R = 24 Ω . R1
The voltmeter and ammeter read the rms
R2

R3

value. The rms value of the voltage is given


by
Vmax
Vrms = √ ∆V
2
100 V
= √
2 Given : P1 = 150 Ω
= 70.7107 V . P2 = 150 Ω
P3 = 100 Ω and
040 (part 2 of 2) 10 points ∆V = 120 V .
Version One – Homework 39 – Ashley Smith 10

The total current is


Correct answer: 6.76379 W.
I = I 1 + I2 + I3 Explanation:
where I1 , I2 and I3 are currents in each bulb,
P
so using I =
∆V Given : Vrms = 15 V ,
P1 P2 P3 R = 8.2 Ω , and
I= + +
∆V ∆V ∆V Rspeaker = 10.4 Ω .
150 Ω + 150 Ω + 100 Ω
=
120 V The resistor R and the speaker are connected
= 3.33333 Ω . in series. Therefore, the equivalent resistance
of the circuit is given by
042 (part 2 of 3) 10 points
Req = R + Rspeaker
What is the resistance of the 150 Ω bulbs?
Correct answer: 96 A.
Explanation: The current in the circuit is given by
∆V 2
We have P = , so Vrms Vrms
R Irms = =
Req R + Rspeaker
∆V 2 (120 V)2 15 V
R1 = = = 96 A . =
P1 100 Ω 8.2 Ω + 10.4 Ω
= 0.806452 A .
043 (part 3 of 3) 10 points
What is the resistance of 100 Ω bulb? The average power dissipated in a resistor
Correct answer: 144 A. that carries an alternating current is given by
Explanation:
2
Pav = Irms R.
∆V 2 (120 V)2
R3 = = = 144 A .
P3 100 Ω In our case, R is the resistance of the speaker,
so
Serway CP 21 05
2
33:02, trigonometry, numeric, > 1 min. Pav = Irms Rspeaker
044 (part 1 of 1) 10 points = (0.806452 A)2 (10.4 Ω)
An audio amplifier, represented in the figure
by the AC source and the resistor R delivers = 6.76379 W .
alternating voltages at audio frequencies to
the speaker.
If the sources puts out an alternating volt- Toaster Heating Element
age of 15 V (rms) and resistance of 8.2 Ω, 33:02, trigonometry, numeric, > 1 min.
and the speaker is equivalent to a resistance 045 (part 1 of 2) 10 points
of 10.4 Ω, what is the time-averaged power If a toaster is plugged into a source of alter-
input to the speaker? nating voltage with an rms value of 110 V,
R an rms current of 7.5 A occurs in the heating
element.
What is the resistance of the toaster’s heat-
ing element?
Speaker Correct answer: 14.6667 Ω.
Explanation:
Version One – Homework 39 – Ashley Smith 11

An AC voltage of peak value 160 V and fre-


V 110 V quency 58 Hz is applied to the circuit. When
R= = = 14.6667 Ω.
I 7.5 A the voltage is 160 V, what is AC current?
Correct answer: 0 A.
046 (part 2 of 2) 10 points Explanation:
What is the average power input to the In an AC circuit containing only induc-
toaster? tance, the current lags the applied voltage by
Correct answer: 825 W. 90◦ . So when the voltage reaches its peak
Explanation: value, the current is 0.

Changing the Frequency


P = V I = (110 V)(7.5 A) = 825 W. 33:03, calculus, numeric, > 1 min.
050 (part 1 of 2) 10 points
When a particular inductor is connected to a
AC Current sinusoidal voltage with a 120 V amplitude, a
33:03, trigonometry, numeric, > 1 min. peak current of 3 A appears in the inductor.
047 (part 1 of 2) 10 points What is the maximum current if the fre-
A voltage of the form v = vmax sin(2πf t), quency of the applied voltage is doubled?
with frequency 60 Hz and voltage 200 V, is Correct answer: 1.5 A.
applied to a pure inductance of 0.2 H. T is Explanation:
the period. What is the current through the The current of the circuit is
circuit when t = T /2?
Vmax
Correct answer: 2.65258 A. Imax =
Explanation: XL
As we know, Vmax
=
ωL
XL = 2πf L when the frequency is doubled, the current is
= 2π(60)(0.2 H)
Vmax
= 75.3982 ΩΩ Inew =
Vmax 2ωL
imax = 1
XL = Imax = 1.5 A .
2
200 V
=
75.3982 Ω
= 2.65258 A 051 (part 2 of 2) 10 points
What is the inductive reactance at the new
i = −imax cos wt
frequency?
Correct answer: 80 Ω.
At t = /2, i1 = imax = 2.65258 A A,
Explanation:
and at t =, i2 = −imax = −2.65258 A A.
The inductive reactance is
048 (part 2 of 2) 10 points Vmax
XL,new =
What is the current through the circuit when Inew
t = T? (120 V)
Correct answer: −2.65258 A. =
(1.5 A)
Explanation: = 80 Ω .

AC Current 02
33:03, trigonometry, numeric, > 1 min. Coil and Iron Core
049 (part 1 of 1) 10 points 33:03, calculus, multiple choice, < 1 min.
A circuit contains only a(n) 0.15 H inductor. 052 (part 1 of 1) 10 points
Version One – Homework 39 – Ashley Smith 12

A light bulb which is visibly lit is connected The maximum current is


to a wall socket which provides an oscillating √
2 Vrms
voltage V0 sin ωt to the circuit. Imax =
X
√ L
2 (120 V)
=
(7.53982 Ω)
= 22.5079 A .

iron The current at time t is


I = Imax sin[ω t]
= (22.5079 A)
× sin[(376.991 rad/s) (0.005 s)]
= (22.5079 A) sin[108◦ ]
= 21.4063 A .
After a piece of iron is shoved into a coil of So the energy stored in the inductor is
wire in the circuit as shown, the light
1
E= L I2
1. is not affected. 2
1
= (20 mH) (21.4063 A)2
2. brightens. 2
= 4.58229 J .
3. dims. correct
Frequency and Current
Explanation:
33:03, calculus, numeric, > 1 min.
Solution: After the iron is inside the coil,
054 (part 1 of 2) 10 points
the magnetic field within the coil becomes
Given: The inductor is 310 mH, the max-
stronger, therefore, the inductance is bigger.
imum voltage is 130 V, and the inductive
Therefore resistance due to the inductance
reactance is 40 Ω.
becomes bigger. This effect will reduce the
310 mH
current so that the light from the bulb dims.

Emax = 130 V
Energy in an Inductor 03 S
33:03, calculus, numeric, > 1 min.
053 (part 1 of 1) 10 points E = Emax sin ωt
A 20 mH inductor is connected to a standard What is the current’s frequency in the in-
outlet (Vrms = 120 V and f = 60 Hz). ductor?
Determine the energy stored in the inductor Correct answer: 20.5361 Hz.
at t = 5 ms, assuming that this energy is zero Explanation:
at t = 0. The frequency of the current is the same as
Correct answer: 4.58229 J. that of the electric potential in the circuit.
Explanation: The inductive reactance is given by
The inductive reactance is
XL = ω L , so
XL
XL = ω L ω=
L
= 2πf L 40 Ω
= 2 π (60 Hz) (20 mH) =
0.31 H
= 7.53982 Ω . = 129.032 rad/s .
Version One – Homework 39 – Ashley Smith 13

For the frequency, we have Ih V XL(l) 2 π fl L f


= = = l ,
ω Il V XL(h) 2 π fh L fh
f=

129.032 rad/s or
=

= 20.5361 Hz . I l fl (10 A)(60 Hz)
fh = =
Ih 5A
= 120 Hz.
055 (part 2 of 2) 10 points
Calculate the maximum current at this fre-
quency.
Inductive RC Circuit
Correct answer: 3.25 A.
33:03, calculus, numeric, > 1 min.
Explanation:
058 (part 1 of 2) 10 points
The maximum current at 20.5361 Hz is
In the purely inductive circuit in the figure
given by
below, the inductance is 25 mH and the rms
Vmax voltage is 150 V.
Imax =
XL 25 mH
130 V
=
40 Ω Erms = 150 V
= 3.25 A . S

Inductance and Frequency E = Emax sin ωt


33:03, trigonometry, numeric, > 1 min. Find the inductive reactance if the angular
056 (part 1 of 2) 10 points velocity is 60 Hz.
In a purely inductive circuit, the rms voltage Correct answer: 9.42478 Ω.
is 120 V. Explanation:
If the rms current is 10 A at a frequency of First, recall that
60 Hz, calculate the inductance.
Correct answer: 31.831 mH. ω = 2πf
Explanation: = 376.991 s−1 .
V 120 V
XL = = = 12 Ω.
I 10 A Then we have
Thus, from XL = 2 π f L, we have
XL 12 Ω XL = ω L
L= = = (376.991 s−1 ) (0.025 H)
2πf 2 π (60 Hz)
= 0.031831 H = 31.831 mH. = 9.42478 Ω .

057 (part 2 of 2) 10 points


At what frequency is the rms current 5 A? 059 (part 2 of 2) 10 points
Correct answer: 120 Hz. Find the rms current in the circuit.
Explanation: Correct answer: 15.9155 A.
The ratio of the current at the unknown Explanation:
higher frequency to that at 60 Hz Hz is found We find the rms current
from
V Imax
I= as Irms = √ ,
XL 2
Version One – Homework 39 – Ashley Smith 14

from Therefore, at 15.5 ms , the current is given by


Vrms
Irms = ³ π´
XL I = (2.79833 A) sin ω (0.0155 s) −
150 V 2
= = −2.79522 A .
9.42478 Ω
= 15.9155 A .

Minimum Inductance
Inductor in an AC Circuit 33:03, calculus, numeric, > 1 min.
33:03, calculus, numeric, > 1 min. 061 (part 1 of 1) 10 points
060 (part 1 of 1) 10 points An inductor is connected to a 20 Hz power
For the circuit shown in the figure, the max- supply that produces a 50 V rms voltage.
imum voltage is 80 V, the angular repetition What inductance is needed to keep the
is 65 rev/s, and the inductor is 70 mH. instantaneous current in the circuit below
The switch is closed at t = 0 . 80 mA?
70 mH Correct answer: 7.03372 H.
Explanation:
Emax = 80 V The rms current is:
S
Imax
Irms = √
E = Emax sin ωt 2
Calculate the current in the inductor at
15.5 ms. and,
Correct answer: −2.79522 A. Vrms
(XL )min =
Explanation: Irms
so the minimum inductance is:
Let : Emax = 80 V ,
L = 70 mH , and (XL )min
Lmin =
ω = 2 π (65 rev/s) = 408.407 rad/s . 2πf

The inductive reactance is given by where f is the frequency of the power supply.
XL = ω L
= (408.407 rad/s) (0.07 H) Minimum Inductance 02
33:03, trigonometry, numeric, > 1 min.
= 28.5885 Ω . 062 (part 1 of 1) 10 points
In a purely inductive circuit driven by An inductor is connected to a 20 Hz power
supply that produces a 50 V rms voltage.
V = Vmax sin ω t , What minimum inductance is needed to
the current is given by keep the maximum current in the circuit be-
³ π´ low 80 mA?
I = Imax sin ωt − . Correct answer: 7.03372 H.
2
Explanation:
The maximum current is obtained from
Vmax
Imax = V 50 V
XL I= =
80 V XL XL
= 0.08 A
28.5885 Ω ≤ √ = 0.0565685 A,
= 2.79833 A . 2
Version One – Homework 39 – Ashley Smith 15

so
50 V 3. The voltage lags the current by 90◦ .
XL ≥ = 883.883 Ω.
0.0565685 A
4. The voltage lags the current by 180◦ .
XL = ω L = 2 π f L
gives 5. The current lags the voltage by 90◦ .
correct
XL
L= Explanation:
2πf
Suppose the voltage is
883.883 Ω

2 π (20 Hz)
V = Vmax sin(ωt).
= 7.03372 H.

Thus, the minimum inductance is 7.03372 H Then the current is


H.
I = −Imax cos(ωt),
Peak Current
33:03, trigonometry, numeric, > 1 min. which lags the voltage by 90◦ .
063 (part 1 of 1) 10 points
An inductor has a 54 Ω reactance at 60 Hz. RC and LC Circuits 05
What will be the peak current if this in- 33:03, highSchool, multiple choice, > 1 min.
ductor is connected to a 50 Hz source that 065 (part 1 of 2) 10 points
produces a 100 V rms voltage? Consider the following circuit.
Correct answer: 3.1427 A. After leaving the switch at the position “a”
Explanation: for a long time, move the switch from “a”
to “b”. There will be the usual LC circuit
XL 54 Ω oscillations.
L= = = 0.143239 H.
2πf 2 π (60 Hz) L
Thus, when f = 50 Hz and Vrms = 100 V ,
C
Vrms 100 V S b
Irms = =
XL 2 π (50 Hz)(0.143239 H)
= 2.22222 A . a
E
R
and
√ √
Imax = 2 Irms = 2 (2.22222 A) The maximum current will be given by
= 3.1427 A .
E √
1. Imax = LC .
R
Phases of an AC Circuit r
L
33:03, trigonometry, multiple choice, > 1 min. 2. Imax =E .
C
064 (part 1 of 1) 10 points √
An AC circuit contains only an inductor. 3. Imax = E LC .
Which of the following is correct? r
1
4. Imax =E .
1. The current and voltage are in phase. LC
E
2. The current lags the voltage by 180◦ . 5. Imax = .
R
Version One – Homework 39 – Ashley Smith 16

r
E L 9. A3 and B3
6. Imax = .
R C Explanation:
r
C At t = 0 left plate of C has q = +qmax ,
7. Imax = E . correct i = 0.
L
T
At t = left plate of C has q = 0, i is
r
E C 4
8. Imax = . clockwise.
R L
T
Explanation: At t = left plate of C has q = −qmax ,
2
Basic Concepts: RCL circuit i = 0.
3T
At t = left plate of C has q = 0, i is
1 2 1 2 1 4
L Imax = qmax = C E 2 counterclockwise.
2 2C 2 5T
So at t = , q is negative, i is counter-
r
C 8
⇒ Imax = E. clockwise.
L

RMS Current 03
066 (part 2 of 2) 10 points
33:03, trigonometry, numeric, > 1 min.
Consider the following statements:
067 (part 1 of 1) 10 points
A1. The current flow is counterclockwise.
The generator in a purely inductive ac circuit
A2. The current is zero.
has an angular frequency of 376.991 rad/s. If
A3. The current flow is clockwise.
the voltage is 140 V and the inductance is
B1. The charge on the left plate of C is
0.1 H, what is the rms current in the circuit?
positive.
Correct answer: 0.835857 A.
B2. The charge on the left plate of C is
Explanation:
zero.
First, we note that
B3. The charge on the left plate of C is
negative. 140 V
Let the time when the switch is moved from Vrms = √ = 98.9949 V.
2
a to b be at t = 0.
Which pair of choices below best describes The inductive reactance is:
5
the situation at t = T , T is the period of XL = 2 π f L = ω L
8
oscillations in LC circiut. = (376.991 rad/s)(0.1 H)
= 118.435 Ω,
1. A1 and B1
and
2. A1 and B2
Vrms 98.9949 V
Irms = = = 0.835857 A.
3. A1 and B3 correct XL 118.435 Ω

4. A2 and B1 RMS Current and Frequency


33:03, trigonometry, numeric, > 1 min.
5. A2 and B2 068 (part 1 of 2) 10 points
In a purely inductive circuit of 30 mH, the
6. A3 and B3 frequency is 60 Hz and the maximum voltage
is 200 V.
7. A3 and B1 Find the rms current in the circuit.
Correct answer: 12.5044 A.
8. A3 3nd B2 Explanation:
Version One – Homework 39 – Ashley Smith 17

The current, as a function of frequency, can


be expressed as 071 (part 2 of 2) 10 points
V At what angular frequency is the maximum
I= current 2.5 A?
XL Correct answer: 942.478 rad/s.
Vm Explanation:
=√
2 (2 π f L)
(200 V) Given : Imax = 2.5 A
=√
2 (2 π)(0.03 H) (60 Hz)
The inductive reactance is given by
= 12.5044 A .
V
XL = ,
069 (part 2 of 2) 10 points I2
Repeat for a frequency of 6000 Hz.
so the angular frequency is
Correct answer: 0.125044 A.
Explanation: XL V
If f = 6000 Hz, ω2 = =
L I2 L
(200 V) 100 V
I=√ =
2 (2 π)(0.03 H)(6000 Hz) (2.5 A)(0.0424413 H)
= 0.125044 A . = 942.478 rad/s .

Serway CP 21 14 Serway CP 21 15
33:03, calculus, numeric, > 1 min. 33:03, calculus, numeric, > 1 min.
070 (part 1 of 2) 10 points 072 (part 1 of 1) 10 points
In a purely inductive AC circuit, as in the An inductor has a 54 Ω reactance at 60 Hz.
figure, the maximum voltage is 100 V. What is the maximum current if this in-
If the maximum current is 7.5 A at 50 Hz, ductor is connected to a 50 Hz source that
find the inductance L. produces a 100 V rms voltage?
Correct answer: 0.0424413 H. Correct answer: 3.1427 A.
Explanation: Explanation:

Given : ∆Vmax = 100 V , Given : XL = 54 Ω ,


I1 = 7.5 A , and f = 60 Hz ,
f1 = 50 Hz . f 0 = 50 Hz , and
Vrms = 100 V .
The inductive reactance is given by
V The maximum voltage of the circuit is
XL = = ω L = 2 π f1 L
à √
I1 Vmax = Vrms 2
so
XL the inductive reactance is
L=
2 π f1
V XL = ω L ,
=
2 π f 1 I1
so we have
100 V
=
2 π (50 Hz)(7.5 A) ω0 f0
XL0 = XL = XL
= 0.0424413 H . ω f
Version One – Homework 39 – Ashley Smith 18

the maximum current is: Calculate the impedance.


√ Correct answer: 1520.43 Ω.
Vmax 2 Vrms f Explanation:
Imax = 0 =
X f 0 XL
√L
2 (100 V)(60 Hz) Given : R = 150 Ω .
=
(50 Hz)(54 Ω) For the impedance,
= 3.1427 A . q
Z = R2 + (XL − XC )2
Serway CP 21 20 = 1520.43 Ω .
33:03, trigonometry, numeric, > 1 min.
073 (part 1 of 5) 10 points
A series AC circuit contains a 150 Ω resistor, 076 (part 4 of 5) 10 points
an 250 mH inductor, a 2 µF capasitor, and a Calculate the maximum current.
generator with maximum voltage 50 Hz. Correct answer: 0.138119 A.
Calculate the inductive reactance. Explanation:
Correct answer: 78.5398 Ω.
Explanation: Given : ∆Vmax = 210 V .
The maximum current in the circuit is ob-
Given : L = 250 mH and tained from
f = 50 Hz . Vmax
Imax =
Z
The angular frequency is 210 V
=
ω = 2πf , 1520.43 Ω
= 0.138119 A .
so the inductive reactance is given by

XL = ω L = 2 π f L 077 (part 5 of 5) 10 points


= 2 π(50 Hz) (0.25 H) Calculate the phase angle in degrees. (Answer
= 78.5398 Ω . between −180◦ and +180◦ .)
Correct answer: −84.3382 ◦ .
Explanation:
074 (part 2 of 5) 10 points In an RLC circuit, the phase angle between
Calculate the capacitive reactance. the current and voltage is
Correct answer: 1591.55 Ω. µ ¶
Explanation: −1 XL − XC
φ = tan
R
µ ¶
Given : C = 2 µF . −1 78.5398 Ω − 1591.55 Ω
= tan
The capacitive reactance is given by 150 Ω

= −84.3382 .
1 1
XC = =
ωC 2πf C
1 AC Generator 02
=
2 π (50 Hz) (2 × 10−6 F) 33:04, calculus, numeric, > 1 min.
= 1591.55 Ω . 078 (part 1 of 1) 10 points
A variable-frequency ac generator with max-
imum voltage 18 V is connected across a
075 (part 3 of 5) 10 points 9.4 × 10−8 F capacitor. At what frequency
Version One – Homework 39 – Ashley Smith 19

should the generator be operated to provide a t = 0.00555556 s, assuming that at t = 0,


maximum current of 5 A? the energy stored in the capacitor is zero.
Correct answer: 470316 Hz. Correct answer: −31.9888 A.
Explanation: Explanation:
The maximum current is: The capacitive reactance is:
Vmax 1 1
Imax = = Vmax ω C = Vmax 2 π f C XC = =
XC ωC 2πf C
so the frequency of the generator is: the maximum current is:

Imax 2 Vrms
f= Imax =
Vmax 2 π C XC
Because the current leads the voltage across a
Capacitive AC Circuit capacitor by 90◦ , at time t,
33:04, trigonometry, numeric, > 1 min.
079 (part 1 of 1) 10 points I = Imax sin (ω t + 90◦ ) = −31.9888 A
The generator in a purely capacitive ac circuit
has an angular frequency of 376.991 rad/s. If
the voltage is 140 V and the capacitance is Capacitive AC Circuit 04
6 µF, what is the rms current in the circuit? 33:04, trigonometry, numeric, > 1 min.
Correct answer: 0.223921 A. 081 (part 1 of 1) 10 points
Explanation: A capacitor is placed parallel across a resis-
1 tive load in the circuit below.
XC = , where ω = 2 π f is the angular
ω
frequency. Thus,
1 E 0.1 µF 190 Ω
XC =
(376.991 rad/s)(6 × 10−6 F)
= 442.097 Ω .
What fraction of the incoming current will
Also, √ pass through the capacitor’s side to ground
Vm = 140 V = 2V ,
rather than through the load’s side when the
140 V frequency is 100 s−1 ?
V = √ = 98.9949 V .
2 Correct answer: 0.0119372 .
Thus, Explanation:

V
I= Let : R = 190 Ω ,
XC
98.9949 V C = 0.1 µF ,
= f = 100 s−1 , and
442.097 Ω
= 0.223921 A . ω = 2 π f = 628.319 rad/s .

The capacitive reactance is


Capacitive AC Circuit 02
33:04, calculus, numeric, > 1 min. 1
XC =
080 (part 1 of 1) 10 points 2πf C
A 1 mF capacitor is connected to a stan- 1
=
dard outlet ( Vrms = 120 V, f = 60 Hz ). 2 π (100 s−1 )(1 × 10−7 F)
Determine the current in the capacitor at = 15915.5 Ω .
Version One – Homework 39 – Ashley Smith 20

The current through the capacitor is IC =


V /XC , and the current through the load is V 9V
XC = = = 360 Ω,
IR = V /R. The current through the capacitor I 0.025 A
and resistor are 90◦ out of phase with one
q 1
another. The total current is IC2 + IR
2 . The ω=
XC C
fraction is 1
=
I R (360 Ω)(2.4 × 10−6 F)
q C =q
IC2 + IR
2 XC2 + R2 = 1157.41 rad/s,

190 Ω and
=p
(15915.5 Ω)2 + (190 Ω)2 ω
f=
= 0.0119372 . 2π
1157.41 rad/s
= = 184.207 Hz.

Capacitive Reactance
33:04, trigonometry, numeric, > 1 min. 084 (part 2 of 2) 10 points
082 (part 1 of 1) 10 points If the capacitor is replaced by an ideal coil
A certain capacitor in a circuit has a capaci- with an inductance of 0.16 H, what is the rms
tive reactance of 30 Ω when the frequency is current through the coil?
120 Hz. Correct answer: 48.6 mA.
What capacitive reactance does the capac- Explanation:
itor have at a frequency of 10000 Hz?
Correct answer: 0.36 Ω.
Explanation: XL = ω L = (1157.41 rad/s)(0.16 H)
The ratio of the capacitive reactance at the = 185.185 Ω,
higher frequency to that at the lower one is
XC(high) 2 π flow C flow V 9V
= = . I= =
XC(low) 2 π fhigh C fhigh XL 185.185 Ω
= 0.0486 A = 48.6 mA.
Thus,
flow
XC(high) = XC(low) RMS Current 02
fhigh 33:04, calculus, numeric, > 1 min.
µ ¶
120 Hz 085 (part 1 of 2) 10 points
= (30 Ω)
10000 Hz A 8 µF capacitor is connected to the terminals
= 0.36 Ω. of a 60 Hz generator whose rms voltage is
150 V.
Find the capacitive reactance.
Frequency and RMS Current Correct answer: 331.573 Ω.
33:04, trigonometry, numeric, > 1 min. Explanation:
083 (part 1 of 2) 10 points Using the equation
A(n) 2.4 µF capacitor is connected across an
1
alternating voltage with an rms value of 9 V. XC =
The rms current through the capacitance is ωC
25 mA. and the fact that
What is the source frequency?
Correct answer: 184.207 Hz. ω = 2πf
Explanation: = 376.991 s−1
Version One – Homework 39 – Ashley Smith 21

gives Explanation:
The time constant is
1
XC =
ωC
1 τ = R C = 220 s
=
(376.991 s ) (8 × 10−6 F )
−1

= 331.573 Ω .
089 (part 2 of 3) 10 points
What is the value of the current when the
086 (part 2 of 2) 10 points switch in the circuit has just been closed?
Find the rms current in the circuit. Correct answer: 3 × 10−6 A.
Correct answer: 0.452389 A. Explanation:
Explanation: The current is
Having found the capacitive reactance XC , we
V
easily find the rms current: I= = 3 × 10−6 A
R
Vrms
Irms =
XC
090 (part 3 of 3) 10 points
150 V
= What is the charge on the capacitor when the
331.573 Ω current is 2 × 10−6 A?
= 0.452389 A
Correct answer: 0.00022 c.
Explanation:
RMS Current 04 The voltage drop across the resistor is
33:04, trigonometry, numeric, > 1 min.
087 (part 1 of 1) 10 points Vr = IR = (2 × 10−6 A)(2 × 106 Ω) .
An AC voltage of peak value 80 V and fre-
quency 48 Hz is applied to a 15 µF capacitor. The voltage drop across the capacitor is then
What is the rms current?
Correct answer: 0.25591 A. Vc = V − V r = 2 V .
Explanation:
Hence the charge on the capacitor is
1 1
Xc = =
ωC 2π f C Q = Vc C = 0.00022 c .
So
µ ¶
1 vm
Irms = √ Serway CP 21 08
2 Xc
µ ¶ 33:04, calculus, numeric, > 1 min.
1
= √ 2π Vm f C 091 (part 1 of 2) 10 points
2 What maximum current is delivered to a
= 0.25591 A 2.2 µF capacitor when connected across a
North American outlet having rms voltage
120 V and frequency 60 Hz?
Series RC Circuit 02
Correct answer: 0.140751 A.
33:04, trigonometry, numeric, < 1 min.
Explanation:
088 (part 1 of 3) 10 points
A circuit consisting of a(n) 110 µF capacitor
in series with a(n) 2 MΩ resistor is connected
across a(n) 6 V battery. Given : C = 2.2 × 10−6 F ,
What is the time constant of the circuit? Vrms = 120 V , and
Correct answer: 220 s. f = 60 Hz .
Version One – Homework 39 – Ashley Smith 22

The rms current is


Vrms Serway CP 21 11
Irms = = Vrms 2 π f C . 33:04, trigonometry, numeric, > 1 min.
XC
and the maximum current is 094 (part 1 of 1) 10 points
√ √ What value of capacitor must be inserted
Imax = 2 Irms = 2 Vrms 2 π f C . in a 60 Hz circuit in series with a generator
Therefore, with the North American outlet, of 170 V maximum output voltage to produce
the maximum current is given by an rms current output of 0.75 A?

Imax = 2 (120 V) 2 π Correct answer: 16.5499 µF.
Explanation:
× (60 Hz) (2.2 × 10−6 F)
= 0.140751 A .
Given : Vmax = 170 V ,
092 (part 2 of 2) 10 points f = 60 Hz , and
For the same capacitor, calculate the max-
I = 0.75 A .
imum current for a European outlet having
rms voltage 240 V and frequency 50 Hz.
Correct answer: 0.234584 A. The rms voltage is
Explanation:
Vmax
Given : Vrms = 240 V and Vrms = √
2
f = 50 Hz .

Imax = 2 Vrms 2 π f C and the inductive rectance is

= 2 (240 V) 2 π
× (50 Hz) (2.2 × 10−6 F) V Vmax
XC = = √
= 0.234584 A . I 2I

Serway CP 21 10 So,
33:04, calculus, numeric, > 1 min.

093 (part 1 of 1) 10 points 1 2I I
What maximum current is delivered by an C= = =√
2 π f XC 2 π f Vmax 2 π f Vmax
AC generator with maximum voltage 48 V
0.75 A
and frequency 90 Hz when connected across a =√
3.7 µF capacitor? 2 π (60 Hz)(170 V)
Correct answer: 0.10043 A. = 1.65499 × 10−5 F = 16.5499 µF .
Explanation:

Given : ∆Vmax = 48 V ,
Serway CP 21 12
f = 90 Hz , and 33:04, calculus, numeric, > 1 min.
C = 3.7 µF = 3.7 × 10−6 F . 095 (part 1 of 1) 10 points
The maximum current is The generator in a purely capacitive AC cir-
∆Vmax cuit has an angular frequency of 120 π rad/s.
Imax = If the maximum voltage 140 V and the
XC
capacitance is6 µF, what is the rms current in
= ∆Vmax 2 π f C the circuit?
= (48 V)(2 π)(90 Hz)(3.7 × 10−6 F)
= 0.10043 A Correct answer: 0.447843 A.
Version One – Homework 39 – Ashley Smith 23

Explanation:

Given : ω = 120 π rad/s , KUT L


C
∆Vmax = 140 V , and V0sinωt R? speaker
−6
C = 6 × 10 F.
KMFA
The inductive reactance is
At what value of the variable capacitance
1 C will the sound volume (i.e., the power dissi-
XC = ,
ωC pated in the speaker) be maximum for KUT?
Correct answer: 3.09274 pC.
the rms current is Explanation:
∆Vmax At resonance
∆Imax = , 1
XC ωKU T = r .
1
and the rms current is L CKU T
√ √ Thus
2 2 ∆Vmax
∆Irms = ∆Imax = 1 1
2 2 XC CKU T = 2
L ωKU T
so, 1
√ =
(1 × 10−6 H)
2
∆Irms = ∆Vmax ω C 1
√2 ×
(5.68628 × 108 Hz)2
2
= (140 V)(120 π rad/s)(6 × 10−6 F) = 3.09274 pC .
2
= 0.447843 A .
097 (part 2 of 6) 10 points
At what value of the variable capacitance
AC Circuit Design C will the sound volume be maximum for
33:05, calculus, numeric, > 1 min. KMFA?
096 (part 1 of 6) 10 points Correct answer: 3.16224 pC.
You are designing a radio receiver based on Explanation:
the idea of a series LRC circuit as shown At resonance
below. The two major radio stations in 1
town are KUT and KMFA, which broad- ωKM F A = r .
1
cast at frequencies νKU T = 90.5 MHz and
νKM F A = 89.5 MHz. L CKM F A
Electromagnetic signals from these ra- Thus
dio stations simultaneously supply sinusoidal 1 1
emfs CKM F A = 2
L ωKM F A
EKU T = V0 sin(ωKU T t) 1
=
and (1 × 10−6 H)
EKM F A = V0 sin(ωKM F A t) 1
×
(5.62345 × 108 Hz)2
to the circuit via an antenna. The receiving = 3.16224 pC .
circuit includes an inductor L = 1 µH, a vari-
able capacitor C, and a speaker the resistance
R which you must choose. 098 (part 3 of 6) 10 points
Version One – Homework 39 – Ashley Smith 24

In order for the radio receiver to be useful Solving for R


it must discriminate between the two radio r · ¸
1 1
stations. Your company’s specifications state R= ωKM F A L −
that when the radio is tuned to KUT, the 99 ωKM F A CKU T
2
r
ωKU L
· ¸
sound volume of KMFA signals should be 100 1
times weaker, and vice versa. = ωKM F A L − T

99 ωKM F A
What resistance R should the speaker have r
1 L £ 2 2
in order for your design to just meet company
¤
= ωKM F A − ωKU
specifications? 99 ωKM F A T

r
Correct answer: 1.27002 Ω. 1 2πL £ 2 2
¤
Explanation: = νKM F A − νKU
99 ν T

In general the average power dissipated in "r KM F A #


1 2πL
the speaker/resistor is = [νKM F A − νKU T ]
99 νKM F A
1 2 1 V02 × [ν + νKU T ]
P = I0 R = . "r KM F A
2 2 Z2
#
−6
1 2 π (1 × 10 H)
=
For KUT signal when radio is tuned for KUT 99 (8.95 × 107 Hz)
× (8.95 × 107 Hz) − (9.05 × 107 Hz)
£ ¤
ZKU T = R ,
× (8.95 × 107 Hz) + (9.05 × 107 Hz)
£ ¤

and = [7.05569 × 10−15 Ω s2 ]


1 V02 × [1 × 106 Hz] [1.8 × 108 Hz]
PKU T = .
2 R = 1.27002 Ω .
For KMFA signal when radio is tuned for
KUT 099 (part 4 of 6) 10 points
For this speaker resistance, with the radio re-
ZKM F A ceiver tuned resonantly to one of the stations,
what amplitude V0 of emf must be supplied
s
· ¸2
1
= R2 + ωKM F A L − , to the circuit in order for the speaker to dis-
ωKM F A CKU T sipate 1 W of average power of that station’s
sound?
and Correct answer: 1.59375 V.
1 V02 R Explanation:
PKM F A = 2
.
2 ZKM FA Given: At resonance, PKU T = 1 W
To meet company’s specifications 1 V02
PKU T =
2 R
PKU T = 100 PKM F A = 1 W.
Solving for V0 , we have
or √
2 V0 = 2 R P
ZKM FA
= 100 R2 p
= 2 (1.27002 Ω) (1 W)
at KMFA resonance frequency. Substituting = 1.59375 V .
the above expression for ZKM F A , this criterion
Off resonance for KMFA and on resonance for
becomes
KUT, we have
¶2
1 V02 R
µ
2 1 2
R + ωKM F A L − = 100 R . PKM F A = 2
ωKM F A CKU T 2 ZKM FA
Version One – Homework 39 – Ashley Smith 25

1 (1.59375 V)2 (1.27002 Ω) radio is tuned to KUT, we have


=
2 (12.7002 Ω)2
= 0.0100001 W . ZKM F A = 12.6525 Ω

Finally (to check Part 3), we have (Note: This is nearly the same as in Part 3)
PKU T (1 W) and
=
PKM F A (0.0100001 W)
1 V02 R
= 99.9994 . PKM F A = 2
2 ZKM FA

1 (1.59375 V)2 (0.635012 Ω)


100 (part 5 of 6) 10 points =
2 (12.6525 Ω)2
If you had put in a speaker with a smaller = 0.00503781 W ,
resistance than the value calculated above,
then
therefore
1. the company’s specifications would be ex- PKU T (2 W)
ceeded, and sound volume at resonance would =
PKM F A (0.00503781 W)
be greater than with the larger resistor. cor-
rect = 396.998 .

2. the company’s specifications would be ex- Therefore the company’s specification is ex-
ceeded, and sound volume at resonance would ceeded.
be the same as with the larger resistor.
101 (part 6 of 6) 10 points
3. the company’s specifications would not be Which graph correctly represents total
met, and sound volume at resonance would be sound volume as the capacitor is tuned?
smaller than with the larger resistor. 1. sound
volume

4. the company’s specifications would not be


met, and sound volume at resonance would be
the same as with the larger resistor.
Explanation:
CKUT CKMFA C
If R = 0.635012 Ω, smaller by a factor of
2, the signal on resonance for KUT would be correct
larger when the radio is tuned to KUT, since 2. sound
volume
the power delivered is inversely proportional
to the resistance in the circuit. For example,
if R = 0.635012 Ω
1 V02
PKU T =
2 R CKUT CKMFA C
1 (1.59375 V)2 sound
= 3.
2 (0.635012 Ω) volume
= 2 W.
Therefore the sound is louder.
Off resonance the power delivered is nearly
proportional to R, since the reactance domi-
CKUT CKMFA C
nates the impedance. Solving for PKM F A when
Version One – Homework 39 – Ashley Smith 26

4. sound = 0.00377044 A
volume
cos φ = R/Z = 30 Ω/26522.1 Ω
= 0.00113113
P = V I cos φ
= (100 V)(0.00377044 A)(0.00113113)
CKUT CKMFA C = 0.000426488 W

Explanation:
There will be two resonance frequencies of 103 (part 2 of 3) 10 points
equal magnitude, since the amplitude of each What is the power factor of the circuit?
signal is the same. Therefore the sound vol- Correct answer: 0.00113113 .
ume as the capacitance is varied is shown Explanation:
below. See part 1.
sound
volume
104 (part 3 of 3) 10 points
What is the power consumed in the circuit?
Correct answer: 0.000426488 W.
Explanation:
See part 1.
CKUT CKMFA C

Effect of a Capacitor
Effective Current in RLC 33:05, trigonometry, multiple choice, > 1 min.
33:05, trigonometry, numeric, > 1 min. 105 (part 1 of 1) 10 points
102 (part 1 of 3) 10 points A circuit containing resistance only is con-
An effective ac voltage of 100 V at 60 Hz is nected to a 110-V,60-Hz line. If a capacitor
applied to an RLC circuit with 10 mH, 0.1 µF, is connected in series with the resistance, how
and 30 Ω. What is the effective current? will the current change?
Correct answer: 0.00377044 A.
Explanation: 1. The current lags the emf.
For RLC series circuit,
q 2. The current is in phase with the emf.
Z = R2 + (XL − XC )2
3. The current is increased.
XL = 2πf L
1 4. The current is decreased. correct
XC =
2πf C
5. The current remains the same.
At f= 60 Hz ,
Explanation:
XL = 2π(60 Hz)(0.01 H) = 3.76991 Ω In an AC circuit containing only capaci-
1 tance and resistance, the current leads the
XC = = 26525.8 Ω
2π(60 Hz)(1 × 10−7 F) voltage by 90◦ . For a capacitor in an AC
q circuit, its resistance
Z = 30 Ω2 + (3.76991 Ω − 26525.8 Ω)2
= 26522.1 Ω 1
Xc =
2πf C
So
It is not equal to 0. So if you connect a
I = V /Z = 100 V/26522.1 Ω capacitor with the resistance in series, the
Version One – Homework 39 – Ashley Smith 27

resistance will be increased, and the current and


will be decreased. 1
XC = ,
ωC
Finding Capacitance in RLC into the above relation, we obtain
33:05, calculus, numeric, > 1 min.
106 (part 1 of 1) 10 points 1
= 2 π f L + R tan φ , (2)
Consider a series RLC circuit. The applied 2πf C
voltage has a maximum value of 120 V and
oscillates at a frequency of 60 Hz. The circuit we obtain
contains a capacitor whose capacitance can be 1
·
1
¸
varied, a 600 Ω resistor, and a 5.6 H inductor. C=
2 π f 2 π f L + R tan φ
Determine the value of the capacitor such · ¸−1
that the voltage across the capacitor is out of 1
= XL + R tan φ
phase with the applied voltage by 63 ◦ , with ω
VL leading Vmax . 1
·
Correct answer: 1.09753 µF. = (2111.15 Ω)
(376.991 Hz)
Explanation: ¸−1
The phase relationships for the voltage ◦
+ (600 Ω) tan(27 )
drops across the elements in the circuit are
shown in the figure. = 1.09753 µF .

VL VR
Finding Inductance in RLC
φ 33:05, calculus, numeric, > 1 min.
107 (part 1 of 1) 10 points
Vm

α
Consider a series RLC circuit. The applied
ax

VC voltage has a maximum value of 120 V and


oscillates at a frequency of 60 Hz. The circuit
contains an inductor whose inductance can
From the figure, we see that the phase angle be varied, a 600 Ω resistor, and a 1.3 µF
is capacitor.
φ = α − 90◦ Determine the value of the inductance such
= (90◦ ) − (63◦ ) that the voltage across the capacitor is out of
= 27◦ = 0.471239 rad . phase with the applied voltage by 62 ◦ , with
Vmax leading VC .
This is because the phasors representing Imax Correct answer: 4.56621 H.
and VR are in the same direction (they are in Explanation:
phase). From the equation The phase relationships for the voltage
XL − X C drops across the elements in the circuit are
tan φ = (1)
R shown in the figure.
and
VC
ω = 2πf
= 2 π (60 Hz) Vmax
= 376.991 Hz .
α
Substituting the expressions for XL and XC ,
φ
XL = ω L VR
= (376.991 Hz) (5.6 H)
= 2111.15 Ω . VL
Version One – Homework 39 – Ashley Smith 28

From the figure, we see that the phase angle 33:05, trigonometry, numeric, > 1 min.
is 108 (part 1 of 1) 10 points
A coil’s impedance is 40 Ω and its inductive
φ = α − 90◦ reactance is 20 Ω. What is its resistance?
= −(−62◦ ) − (90◦ ) Correct answer: 34.641 Ω.
= −28◦ = −0.488692 rad . Explanation:
The resistance, impedance, and reactance
This is because the phasors representing Imax are related through the basic equation
and VR are in the same direction (they are in
phase). From the equations Z = [R2 + (XL − XC )2 ]1/2 .

XL − X C Thus, since XC = 0 in our problem,


tan φ = (1)
R
R = [Z 2 − XL2 ]1/2
and
= [(40 Ω)2 − (20 Ω)2 ]1/2
ω = 2πf = 34.641 Ω.
= 2 π (60 Hz)
= 376.991 Hz . Impedance of RLC
Substituting the expressions for XL and XC , 33:05, calculus, numeric, > 1 min.
109 (part 1 of 2) 10 points
XL = ω L In a certain series RLC circuit, the rms cur-
rent is 9 A, the rms voltage is 180 V and the
and current leads the voltage by 37◦ .
What is the total resistance of the circuit?
1
XC = Correct answer: 15.9727 Ω.
ωC Explanation:
1
= To solve this problem, we use the result that
(376.991 Hz) (1.3 µF) the average power delivered by the generator
= 2040.45 Ω . is dissipated as heat in the resistor. The
average power delivered by the generator is
into the above relation, we obtain
given by
1
2πf L = + R tan φ , (2) Pav = Irms Vrms cos(φ)
2πf C
= (9 A) (180 V) cos(−37◦ )
we obtain = 1293.79 W .
· ¸
1 1
L= + R tan φ Note that, in the above expression, the phase
2πf 2πf C angle is −37◦ , since the voltage lags the cur-
· ¸
1 rent. The power dissipated in the resistor is
= XC + R tan φ
ω given by
2
Pav = Irms R.
·
1
= (2040.45 Ω)
(376.991 Hz) So, we obtain for the resistance
¸

+ (600 Ω) tan(−28 ) Pav
R= 2
Irms
= 4.56621 H .
1293.79 W
=
(9 A)2
Impedance and Reactance = 15.9727 Ω .
Version One – Homework 39 – Ashley Smith 29

110 (part 2 of 2) 10 points 112 (part 2 of 4) 10 points


Calculate the reactance of the circuit (XL − What is the phase angle between the current
XC ). and the applied voltage?
Correct answer: −12.0363 Ω. Correct answer: 57.5184 ◦ .
Explanation: Explanation:
To get the reactance of the circuit, we use The phase angle between the current and
applied voltage is given by
XL − X C
tan φ = µ ¶
R −1 R
X − XC φ = cos
tan(−37◦ ) =
L Z
15.9727 Ω µ
18 Ω

−1
XL − XC = −12.0363 Ω . = cos
33.5177 Ω

= 57.5184 .
Inductive AC Circuit
33:05, trigonometry, numeric, > 1 min. Alternate Solution:
111 (part 1 of 4) 10 points µ ¶
−1 XL − XC
A 90 mH inductor with a resistance of 18 Ω φ = tan
R
is connected to an AC power source of µ ¶
110 V(rms) and 50 Hz. −1 28.2743 Ω
= tan
Find the rms current in the circuit. 18 Ω

Correct answer: 3.28185 A. = 57.5184 .
Explanation:
The rms current in an AC circuit is given
by 113 (part 3 of 4) 10 points
Vrms What is the power factor for the circuit?
Irms = , Correct answer: 0.537029 .
Z
where Z is the impedance, Explanation:
q The power factor of the circuit is
Z = R2 + (XL − XC )2 , cos φ = cos(57.5184◦ )
Vmax = 0.537029 .
and Vrms = √ is the effective voltage.
2
Here
114 (part 4 of 4) 10 points
What is the power loss in the inductor’s resis-
XL = 2 π f L tance?
= 2 π (50 Hz) (90 mH) = 28.2743 Ω . Correct answer: 193.869 W.
Explanation:
Then The power is entirely lost to the resistance
q RL in the inductor. It is given by
Z= R2 + XL2 2
q Pav = Vrms Irms cos φ = Irms R
= (18 Ω)2 + (28.2743 Ω)2 = (110 V)(3.28185 A)(0.537029)
= 33.5177 Ω . = (3.28185 A)2 (18 Ω)
= 193.869 W .
Therefore
110 V
Irms = = 3.28185 A . LC Circuit
33.5177 Ω
Version One – Homework 39 – Ashley Smith 30

33:05, trigonometry, numeric, > 1 min. The light bulb glows most brightly at
115 (part 1 of 2) 10 points
A circuit consists of an idealized 50 mH in- 1
1. low frequencies, 0 < ω < √ .
ductor and an idealized 10 µF capacitor. LC
What is the natural frequency of this circuit 2. steady DC voltage, ω = 0.
if the capacitor is first charged to 100 µc
and then allowed to discharge through the 1
3. the resonant frequency, ω = √ .
inductor? LC
Correct answer: 225.079 Hz. 1
Explanation: 4. high frequencies, √ < ω < ∞. cor-
LC
The natural frequency is given by rect
r
1 1 Explanation:
f= The voltage across L is
2π LC
= 225.079 Hz .
E = I L ZL ,

116 (part 2 of 2) 10 points where the impedance is


Find the current in the circuit when the ca- q
pacitor has lost 20 % of its initial energy. ZL = XL2 .
Correct answer: 0.0632456 A.
Explanation: Since XL = ω L , the current through the
Q is the initial charge, q the final charge, inductor is
and I is the final current. The total energy is
conserved over time, so E
IL = q .
Q2 q2 1 (ω L)2
= + LI 2 .
2C 2C 2
As the frequency ω increases, the current IL
According to the problem,
will decrease.
Q2 q2 Q2 The voltage across R and C is
− = (20 %)( ).
2C 2C 2C
E = IRC ZRC ,
r
20 % 1
I=Q where the impedance is
100 LC
= 0.0632456 A q
ZRC = R2 + XC2 .

Light Bulb in LC Circuit 02 1


33:05, trigonometry, multiple choice, < 1 min. Since XC = , the current through the
ωC
117 (part 1 of 1) 10 points light bulb is
The emf E drives the circuit shown below at
frequency ω. E
IRC = s µ ¶2 .
1
R2 +
R ωC

E L As the frequency ω increases, the current IRC


C will also increase.
Therefore the bulb will glow most brightly
at high frequencies.
Version One – Homework 39 – Ashley Smith 31

Since XL = ω L , the current through the


Light Bulb in LC Circuit 03 inductor is
33:05, trigonometry, multiple choice, < 1 min.
118 (part 1 of 1) 10 points E
IRL = q .
The emf E drives the circuit shown below 2
at an angular velocity ω. (R2 + ω L)

As the angular velocity ω decreases, the cur-


R rent IRL will increase. The limiting case is
E C L constant voltage.
Therefore the light bulb will glow most
brightly at a constant voltage, i.e., DC volt-
The light bulb (with resistance R) glows age.
most brightly at
Light Bulb in LC Circuit 04
1 33:05, trigonometry, multiple choice, < 1 min.
1. low angular velocity, 0 < ω < √ .
LC 119 (part 1 of 1) 10 points
The emf E drives the circuit shown below at
2. steady DC voltage, ω = 0. correct
frequency ω.
1
3. the resonant angular velocity, ω = √ . R
LC
1
4. high angular velocity, √ < ω < ∞. E
LC L
C
Explanation:
The voltage across C is

E = I C ZC , The light bulb glows the dimmest at

where the impedance is 1


1. low frequencies, 0 < ω < √ .
q LC
ZC = XC2 . 2. steady DC voltage, ω = 0.

1 1
Since XC = , the current through the 3. the resonant frequency, ω = √ . cor-
ωC LC
light bulb is rect
E 1
IC = s µ 4. high frequencies, √ < ω < ∞.
¶2 . LC
1
ωC Explanation:
The voltage across C and L is
As the angular velocity ω decreases, the cur-
rent IC will also decrease.
E = I R + I ZLC ,
The voltage across R and L is

E = IRL ZRL , where the impedance for a parallel circuit is

where the impedance is 1


Z12 = ,
q 1 1
ZRL = R2 + XL2 . +
Z1 Z2
Version One – Homework 39 – Ashley Smith 32

or for the inductor and capacitor parallel where the impedance is


1 q
ZLC = ZRLC = R2 + (XL − XC )2 .
1
+ XC
XL 1
1 Since XL = ω L and XC = , the current
= , ωC
1 through the light bulb is
+ωC
ωL E
1 I=s ¶2 .
where Z1 = XL = ω L and Z2 = XC = .
µ
1
ωC R2 + ω L −
By inspection, ZLC will approach zero ωC
(ZLC = XL − XC → 0 Ω) when either the
frequency is high or When the angular velocity ω is on resonance
µ low. ZLC will¶ be maxi-
1 for the LC part of the circuit, the reactance
mum at resonance ZLC = √ . will be zero, since (ZLC = XL − XC = 0 Ω).
LC
Consequently the light bulb will glow the Consequently, the current will be maximum
1
dimmest at resonance frequency, when the at ω = .
impedance in both the capacitance and in- LC
ductance legs of the LC part of the circuit are Max Current in RLC
equal. That is, the current I is through the 33:05, calculus, numeric, > 1 min.
light bulb is the smallest. 121 (part 1 of 2) 10 points
In a series RLC ac circuit, the resistance is
Light Bulb in LC Circuit 05 21 Ω , the inductance is 25 mH, and the
33:05, trigonometry, multiple choice, < 1 min. capacitance is 17 µF. The maximum poten-
120 (part 1 of 1) 10 points tial is 150 V, and the angular frequency is
The emf E drives the circuit shown below 636.62 rad/s.
at angular velocity ω. Calculate the maximum current in the cir-
R cuit.
L Correct answer: 1.8912 A.
Explanation:
E C The capacitive reactance is
1
XC =
ωC
The light bulb (with resistance R) glows 1
most brightly at =
(636.62 rad/s) (1.7 × 10−5 F)
1 = 92.3998 Ω .
1. low frequencies, 0 < ω < √ .
LC The inductive reactance is
2. steady DC voltage, ω = 0. XL = ω L
1 = (636.62 rad/s) (0.025 H)
3. the resonant angular velocity, ω = √ .
LC = 15.9155 Ω .
correct
The maximum current is
1
4. high frequencies, √ < ω < ∞. Vmax
LC Imax =
Z
Explanation: Vmax
The voltage across R, C, and L is =p
R2 + (XL − XC )2
E = I ZRLC , = 1.8912 A .
Version One – Homework 39 – Ashley Smith 33

122 (part 2 of 2) 10 points 124 (part 2 of 5) 10 points


What is the power factor for the circuit? Determine the maximum voltage across the
Correct answer: 0.264768 . resistor.
Explanation: Correct answer: 39.7152 V.
The phase angle between the voltage and the Explanation:
current in the circuit is The maximum voltage across the resistor is
µ ¶
−1 XL − XC
φ = tan . VR = Imax R
R
= (1.8912 A) (21 Ω)
The power factor is = 39.7152 V .
· µ ¶¸
−1 XL − XC
cos φ = cos tan
R
= 0.264768 . 125 (part 3 of 5) 10 points
Determine the maximum voltage across the
inductor.
Max Current in RLC 05 Correct answer: 30.0994 V.
33:05, calculus, numeric, > 1 min. Explanation:
123 (part 1 of 5) 10 points The maximum voltage across the inductor is
In a series RLC ac circuit, the resistance is
21 Ω , the inductance is 25 mH, and the VL = Imax XL
capacitance is 17 µF. The maximum poten-
= (1.8912 A) (15.9155 Ω)
tial is 150 V, and the angular frequency is
636.62 rad/s. = 30.0994 V .
Calculate the maximum current in the cir-
cuit.
Correct answer: 1.8912 A. 126 (part 4 of 5) 10 points
Explanation: Determine the maximum voltage across the
The capacitive reactance is capacitor.
1 Correct answer: 174.746 V.
XC = Explanation:
ωC
1 The maximum voltage across the capacitor is
=
(636.62 rad/s) (1.7 × 10−5 F)
= 92.3998 Ω . VC = Imax XC
= (1.8912 A) (92.3998 Ω)
The inductive reactance is = 174.746 V .
XL = ω L
= (636.62 rad/s) (0.025 H)
127 (part 5 of 5) 10 points
= 15.9155 Ω .
What is the power factor for the circuit?
The maximum current is Correct answer: 0.264768 .
Explanation:
Vmax The phase angle between the voltage and the
Imax =
Z current in the circuit is
Vmax
=p
R2 + (XL − XC )2
µ ¶
−1 XL − X C
φ = tan .
= 1.8912 A . R
Version One – Homework 39 – Ashley Smith 34

The power factor is


· µ ¶¸ E
−1 XL − XC
cos φ = cos tan φ
R
= 0.264768 . 4.

Oscillating EMF
33:05, calculus, multiple choice, < 1 min.
128 (part 1 of 1) 10 points E
Assume: Counter-clockwise rotation.
The graph below depicts an oscillating emf.
5.
φ
Emax φ

Explanation:
The oscillating emf is proportional to
0
sin(ωt − φ). So the right answer is
E

Emin

0 T /2 t −→ T
Which phasor diagram correctly represents φ
this oscillation?

E Phase Angle in AC Circuit


33:05, calculus, numeric, > 1 min.
129 (part 1 of 1) 10 points
1. An RLC circuit consists of a 150 Ω resistor,
a 21 µF capacitor, and a 460 mH inductor,
φ connected in series with a 120 V, 60 Hz power
supply.
correct What is the phase angle between the cur-
rent and the applied voltage? Answer be-
E tween −180◦ and +180◦ .
Correct answer: 17.4332 ◦ .
φ Explanation:
2. The inductive reactance is:
XL = ω L = 2 π f L
the capacitive reactance is:
E 1 1
XC = =
φ ωC 2πf C
the phase angle between the current and the
3. voltage is:
µ ¶
XL − X C
φ = arctan = 17.4332◦
R
Version One – Homework 39 – Ashley Smith 35
s
1 1
Resonance Conditions in RLC =
2 π (20 mH) (100 nF)
33:05, trigonometry, numeric, > 1 min.
130 (part 1 of 2) 10 points = 3.55881 kHz .
A 0.25 H inductor is connected to a capacitor
and a 30 Ω resistor along with a 60 Hz, 30 V
generator. 133 (part 2 of 3) 10 points
To what value would the capacitor have to Find the amplitude of the current at the res-
be adjusted to produce resonance? onant frequency.
Correct answer: 28.1448 µF. Correct answer: 5 A.
Explanation: Explanation:
From At resonance,
1
f0 = √ , Vmax
2π LC Imax =
R
we find (100 V)
=
1 (20 Ω)
C= = 5 A.
4 π 2 L f0 2
1
=
4 π 2 (0.25 H) (60 Hz)2
134 (part 3 of 3) 10 points
= 2.81448 × 10−5 F Find the amplitude of the voltage across the
= 28.1448 µF . inductor at resonance.
Correct answer: 2236.07 V.
Explanation:
131 (part 2 of 2) 10 points The amplitude of the voltage across the
Find the voltage drop across the resistor at inductor at resonance is
resonance.
Correct answer: 30 V. VL = XL Imax
Explanation: = ω0 L Imax
At resonance, XL = XC . Therefore, Z = (22360.7 rad/s)
across the capacitor-inductor combination is
zero, and the voltage drop across the resistor × (20 mH) (5 A)
is 30 V. = 2236.07 V .

Resonance of an AC Circuit 02
33:05, calculus, numeric, > 1 min. RLC Circuit Impedance
132 (part 1 of 3) 10 points 33:05, trigonometry, numeric, > 1 min.
An AC power supply with a maximum poten- 135 (part 1 of 2) 10 points
tial of 100 V is in series with a RLC circuit, Find the impedance of an RLC series cir-
where the inductance is 20 mH, the capaci- cuit with inductance 30 mH, capacitance
tance is 100 nF, the resistance is 20 Ω. 12 µF, and resistance 100 Ω at a frequency
Find the resonant frequency in kHz. of 60 Hz.
Correct answer: 3.55881 kHz. Correct answer: 232.358 Ω.
Explanation: Explanation:
The resonance frequency for a series RLC For RLC series circuit, at f=60 Hz, where
circuit is ω = 2 π f = 376.991 s−1 ,

XL = 2 π f L
r
1 1
f= = (376.991 s−1 ) (0.03 H)
2π LC
Version One – Homework 39 – Ashley Smith 36

= 11.3097 Ω lost in R is compensated by energy stored in


1 C and L.
XC =
2πf C
1 3. The current through C is 90◦ out of phase
= with the one through L.
(376.991 s ) (1.2 × 10−5 F)
−1

= 221.049 Ω
q 4. The current through C is 180◦ out of
Z= R2 + (XL − XC )2 phase with the one through L.
q
= (100 Ω)2 + (−209.739 Ω)2
5. All energy is dissipated in R. correct
= 232.358 Ω ,
Explanation:
where XL −XC = (11.3097 Ω)−(221.049 Ω) = Let’s look at the statements one by one.
−209.739 Ω. a) L stores kinetic energy. So, we cannot
say that potential energy oscillates between L
136 (part 2 of 2) 10 points and C anyway. This statement is therefore
Find the impedance at a frequency of 600 Hz. false. b) The energy generated by the source
Correct answer: 135.202 Ω. is dissipated in the resistor. So the source has
Explanation: to do some net work in order to compensate
Similarly, at f = 600 Hz, where ω = 2 π f = for this loss. The statement is false. c) and
3769.91 s−1 . d) Since everything is connected in series, the
current is same for all the elements (R, L and
XL = 2 π f L C). These two are both false. e) Energy can-
= (3769.91 s−1 ) (0.03 H) not be dissipated either in L or C. So it is all
= 113.097 Ω dissipated in R.
1
XC =
2πf C
1 RLC TF Questions 01
=
(3769.91 s ) (1.2 × 10−5 F)
−1 33:05, calculus, multiple choice, > 1 min.
= 22.1049 Ω 138 (part 1 of 3) 10 points
This RLC circuit is driven by an oscillating
q
Z= R2 + (XL − XC )2 emf, E.
q
= (100 Ω)2 + (90.9925 Ω)2
R L
= 135.202 Ω .

where XL −XC = (113.097 Ω)−(22.1049 Ω) = E C


90.9925 Ω.

RLC Circuit Relations


33:05, calculus, multiple choice, < 1 min. Which of the following statements is true?
137 (part 1 of 1) 10 points a) All energy is dissipated in R .
Consider a series RLC circuit driven by an
AC source. Which of the following statements 1. True correct
is/are true?
2. False
1. Potential energy oscillates between C and
L. 3. Cannot be determined

2. The source does no net work, since energy Explanation:


Version One – Homework 39 – Ashley Smith 37

a) L stores magnetic energy and C stores This RLC circuit is driven by an oscillating
electric energy. Without resistance these en- emf, E.
ergies oscillate between L and C . This energy
is not dissipated since it is conserved. How-
ever, the resistor directly dissipates energy
and, as it is in the circuit, energy is always

C
E
dissipated
PR = I 2 R > 0 .
Statement (a) is true.
Which of the following statements is true?
139 (part 2 of 3) 10 points a) All energy is dissipated in R .
b) The currents across C and L are 180◦ out
of phase. 1. True correct

1. False correct 2. False

2. True 3. Cannot be determined

3. Cannot be determined Explanation:


a) L stores magnetic energy and C stores
Explanation: electric energy. Without resistance these en-
b) since all elements are in series, the cur- ergies oscillate between L and C . This energy
rent is the same in each element (they are in is not dissipated since it is conserved. How-
phase in L and C as well as R) although the ever, the resistor directly dissipates energy
voltage across L and C are 180◦ out of phase. and, as it is in the circuit, energy is always
Statement (b) is false. dissipated
PR = I 2 R > 0 .
140 (part 3 of 3) 10 points
c) The source E does no net work, since energy Statement (a) is true.
lost in R is compensated by energy stored in
C and L . 142 (part 2 of 3) 10 points
b) The electric potential across C and L are
1. False correct 180◦ out of phase.

2. True 1. False correct

3. Cannot be determined 2. True


Explanation:
3. Cannot be determined
c) Magnetic energy in L is converted to elec-
tric energy in C, but the energy dissipated in Explanation:
the resistor R is extracted from the electrical b) since all elements are in parallel, the po-
system supplied by the source E, which must tential difference is the same in across element
do work. (they are in phase in L and C as well as R)
So all the energy is dissipated in R and although the current across L and C are 180◦
statement (c) is false. out of phase.
Statement (b) is false.
RLC TF Questions 02
33:05, highSchool, multiple choice, > 1 min. 143 (part 3 of 3) 10 points
141 (part 1 of 3) 10 points c) The source E does no net work, since energy
Version One – Homework 39 – Ashley Smith 38

lost in R is compensated by energy stored in and finally


C and L .
vrms
Irms =
1. False correct Z
106.066 V
=
2. True 735.109 Ω
= 0.144286 A .
3. Cannot be determined
Explanation:
c) Magnetic energy in L is converted to elec- 145 (part 2 of 4) 10 points
tric energy in C, but the energy dissipated in Find the rms voltage across R.
the resistor R is extracted from the electrical Correct answer: 7.35859 V.
system supplied by the source E, which must Explanation:
do work.
So all the energy is dissipated in R and
statement (c) is false. VR = Irms R
= (0.144286 A) (51 Ω)
RMS Current and Potential
33:05, trigonometry, numeric, > 1 min. = 7.35859 V
144 (part 1 of 4) 10 points
A 51 Ω resistor is connected in series with
a 52 mH inductor and a 0.2 µF capac- 146 (part 3 of 4) 10 points
itor. The applied voltage has the form Find the rms voltage across L.
E = 150 V sin(2 π f t), where the frequency Correct answer: 37.7136 V.
is f = 800 s−1 . Explanation:
Find the rms current.
Correct answer: 0.144286 A.
Explanation: VL = Irms XL
The rms current I is computed using the
v = (0.144286 A) (261.381 Ω)
rms voltage, vrms = √ = 106.066 V. Regard
2 = 37.7136 V
the net impedance Z as a resistance and apply
Ohm’s law in that form:
Vrms = Z I . 147 (part 4 of 4) 10 points
The impedance is Find the rms voltage across C.
¤1/2 Correct answer: 143.524 V.
Z = R2 + (XL − XC )2
£
, Explanation:
where XL and XC are the reactances of the
inductor and capacitor, given (symmetrically)
by VC = Irms XC
XL = 2 π f L = 261.381 Ω , = (0.144286 A) (994.718 Ω)
and = 143.524 V
1
XC = = 994.718 Ω .
(2 π f C)
From this, RMS Potential in RLC
¤1/2 33:05, trigonometry, numeric, > 1 min.
Z = 51 Ω2 + (261.381 Ω − 994.718 Ω)2
£
148 (part 1 of 1) 10 points
= 735.109 Ω , A(n) 10 Ω resistor is connected in series with
Version One – Homework 39 – Ashley Smith 39

a(n) 100 mH inductor and a(n) 0.1 µF ca-


pacitor. The applied voltage has the form 150 (part 2 of 2) 10 points
V = V0 sin(2πf t), with initial voltage 100 V What is the capacitance?
and frequency 1000 Hz. Correct answer: 26.7528 µF.
Find the effective voltage V across the ca- Explanation:
pacitor.
Correct answer: 116.829 V. Series RC Circuit 11
Explanation: 33:05, trigonometry, numeric, > 1 min.
The impedance of the circuit is 151 (part 1 of 3) 10 points
q A(n) 10 µF capacitor is put in series with
Z = R2 + (XL − XC )2 a(n) 40 Ω resistor and connected to a(n)
100 V(effective), 60 Hz power line. Find the
where effective current in the circuit.
XL = 2πf L Correct answer: 0.372777 A.
1 Explanation:
XC = .
2πf C
The effective voltage of the source is
V0 1
V =√ , XC =
2 2πf C
1
=
XC 2π(60 Hz)(1 × 10−5 F)
VC = V
Z = 265.258 Ω
= 116.829 V V .
q
Z= R 2 + XC 2
p
= 40 Ω2 + 265.258 Ω2
Series RC Circuit 08
33:05, trigonometry, numeric, > 1 min. = 268.257 Ω
149 (part 1 of 2) 10 points Ief f = V /Z = 100 V/268.257 Ω
A capacitor is connected in series with a 13 Ω = 0.372777 A
resistance. If the impedance of the circuit cos φ = R/Z = 40 Ω/268.257 Ω
at 60 HZ is 100 Ω, what is the capacitive
= 0.149111
reactance of the capacitor?
Correct answer: 99.1514 Ω. so φ = 81.4246◦
Explanation: and P = Ief f Vef f cos φ
As we know, = (0.372777 A)(100 V)(0.149111)
q = 5.5585 W
Z = R 2 + XC 2
p
XC = Z 2 − R 2
p
= 100 Ω2 − 13 Ω2
152 (part 2 of 3) 10 points
= 99.1514 Ω Find the phase angle between the current and
1
From XC = the applied voltage.
2πf C Correct answer: 81.4246 ◦ .
1 Explanation:
we get C =
2πf Xc
1 153 (part 3 of 3) 10 points
=
2π(60)(99.1514 Ω) Find the power loss in the inductor.
= 26.7528 µF Correct answer: 5.5585 W.
Version One – Homework 39 – Ashley Smith 40

Explanation:
Serway CP 21 21
Series RLC Circuit 16 33:05, calculus, numeric, > 1 min.
33:05, trigonometry, numeric, > 1 min. 157 (part 1 of 3) 10 points
154 (part 1 of 3) 10 points A 900 Ω resistor, a 0.25 µF capasitor, and
In a certain RLC circuit, the rms current is 2.5 H inductor are connected in series across
6 A, the rms voltage is 240 V, and the current a 240 Hz AC source for which the maximum
leads the voltage by 53◦ . voltage is 140 V.
What is the total resistance of the circuit? Calculate the impedance of the circuit.
Correct answer: 24.0726 Ω. Correct answer: 1434.72 Ω.
Explanation: Explanation:

V 240 V
Z= = = 40 Ω ,
I 6A Given : R = 900 Ω ,
and C = 0.25 µF ,
R = Z cos φ L = 2.5 × 10−7 F ,
= (40 Ω) cos(−53◦ ) f = 240 Hz , and
= 24.0726 Ω . ∆Vmax = 140 V .

We first calculate the inductive reactance


155 (part 2 of 3) 10 points
Calculate the total reactance, XL − XC . ω = 2πf
Correct answer: −31.9454 Ω.
Explanation: = 1507.96 rad/s .
From
XL − X C
tan φ = ,
R XL = ω L
we find = (1507.96 rad/s) (2.5 H)
XL − XC = R tan φ = 3769.91 Ω .
= (24.0726 Ω) tan(−53◦ )
The capacitive reactance is given by
= −31.9454 Ω .
1
XC =
156 (part 3 of 3) 10 points ωC
1
Find the average power dissipated in the cir- =
cuit. (1507.96 rad/s) (2.5 × 10−7 F)
Correct answer: 866.613 W. = 2652.58 Ω .
Explanation:
For the impedance, we have

P = I V cos φ q
Z = R2 + (XL − XC )2
= (6 A) (240 V) cos(−53◦ ) q
= 866.613 W Z = (900 Ω)2 + (3769.91 Ω − 2652.58 Ω)2
or = 1434.72 Ω .
P = I2 R
= (6 A)2 (24.0726 Ω) 158 (part 2 of 3) 10 points
= 866.613 W . Calculate the maximum current delivered by
Version One – Homework 39 – Ashley Smith 41

the source. for example, when it is first charged. There-


Correct answer: 0.09758 A. fore
Explanation:
1
The maximum current in the circuit is ob- E = Q2 /C
tained from 2
= 0.000745033 J.
Vmax
Imax =
Z Since there is no resistance in the circuit, the
140 V combined energy in the capacitor and the in-
= ductor is conserved. Since the circuit acts like
1434.72 Ω
= 0.09758 A . a simple oscillator, the energy in the inductor
will, once during each period of oscillation,
equals the total energy in the circuit.
159 (part 3 of 3) 10 points
Calculate the phase angle between the current E = 0.000745033 J.
and voltage.
Correct answer: 51.1488 ◦ .
Explanation: 161 (part 2 of 3) 10 points
In an RLC circuit, the phase angle between What is the maximum energy stored in the
the current and voltage is capacitor?
µ ¶ Correct answer: 0.000745033 J.
−1 XL − XC Explanation:
φ = tan
R
µ ¶
−1 3769.91 Ω − 2652.58 Ω 162 (part 3 of 3) 10 points
= tan
900 Ω What is the maximum energy stored in the

= 51.1488 . inductor?
Correct answer: 0.000745033 J.
Explanation:
Simple LC Circuit 03
33:05, trigonometry, numeric, > 1 min. Effective Voltage
160 (part 1 of 3) 10 points 33:06, trigonometry, numeric, > 1 min.
A circuit consists of an idealized 150 mH in- 163 (part 1 of 3) 10 points
ductor and a 15.1 µF capacitor. An AC generator develops a maximum volt-
What is the natural (linear) frequency of age of 150 V. It delivers a maximum current
the circuit if the capacitor is first charged to of 30 A to an external circuit.
150 µC and then allowed to discharge through What is the effective voltage of the genera-
the inductor? tor?
Correct answer: 105.751 s−1 . Correct answer: 106.066 V.
Explanation: Explanation:
The natural frequency f of an LC-circuit is Both the effective voltage and the effective
given by current in an AC generator are less that the
maximum output.
1 1
f=
2π (LC)1/2
r
1 Vmax
1 1 Vef f = Vmax = √
= 2 2
2π [(0.15 H)(1.51 × 10−5 F)]1/2
= 105.751 s−1 . 164 (part 2 of 3) 10 points
What effective current does it deliver to the
The energy E stored in the capacitor is maxi- external circuit?
mum when the charge on it is at its maximum, Correct answer: 21.2132 A.
Version One – Homework 39 – Ashley Smith 42

Explanation: The phase angle is


r
1 Imax XL − X C
Ief f = Imax = √ φ = arctan
2 2 R
226.2 Ω − 757.863 Ω
165 (part 3 of 3) 10 points = arctan
250 Ω
What is the effective power dissipated in = −1.13125 rad = −64.816◦ .
the circuit?
Correct answer: 2.25 kW. Thus, the phase factor, cos φ, is equal to
Explanation:

Pef f = Ief f Vef f cos φ = cos(−1.13125 rad)


= 0.425526 ,
Dimensional analysis for Pef f :
1 kW and hence the averaged power is
A·V=W· = kW
1000 W
Pav = Irms Vrms cos φ
Impedance in an RL Circuit
= (0.180536 A) (106.066 V) (0.425526)
33:06, calculus, numeric, > 1 min.
166 (part 1 of 1) 10 points = 8.14827 W .
Calculate the average power delivered to
the series RLC circuit with R = 250 Ω, L =
0.6 H, C = 3.5 µF, ω = 377 s−1 , if the Power in an RLC Circuit
maximum value of the applied voltage equal 33:06, calculus, multiple choice, < 1 min.
to 150 V. 167 (part 1 of 1) 10 points
Correct answer: 8.14827 W. A light bulb is connected to a wall
Explanation: socket which provides an oscillating voltage
The reactances are V0 sin(ω t) to the circuit, and is visibly lit.
After a piece of iron is shoved into a coil of
XL = ω L
wire in the circuit as shown, the light
= 226.2 Ω,

1 1. brightens
XC =
ωC 2. dims correct
= 757.863 Ω.
Therefore, the impedance is 3. is not affected
q
Z = R2 + (XL − XC )2 Explanation:
q The impedance of the circuit is
= (250 Ω)2 + (226.2 Ω − 757.863 Ω)2
p
= 587.508 Ω. Z= R2 + ωL2 ,
Now let us calculate the rms voltage and rms
current: where R is the bulb’s resistance, L is the coil’s
Vmax inductance. The rms current in the circuit is
Vrms = √ then
2 Vrms
= 106.066 V , Irms = .
Z
Vrms
Irms =
Z When we shove the piece of iron into the coil,
= 0.180536 A . its inductance increase, and consequently, the
Version One – Homework 39 – Ashley Smith 43

total impedance increases. This results in the We obtain for the power supplied by the
decrease in the rms current, and hence in the source
2
decrease of the power P = Irms R dissipated
in the bulb, causing it to dim. P = Vrms Irms cos(φ)
= (100 V) (2 A) cos(36.8699◦ )
Power in an RLS Circuit = 160 W .
33:06, calculus, numeric, > 1 min.
168 (part 1 of 2) 10 points
In a series RLC circuit, the resistance is 40 Ω, 169 (part 2 of 2) 10 points
the capacitance is 50 µF and the inductance Find the power dissipated in the resistor.
is 50 mH. The ac generator provides an rms Correct answer: 160 W.
voltage of 100 V operating at 1000 rad/s. Explanation:
Find the power supplied by the source. The power dissipated in a resistor that carries
Correct answer: 160 W. an alternating current is
Explanation: 2
PR = Irms R.
The power supplied by an ac source is given
by So, we have
P = Vrms Irms cos(φ) ,
PR = (2 A)2 (40 Ω)
where φ is the phase angle between the source
= 160 W .
voltage and source current. In a RLC circuit,
the phase angle is given by We note that the average power delivered by
µ ¶ the generator is dissipated as heat in the re-
−1 XL − XC
φ = tan . sistor.
R
For the reactances, we have Serway CP 21 29
33:06, trigonometry, numeric, > 1 min.
XL = ω L 170 (part 1 of 3) 10 points
= (1000 rad/s) (0.05 H) A multimeter in an RL circuit records an rms
= 50 Ω . current of 0.5 A and a 60 Hz rms generator
voltage of 104 V. A wattmeter shows that
And the average power delivered to the resistor is
1 10 W.
XC =
ωC Find the impedance in the circuit.
1 Correct answer: 208 Ω.
=
(1000 rad/s) (5 × 10−5 F) Explanation:
= 20 Ω .
Therefore Given : I = 0.5 A ,
µ
50 Ω − 20 Ω
¶ f = 60 Hz ,
φ = tan−1 V = 104 V , and
40 Ω

= 36.8699 . P = 10 W .

The rms current is given by Since V = I Z, we have

Vrms V
Irms = Z=
Z I
100 V (104 V)
= =
50 Ω (0.5 A)
=2A. = 208 Ω .
Version One – Homework 39 – Ashley Smith 44

When the switch of the LC circuit is first


171 (part 2 of 3) 10 points closed, the voltage across the capacitor is
Find the resistance R. V and the initial current, I(0), in the cir-
Correct answer: 40 Ω. cuit is zero. The voltage will oscillate and
Explanation: never be larger than its initial voltage, so that
The power dissipated is the change across the capacitor will never be
larger than its initial charge which is
P = I 2 R,

so that Q = CV
P = (4 × 10−5 F)(120 V)
R= = 0.0048 C .
I2
(10 W)
=
(0.5 A)2 174 (part 2 of 3) 10 points
= 40 Ω . What is the resonant frequency of the cir-
cuit?
Correct answer: 251.646 Hz.
172 (part 3 of 3) 10 points Explanation:
Find the inductance L. An LC circuit
Correct answer: 0.541439 H. p oscillates with an angular
frequency ω = 1/L C, which would be the
Explanation: q resonant frequency. Thus
In an RL circuit, Z = R2 + XL2 , or
r
p ω 1 1
XL = Z 2 − R2 f= =
q 2π 2π LC
= (208 Ω)2 − (40 Ω)2
s
1 1
= 204.118 Ω . =
2π (0.01 H)(4 × 10−5 F)
Therefore, = 251.646 Hz .

XL
L= 175 (part 3 of 3) 10 points
2πf
What is the maximum current in the circuit?
(204.118 Ω)
= Correct answer: 7.58947 A.
2 π (60 Hz) Explanation:
= 0.541439 H . Initially the voltage across the capacitor
is V0pand it oscillates as V0 cos(ω t), where
ω = 1/L C. The phase δ is zero, since the
An LC Circuit
maximum voltage occurs at t = 0. To within
33:07, trigonometry, numeric, > 1 min.
a sign, the voltage across the inductor is also
173 (part 1 of 3) 10 points
this expression, as by Kirchoff’s voltage law,
A capacitor of 40 µF is connected in series to
the voltage drops around a closed loop is zero.
an inductor of 10 mH and to an open switch.
Then
The capacitor is first charged to a voltage of
V = 120 V. The charging battery is then dI
V = −L = V0 cos ω t .
removed, and the switch is closed. dt
What is the maximum charge on the capac- Then integrating gives
itor? µ ¶1/2
Correct answer: 0.0048 C. V0 C
I(t) = − sin ω t = − V0 sin ω t .
Explanation: ωL L
Version One – Homework 39 – Ashley Smith 45

Thus the maximum current is The rms current at the operating frequency is

µ ¶1/2
C Vrms
I = V0 Irms =
L Z
50 V
·
4 × 10−5 F
¸1/2 =
= 120 V 18.0278 Ω
0.01 H = 2.7735 A .
= 7.58947 A .
For the period, we have
Heat Dissipated in AC Circuit

33:07, calculus, numeric, > 1 min. T =
176 (part 1 of 1) 10 points ω
= 0.00314159 s .
A 10 Ω resistor, 10 mH inductor, and
100 µF capacitor are connected in series to
a 50 V(rms) source having variable frequency. Finally, we obtain for the heat dissipated in
Find the heat dissipated in the circuit during one period
one period if the operating frequency is twice
2
the resonance frequency. Heat = Irms RT
Correct answer: 0.241661 J. = (2.7735 A)2 (10 Ω) (0.00314159 s)
Explanation: = 0.241661 J .
The power dissipated in the resistor is given
by
2
Pav = Irms R.
Resonance C in RLC
Pav is the rate of energy dissipation. To get 33:07, trigonometry, numeric, > 1 min.
the heat dissipated in the circuit during one 177 (part 1 of 2) 10 points
period, we need to multiply Pav by the period. A series RLC circuit has a resonance fre-
So, we have quency of 636.62 Hz. When it is operating
Heat = Pav T at a frequency of ω > ω0 , XL = 12 Ω and
2
= Irms RT . XC = 8 Ω. Calculate the value of C for the
circuit.
Since the operating frequency is twice the Correct answer: 25.5155 µF.
resonant frequency, we have Explanation:
ω = 2 ω0 Combining XL = ω L = 12 Ω, and XC =
2 1
=√ = 8 Ω, we find
LC ωC
2
=p L = (12 Ω)(8 Ω) C.
(0.01 H) (0.0001 F)
= 2000 rad/s . Substitute this into the expression
The reactances are given by
1
XL = ω L = 20 Ω , ω0 = √ = 4000 rad/s
1 LC
XC = =5Ω.
ωC
to find
The impedance is
q C = 25.5155 µF,
Z = R2 + (XL − XC )2
= 18.0278 Ω . L = 2.44949 mH.
Version One – Homework 39 – Ashley Smith 46
s
µ ¶2
1
178 (part 2 of 2) 10 points = R2+ ωL−
Calculate the value of L for the circuit. ωC
Correct answer: 2.44949 mH. = 662.467 Ω .
Explanation:

Resonance Frequency 181 (part 2 of 4) 10 points


33:07, calculus, numeric, > 1 min. Determine the resonant frequency in kHz.
179 (part 1 of 1) 10 points Correct answer: 1.77941 kHz.
Calculate the resonance frequency of a se- Explanation:
ries RLC circuit for which the capacitance is The frequency at resonance is
8.4 µF and the inductance is 120 mH. r
1 1
Correct answer: 158.522 Hz. f0 =
Explanation: 2π LC
s
The resonance frequency is the frequency at 1 1
which the current becomes maximum, or the =
2 π (0.002 H) (4 × 10−6 F)
impedance becomes minimum. This occurs
= 1.77941 kHz .
when
XL = X C .
In other words, when 182 (part 3 of 4) 10 points
Determine the impedance at resonance.
1
ωL = . Correct answer: 10 Ω.
ωC Explanation:
From this condition, the resonance frequency At resonance
is given by
Z = R = 10 Ω .
1
f= √
2π LC
1 183 (part 4 of 4) 10 points
= p Determine the impedance at a frequency
2 π (0.12 H) (8.4 × 10−6 F)
equal to one-half the resonant frequency.
= 158.522 Hz .
Correct answer: 35 Ω.
Explanation:
Resonance in RLC The angular frequency is
33:07, calculus, numeric, > 1 min. r
180 (part 1 of 4) 10 points 1 1
ω=
A series RLC circuit has resistance 10 Ω, in- 2 LC
ductance 2 mH, and capacitance 4 µF. Deter- = 5590.17 rad/s .
mine the impedance at 60 Hz
Correct answer: 662.467 Ω. The impedance is
Explanation: q
The angular frequency is Z = R2 + (XL − XC )2
s
µ ¶2
ω = 2πf 2
1
= R + ωL−
= 376.991 rad/s . ωC
= 35 Ω .
The impedance is
q
Z = R2 + (XL − XC )2 Resonance L in RLC
Version One – Homework 39 – Ashley Smith 47

33:07, calculus, numeric, > 1 min. The resonance frequency for a series RLC
184 (part 1 of 2) 10 points circuit is
A series RLC circuit in which the 1500 Ω 1
resistor and the 15 nF capacitor is connected ω0 = √
to an ac generator whose frequency can be LC
1
varied. When the frequency is adjusted to =p
50.5 kHz, the current in the circuit reaches a (20 mH) (100 nF)
maximum at 0.14 A. = 22360.7 rad/s .
Determine the inductance.
Correct answer: 0.000662165 H.
Explanation: 187 (part 2 of 4) 10 points
The current in the RLC circuit reaches a Find the amplitude of the current at the res-
maximum when onant frequency.
Correct answer: 5 A.
1 Explanation:
ω0 L =
ω0 C At resonance,
so the inductance is Vmax
Imax =
1 R
L= (100 V)
ω02 C =
(20 Ω)
1
= = 5 A.
[2 π f ]2 C
1
=
[2 π (50.5 kHz)]2 (15 nF) 188 (part 3 of 4) 10 points
= 0.000662165 H . Find the quality factor of the circuit.
Correct answer: 22.3607 .
Explanation:
185 (part 2 of 2) 10 points The quality factor Q0 is given by
Determine the rms value of the generator volt-
ω0 L
age. Q0 ≡
Correct answer: 148.492 V. R
Explanation: (22360.7 rad/s) (20 mH)
=
At resonance, Z = R, so, (20 Ω)
= 22.3607 .
Vrms = Irms R
= (0.0989949 A) (1500 Ω)
189 (part 4 of 4) 10 points
= 148.492 V . Find the amplitude of the voltage across the
inductor at resonance.
Correct answer: 2236.07 V.
Resonance of an AC Circuit
Explanation:
33:07, calculus, numeric, > 1 min.
The amplitude of the voltage across the
186 (part 1 of 4) 10 points
inductor at resonance is
In a series RLC circuit, the inductance is
20 mH, the capacitance is 100 nF, the resis- VL = XL Imax
tance is 20 Ω, and the maximum potential is = ω0 L Imax
100 V.
Find the resonant angular frequency. = (22360.7 rad/s)
Correct answer: 22360.7 rad/s. × (20 mH) (5 A)
Explanation: = 2236.07 V .
Version One – Homework 39 – Ashley Smith 48

Explanation:
Serway CP 21 33
33:07, calculus, numeric, > 1 min.
Given : L = 0.2 mH and
190 (part 1 of 1) 10 points
An RLC circuit is used in a radio to tune C = 2.99792 × 108 m/s .
into an FM radio station broadcasting at
The frequency of the station is equal to the
88.9 MHz. The resistance in the circuit is
resonant frequency of the tuning circuit.
12 Ω, and the capacitance is 1.4 pF.
What inductance should be present in the 1
f0 = √
circuit? 2π LC
Correct answer: 2.28933 H. 1
Explanation: = p
2 π (0.0002 H)(3 × 10−11 F)
= 2.05468 × 106 Hz.
Given : f = 88.9 MHz ,
R = 12 Ω , and
192 (part 2 of 2) 10 points
C = 1.4 pF . What wavelength does it send?
Correct answer: 145.907 m.
The resonance frequency is given by
Explanation:
ω0 2 L C = 1 .
c 2.99792 × 108 m/s
For the angular frequency ω0 , we have λ= = = 145.907 m.
f 2.05468 × 106 Hz
ω0 = 2 π f
RC HighPass Filter
= 2 π (8.89 × 107 Hz)
33:08, calculus, numeric, > 1 min.
= 5.58575 × 108 rad/s . 193 (part 1 of 1) 10 points
Consider the circuit shown in the figure, with
Therefore, we have for the capacitance
resistance 800 Ω and and capacitance 0.09 µF.
1 1 C
L=
ω02 C
1
=
(5.58575 × 108 rad/s)2 Vin R Vout
1
×
(1.4 × 10−12 F)
= 2.28933 × 10−6 H
Calculate the ratio Vout /Vin for angular ve-
= 2.28933 µH . locity 300 rad/s.
Correct answer: 0.021595 .
Explanation:
Serway CP 21 34
The gain is
33:07, trigonometry, numeric, > 1 min.
191 (part 1 of 2) 10 points Vout RI
A resonant circuit in a radio receiver is tuned =
Vin ZI
to a certain station when the inductor has a R
value of 0.2 mH and the capacitor has a value =
Z
of 30 pF. R
Find the frequency of the radio station. =q .
Correct answer: 2.05468 × 106 Hz. R2 + XC2
Version One – Homework 39 – Ashley Smith 49

The capacitive reactance is With G = 0.5, we obtain


1
XC = s
ωC 1 (0.5)2
1 C=
= 2 π (300 Hz) (0.5 Ω) 1 − (0.5)2
(300 rad/s) (9 × 10−8 F)
= 37037 Ω . = 612.588 µF .
So, we obtain for the gain
Vout 800 Ω
=p 195 (part 2 of 2) 10 points
Vin (800 Ω)2 + (37037 Ω)2 What is the gain for a 600 Hz signal?
= 0.021595 . Correct answer: 0.755929 .
Explanation:
RC HighPass Filter 02 At 600 Hz , the capacitive reactance, for the
33:08, calculus, numeric, > 1 min. capacitor found in Part 1, is
194 (part 1 of 2) 10 points
The RC high-pass filter shown in the figure 1
XC =
has a resistance 0.5 Ω. 2πf C
C 1
=
2 π (600 Hz) (0.000612588 F)
= 0.433013 Ω .
Vin R Vout
The gain is

R
What capacitance gives an output signal G= q
that has one-half the amplitude of a 300 Hz R2 + XC2
input signal? 0.5 Ω
Correct answer: 612.588 µF. =p
Explanation: (0.5 Ω) + (0.433013 Ω)2
2

The gain is given by = 0.755929 .


Vout
G=
Vin
RI Changing Power Factor
= 33:09, calculus, numeric, > 1 min.
ZI
R 196 (part 1 of 3) 10 points
=q In an LR circuit, a 120 V(rms), 60 Hz source
R2 + XC2 is in series with a 25 mH inductor and a 20 Ω
R resistor.
=q ¡ 1 ¢2 . What is the maximum rms current?
2
R + ωC Correct answer: 5.42755 A.
Solving the above equation for the capaci- Explanation:
tance, we obtain The inductive reactance is
s
1 G2
C= XL = ω L
ω R 1 − G2
s = 2πf L
1 G2 = 2 π (60 Hz) (0.025 H)
= .
2 π f R 1 − G2 = 9.42478 Ω .
Version One – Homework 39 – Ashley Smith 50

The impedance of an RL circuit is the same So, the capacitance is


as that of an RLC circuit, but without the 1
capacitive reactance. Namely C=
2 π f XL
= 281.448 µF .
q
Z = R2 + XL2
q
= (20 Ω)2 + (9.42478 Ω)2 Hair Drier
= 22.1094 Ω . 33:09, trigonometry, numeric, > 1 min.
199 (part 1 of 2) 10 points
The rms current is A hair drier uses 10 A at 120 V. It is used
Vrms with a transformer in England, where the line
Irms = voltage is 240 V.
Z
120 V What should be the ratio of the turns of the
= transformer (primary to secondary)?
22.1094 Ω
= 5.42755 A . Correct answer: 2 .
Explanation:
The voltage is higher in England, so the
197 (part 2 of 3) 10 points transformer is a step-down transformer.
What is the power factor?
Correct answer: 0.904592 . V ∝n
Explanation: so the ratio of turns (primary to secondary) is
The phase angle in an RL circuit is the same
ne
as that of an RLC circuit, but without the r=
capacitive reactance. Namely n
Ve
µ ¶ =
−1 XL V
φ = tan 240 V
R =
µ ¶ 120 V
−1 9.42478 Ω
= tan = 2.
20 Ω

= 25.2316
200 (part 2 of 2) 10 points
So, the power factor is What current will it draw from the 240 V
line?
cos φ = 0.904592 .
Correct answer: 5 A.
Explanation:
198 (part 3 of 3) 10 points Energy is conserved, so
What capacitor, in microfarads, must be Ps = P p
added in series to make the power factor 1
? IV = Ie Ve
Correct answer: 281.448 µF.
Explanation: IV
Ie =
In an RLC circuit, the phase angle is given by Ve
V
µ
XL − X C
¶ =I
tan φ = . Ve
R I
=
We require cos φ = 1, or φ = 0. From the r
above equation, this condition is satisfied for 10 A
=
2
XL = X C . = 5 A.
Version One – Homework 39 – Ashley Smith 51

Given:
Holt SF 22D 01
33:09, highSchool, numeric, < 1 min. ∆V1 = 12 V
201 (part 1 of 1) 10 points ∆V2 = 2.0 × 104 V
A step-down transformer providing electric- N1 = 21 turns
ity for a residential neighborhood has exactly
2680 turns in its primary. When the potential Solution:
difference across the primary coil is 5850 V,
the potential difference across the secondary ∆V2 N1
N2 =
is 120 V. ∆V1
How many turns are in the secondary? (20000 V)(21 turns)
=
(Round the answer to the nearest whole num- 12 V
ber.) = 35000 turns
Correct answer: 55 turns.
Explanation:
Basic Concept: Holt SF 22D 03
33:09, highSchool, numeric, < 1 min.
N2 203 (part 1 of 1) 10 points
∆V2 = ∆V1
N1 A step-up transformer for long-range trans-
mission of electric power is used to create a
Given: potential difference of 119340 V across the
secondary. The potential difference across
∆V1 = 5850 V
the primary is 117 V and the secondary has
∆V2 = 120 V 25500 turns.
N1 = 2680 turns How many turns are in the primary?
Correct answer: 25 turns.
Solution: Explanation:
∆V2 N1 Basic Concept:
N2 =
∆V1 N2
(120 V)(2680 turns) ∆V2 = ∆V1
= N1
5850 V
= 54.9744 , Given:

which rounds to 55 turns. ∆V1 = 117 V


∆V2 = 119340 V
Holt SF 22D 02 N2 = 25500 turns
33:09, highSchool, numeric, < 1 min.
202 (part 1 of 1) 10 points Solution:
A step-up transformer used in an automobile
∆V1 N2
has a potential difference across the primary N1 =
of 12 V and a potential difference across the ∆V2
secondary of 2.0 × 104 V. There are 21 turns (117 V)25500 turns
=
in the primary coil. 119340 V
How many turns are in the secondary? = 25 turns
Correct answer: 35000 turns.
Explanation:
Basic Concept: Holt SF 22D 04
33:09, highSchool, numeric, < 1 min.
N2 204 (part 1 of 1) 10 points
∆V2 = ∆V1
N1 A potential difference of 0.750 V is needed to
Version One – Homework 39 – Ashley Smith 52

provide a large current for arc welding. The


potential difference across the primary of a Holt SF 22D 06
step-down transformer is 117 V. 33:09, highSchool, numeric, < 1 min.
How many turns must be on the primary 206 (part 1 of 1) 10 points
for each turn on the secondary? A step-down transformer has 525 turns in
Correct answer: 156 . its secondary and 12500 turns in its primary.
Explanation: The potential difference across the primary is
Basic Concept: 3510 V.
N2 What is the potential difference across the
∆V2 = ∆V1 secondary?
N1
Correct answer: 147.42 V.
Given: Explanation:
∆V1 = 117 V Basic Concept:
∆V2 = 0.750 V
Solution: N2
∆V2 = ∆V1
N1 ∆V1 N1
=
N2 ∆V2
117 V Given:
=
0.75 V
= 156 ∆V1 = 3510 V
N1 = 12500 turns
Holt SF 22D 05 N2 = 525 turns
33:09, highSchool, numeric, < 1 min.
205 (part 1 of 1) 10 points
A television picture tube requires a high po- Solution:
tential difference, which in older models is
provided by a step-up transformer. The ∆V1 N2
transformer has 12 turns in its primary and ∆V2 =
N1
2550 turns in its secondary. A potential dif- (3510 V)(525 turns)
ference of 120 V is placed across the primary. =
12500 turns
What is the output potential difference? = 147.42 V
Correct answer: 25500 V.
Explanation:
Basic Concept:
N2 Holt SF 22Rev 35
∆V2 = ∆V1 33:09, highSchool, numeric, < 1 min.
N1
207 (part 1 of 1) 10 points
Given: A transformer is used to convert 120 V to
∆V1 = 120 V 9.0 V for use in a portable CD player. The
N1 = 12 turns primary coil connected to the outlet has
640 turns.
N2 = 2550 turns How many turns does the secondary have?
Solution: Correct answer: 48 turns.
∆V1 N2 Explanation:
∆V2 = Basic Concept:
N1
(120 V)(2550 turns)
=
12 turns N2
∆V2 = ∆V1
= 25500 V N1
Version One – Homework 39 – Ashley Smith 53

Given: primary operates at an rms voltage of 120 V,


the primary impedance is 24 Ω and the trans-
∆V1 = 120 V former is 90 percent efficient. What is the
∆V2 = 9.0 V secondary to primary turn ratio?
N1 = 640 turns Correct answer: 83.3333 .
Explanation:
Solution: The turn ratio is
∆V2 N1 N2 V2
N2 = =
∆V1 N1 V1
(9 V)(640 turns) 10000 V
= =
120 V 120 V
= 48 turns = 83.3333 .

Holt SF 22Rev 36
210 (part 2 of 3) 10 points
33:09, highSchool, numeric, < 1 min.
What is the current in the secondary?
208 (part 1 of 1) 10 points
Correct answer: 0.054 A.
A transformer is used to convert 120 V to 6.3
Explanation:
V in order to power a toy electric train. There
Due to losses, 0.9 of the power in the primary
are 210 turns in the primary.
is transmitted to the secondary. So, we have
How many turns should there be in the
secondary? (Round the answer to the nearest
0.9 I1 V1 = I2 V2
whole number.)
V1
Correct answer: 11.025 turns. 0.9 V1 = I 2 V2
Explanation: Z1
Basic Concept: (120 V)
0.9 120 V = I2 (10000 V) .
(24 Ω)
N2
∆V2 = ∆V1
N1 I2 = 0.054 A .
Given:
211 (part 3 of 3) 10 points
∆V1 = 120 V
What is the impedance in the secondary?
∆V2 = 6.3 V Correct answer: 185.185 kΩ.
N1 = 210 turns Explanation:
The impedance of the secondary is
Solution:
∆V2 N1 V2
N2 = Z2 =
∆V1 I2
(6.3 V)(210 turns) 10000 V
= =
120 V 0.054 A
= 11.025 turns = 185.185 kΩ .

which, rounded downward, equals X turns


Power Dissipated as Heat
Ignition Transformer 33:09, trigonometry, numeric, > 1 min.
33:09, calculus, numeric, > 1 min. 212 (part 1 of 1) 10 points
209 (part 1 of 3) 10 points An ac power generator produces 50 A (rms)
The secondary voltage of an ignition trans- at 3600 V. The voltage is stepped up to
former in a furnace is 10 kV. When the 100000 V by an ideal transformer, and the
Version One – Homework 39 – Ashley Smith 54

energy is transmitted through a long-distance N1 = 20 turns ,


power line that has a resistance of 100 Ω. N2 = 8 turns ,
What percentage of the power delivered N1 5
by the generator is dissipated as heat in the = , and
N2 2
power line? E = 80 Vrms .
Correct answer: 0.18 .
Explanation: The rms voltage across the transformer pri-
The current after being stepped up is mary is
N1
I 1 V1 V1 = V2 ,
I2 = N2
V2
(50 A)(3600 V) so the source voltage is
=
100000 V VS = V R S + V 1
= 1.8 A.
N1
= I 1 RS + V2 .
Then, N2
Plost = I22 R = (1.8 A)2 (100 Ω) = 324 W. The secondary current is

The original power is V2


I2 = .
RL
P = I1 V1 = (50 A)(3600 V) = 180000 W,
The primary current, in terms of the sec-
so the percentage of the power lost is ondary current, is given by
Plost 324 W N2
η= = = 0.18%. I1 = I 2 .
P 180000 W N1

StepDown Transformer Substituting the expression for I2 in the above


33:09, calculus, numeric, > 1 min. equation, we obtain
213 (part 1 of 1) 10 points V 2 N2
A transformer shown in the figure below I1 = .
R L N1
having a primary with 20 turns and secondary
with 8 turns. Substituting the above result for the primary
The load resistor is 50 Ω. current in the expression for VS , we obtain
The source voltage is 80 Vrms .
RS N2 R S N1
VS = V2 + V2 .
N1 R L N2
20 turns

8 turns

Then
50 Ω

· ¸
80 Vrms N1 R L N1
RS = VS − V2
N2 V 2 N2
5 (50 Ω)
If a voltmeter across the load measures =
25 Vrms , what is the source resistance RS ? 2 (25 Vrms )
· ¸
Correct answer: 87.5 Ω. 5
× (80 Vrms ) − (25 Vrms )
Explanation: 2
= 87.5 Ω .
Let : RS = Source Resistor ,
RL = 50 Ω , StepUp Transformer
Version One – Homework 39 – Ashley Smith 55

33:09, trigonometry, numeric, > 1 min. transformer used?


214 (part 1 of 2) 10 points Correct answer: 2 .
A step-up transformer is connected to a gen- Explanation:
erator that is delivering 125 V and 95 A. The The ratio is
ratio of the turns on the secondary to the N2
turns on the primary is 1000 to 1. What volt- R=
N1
age is across the secondary?
V2
Correct answer: 125 kV. =
Explanation: V1
20000 V
=
V ∝n 10000 V
= 2.
so
V2 ns The power is the same
=
V1 np
P = I 1 V1
The stepped up voltage is
= (50000 A) (10000 V)
ns = 5 × 108 W and
V2 = V 1 ·
np P = I 2 V2
Dimensional analysis for Vs : = (25000 A) (20000 V)
= 5 × 108 W .
1 kV
V· = kV
1000 V
217 (part 2 of 2) 10 points
What current would be sent out over the
215 (part 2 of 2) 10 points
power lines if it were transmitted at 20000 V?
What current flows in the secondary?
Correct answer: 25000 A.
Correct answer: 95 mA.
Explanation:
Explanation:
Energy is conserved, so
P
I2 =
Pp = P s V2
(0.5 GW) (1 × 109 W/GW)
Is Vs = IV =
(20000 V)
IV = 25000 A .
Is =
Vs
Dimensional analysis for Is :
Transformer
A · V 1 kV 1000 mA 33:09, trigonometry, numeric, > 1 min.
· · = mA 218 (part 1 of 1) 10 points
kV 1000 V 1A
A transformer has input voltage and current
of 12 V and 3 A respectively, and an output
StepUp Transformer 03 current of 0.75 A. If there are 1200 turns on
33:09, trigonometry, numeric, > 1 min. the secondary side of the transformer, how
216 (part 1 of 2) 10 points many turns are on the primary side?
An electric generating plant produces electric Correct answer: 300 .
energy at a rate of 0.5 GW and a voltage of Explanation:
10000 V. Energy is conserved, so
If it is desired to transmit this power at
20000 V, what must be the turns ratio of the Pp = P s
Version One – Homework 39 – Ashley Smith 56
q
IV = Is Vs Z = XL2 + R2
V Is q
= = (942.478 Ω)2 + (80 Ω)2
Vs I
For the transformer = 945.867 Ω.

So
V ∝n
V 110 V
n V Is I= = = 0.116295 A.
= = Z 945.867 Ω
ns Vs I
Is 221 (part 2 of 4) 10 points
n = ns If the primary is connected to 110 V dc, what
I
is the current through the primary? Disregard
Transformer 02 initial effects.
33:09, trigonometry, numeric, > 1 min. Correct answer: 1.375 A.
219 (part 1 of 1) 10 points Explanation:
A transformer consists of two coils of wire V 110 V
wound on a common toroidal iron core. The I= = = 1.375 A.
R 80 Ω
mutual inductance of the pair is 100 mH and
the current in the first coil decreases from
222 (part 3 of 4) 10 points
13 A to 0 in 0.2 s. What is the induced emf in
In case 1, find the power dissipated in the
the second coil?
resistance.
Correct answer: 6.5 V.
Correct answer: 1.08197 W.
Explanation:
Explanation:
EMF due to mutual induction is
In case 1,
∆I
E = −M P = I 2 R = (0.116295 A)2 (80 Ω)
∆t
13 A = 1.08197 W.
= (0.1 H)
0.2 s
= 6.5 V
223 (part 4 of 4) 10 points
In case 2, find the power dissipated in the
resistance.
Coils of a Transformer
Correct answer: 151.25 W.
33:99, trigonometry, numeric, > 1 min.
Explanation:
220 (part 1 of 4) 10 points
In case 2,
The primary coil of a certain transformer has
an inductance of 2.5 H and a resistance of P = I 2 R = (1.375 A)2 (80 Ω)
80 Ω. If the primary coil is connected to an
= 151.25 W.
ac source with a frequency of 60 Hz and a
voltage of 110 V rms, what is the rms current
through the primary? Current in an AC Circuit
Correct answer: 0.116295 A. 33:99, trigonometry, numeric, > 1 min.
Explanation: 224 (part 1 of 3) 10 points
An ac voltage of the form v = vmax sin 2πf t,
with frequency 60 Hz and maximum voltage
XL = 2 π f L 170 V, is applied across a 60 W light bulb.
= 2 π (60 Hz)(2.5 H) When the voltage is first applied, what is the
= 942.478 Ω . current through the circuit?
Version One – Homework 39 – Ashley Smith 57

1. the maximum current


XL = 2 π f L = 2 π (60 Hz)(0.7 H)
2. 0 correct = 263.894 Ω Ω.
Now use V 2 = VR2 + VL2 , we get
3. the rms current
q
Explanation: VL = V 2 − VR2
The current in the circuit is q
= (120 V)2 − (40 V)2
I = Imax sin 2πf t
= 113.137 V.
when t = 0, I = 0; when t = T /4 = 1/4f , Therefore,
1 VL 113.137 V
I = Imax sin 2πf = Imax I= = = 0.428722 A.
4f XL 263.894 Ω Ω

where the maximum current is


Holt SF 22Rev 40
2P 33:99, highSchool, numeric, < 1 min.
Imax = 228 (part 1 of 1) 10 points
vmax
A transformer has 22 turns of wire in its pri-
the rms current is mary and 88 turns in its secondary. A poten-
tial difference of 110 V ac is applied to the
Imax
I= √ primary.
2 What is the output potential difference?
Correct answer: 440 V.
225 (part 2 of 3) 10 points Explanation:
When t = T /4, where T is the period (equal Basic Concept:
to 1/f ) of the applied voltage, what is the N2
current in the circuit? ∆V2 = ∆V1
N1
Correct answer: 0.705882 A.
Explanation: Given:

226 (part 3 of 3) 10 points ∆V1 = 110 V


What is the rms current in the circuit? N1 = 22 turns
Correct answer: 0.499134 A. N2 = 88 turns
Explanation:
Solution:
Fluorescent Lamp ∆V1 N2
33:99, trigonometry, numeric, > 1 min. ∆V2 =
N1
227 (part 1 of 1) 10 points (110 V)(88 turns)
A 0.7 H inductor is connected in series with a =
22 turns
fluorescent lamp to limit the current drawn by = 440 V
the lamp. If the combination is connected to
a 60 Hz, 120 V line, and if the voltage across
the lamp is to be 40 V, what is the current Holt SF 22Rev 41
in the circuit? (The lamp is a pure resistive 33:99, highSchool, numeric, < 1 min.
load.) 229 (part 1 of 1) 10 points
Correct answer: 0.428722 A. The potential difference in the lines that carry
Explanation: electric power to homes is typically 20.0 kV.
Version One – Homework 39 – Ashley Smith 58

How many turns must be on the primary Solution:


for each turn on the secondary if the output
potential difference is 117 V? (Round the Pdissipated = I2 2 R
answer to the nearest whole ratio.) µ ¶2
Correct answer: 170.94 . P
= R
Explanation: ∆V2
¶2 µ ¶
Basic Concept:
µ
P R
= d
N2 ∆V2 d
∆V2 = ∆V1 ¶2 µ
N1 5 × 106 W
µ ¶

= 0.00045
Given: 510000 V m
· (644000 m)
∆V1 = 20.0 kV = 27.8547 kW
∆V2 = 117 V

Solution:
231 (part 2 of 2) 10 points
N1 ∆V1 b) How much power would be lost through
=
N2 ∆V2 transmission if the generator’s output poten-
20000 V tial difference were not stepped up?
=
117 V Correct answer: 357778 kW.
= 170.94 Explanation:
Given:
Holt SF 22Rev 43 ∆V1 = 4500 V
33:99, highSchool, numeric, > 1 min.
230 (part 1 of 2) 10 points Solution:
A generator supplies 5.0 ×103 kW of power.
The output potential difference is 4500 V be- Pdissipated = I2 2 R
fore it stepped up to 510 kV. The electricity µ
P
¶2
travels 410 miles (6.4400 ×105 m) through = R
∆V1
a transmission line that has a resistance per ¶2 µ ¶
unit length of 4.5 ×10−4 Ω/m.
µ
P R
= d
a) How much power is lost through trans- ∆V1 d
mission of the electrical energy along the ¶2 µ
5 × 106 W
µ ¶

line? = 0.00045
Correct answer: 27.8547 kW. 4500 V m
Explanation: · (644000 m)
Basic Concepts: = 357778 kW
P = I∆V = I 2 R
The power dissipated by the alternating
∆V current whose potential difference has not
R=
I been stepped up is more than the power gen-
Given: erated. This indicates that without stepping
up its potential difference, electricity cannot
P = 5.0 × 103 kW be conveyed very far along a transmission line.
∆V2 = 510 kV
R Holt SF 22Rev 44
= 4.5 × 10−4 Ω/m
d 33:99, highSchool, numeric, > 1 min.
D = 644000 m 232 (part 1 of 2) 10 points
Version One – Homework 39 – Ashley Smith 59

The alternating potential difference of a gen- average power delivered by the generator is
erator is represented by the equation given by

emf = (245 V)(sin 560 t), Pav = Irms Vrms cos(φ)


= (9 A) (180 V) cos(−37◦ )
where emf is in volts and t is in seconds. = 1293.79 W .
Find the frequency of the potential differ-
ence of the source. Note that, in the above expression, the phase
Correct answer: 89.1268 Hz. angle is −37◦ , since the voltage lags the cur-
Explanation: rent. The power dissipated in the resistor is
Basic Concepts: given by
2
Pav = Irms R.
emf = Vmax (sin ωt)
So, we obtain for the resistance
ω = 2πf
Pav
R= 2
Given: Irms
1293.79 W
emf = (245 V)(sin 560 t) =
(9 A)2
= 15.9727 Ω .
Solution:
ω
f= Inductance in a Circuit

560 rad/s 33:99, calculus, numeric, > 1 min.
= 235 (part 1 of 1) 10 points

= 89.1268 Hz As a way of determining the inductance of
a coil used in a research project, a student
first connects the coil to a 12 V battery and
233 (part 2 of 2) 10 points measures a current of 0.63 A. The student
Find the maximum potential difference out- then connects the coil to a 24 V(rms), 60 Hz
put of the source. generator and measures an rms current of
Correct answer: 245 V. 0.57 A.
Explanation: What is the inductance?
Solution: Correct answer: 0.0996058 H.
Explanation:
Vmax = 245 V The resistance of the wire of the coil is given
by Ohm’s Law as
V
Impedance of RLC mid3 R=
I
33:99, calculus, numeric, > 1 min. 12 V
234 (part 1 of 1) 10 points =
0.63 A
In a certain series RLC circuit, the rms cur-
= 19.0476 Ω .
rent is 9 A, the rms voltage is 180 V and the
current leads the voltage by 37◦ . What is the The impedance of the coil is
total resistance of the circuit?
Correct answer: 15.9727 Ω. Vrms
Z=
Explanation: Irms
To solve this problem, we use the result that 24 V
=
the average power delivered by the generator 0.57 A
is dissipated as heat in the resistor. The = 42.1053 Ω .
Version One – Homework 39 – Ashley Smith 60

The coil has an ohmic resistance and an in- We notice that, when the frequency is very
ductive reactance. Therefore, its impedance large, ω L is very large, and the impedance
is of the bottom branch is very large; so the
q the bottom branch carries negligible current.
Z = R2 + XL2 1
Also, will be negligible compared to Rt ;
p ωC
= R 2 + ω 2 L2 so the impedance of the top branch reduces to
. Zt = Rt . The current that flows in the power
supply and the top branch is
From the above equation, the inductance is
1p 2 Vrms
L= Z − R2 It =
ω Zt
1 p 2
= Z − R2 Vrms
2πf =
Rt
1
= 45 V
2 π (60 Hz) =
q 200 Ω
(42.1053 Ω)2 − (19.0476 Ω)2 = 0.225 A .
= 0.0996058 H .

237 (part 2 of 2) 10 points


RMS Currents Find the rms current delivered by the power
33:99, calculus, numeric, > 1 min. supply when the frequency is very small.
236 (part 1 of 2) 10 points Correct answer: 0.45 A.
In the figure shown, the capacitance is 200 µF Explanation:
and the inductance is 3 mH. The resistance Now, when the frequency is very small,
in the top branch is 200 Ω, and in the bottom 1
branch is 100 Ω. The potential of the power → ∞ and ω L → 0. This means that the
ωC
supply is 45 V(rms). top branch, with very high impedance, car-
C ries negligible current; while the impedance of
Rt
the bottom branch reduces to Rb . The cur-
rent that flows in the power supply and the
bottom branch is

Vrms Vrms
Ib =
Zb
Vrms
Rb L =
Rb
Find the rms current delivered by the power 45 V
supply when the frequency is very large. =
100 Ω
Correct answer: 0.225 A. = 0.45 A .
Explanation:
The impedance of the top branch is
s
¶2
Serway CP 21 66
µ
2 1
Zt = R t + . 33:99, trigonometry, numeric, > 1 min.
ωC
238 (part 1 of 1) 10 points
The impedance of the bottom branch is Suppose you wish to use a transformer as an
q impedance-matching device between an au-
Zb = Rb2 + (ω L)2 . dio amplifier that has an output impedance
Version One – Homework 39 – Ashley Smith 61

of 8 kΩ and a speaker that has an input we have


impedance of 8 Ω. 1
What should be the ratio of primary to L=
4 π2 C f02
secondary turns on the transformer?
Correct answer: 31.6228 . = 5.17877 × 10−6 H = 5.17877 µH .
Explanation:

Given : Z1 = 8 kΩ and Small Transformer


33:99, trigonometry, numeric, > 1 min.
Z2 = 8 Ω .
240 (part 1 of 3) 10 points
We know that A small transformer is used to supply an ac
voltage of 6 V to a model-railroad lighting
N1 V1 I1 Z1 circuit. The primary has 220 turns and is
= =
N2 V2 I2 Z2 connected to a standard 110 V, 60 Hz line.
Although the resistance of the primary may
using the impedance
be neglected, it has an inductance of 150 mH.
N1 Z1 I1 How many turns are required on the sec-
= . ondary winding?
N2 Z2 I2
Correct answer: 12 turns.
But Explanation:
I1 V2 N2
= = .
I2 V1 N1 Vs 6V
so Ns = Np =
Vp 110 V
N1 Z 1 N2 = 12 turns.
=
N2 Z 2 N1
N12 Z1 241 (part 2 of 3) 10 points
=
N22 Z2 If the transformer is left plugged in, what
current is drawn by the primary when the
r r
N1 Z1 8000 Ω
= = = 31.6228 . secondary is open?
N2 Z2 8Ω
Correct answer: 1.94523 A.
Explanation:
Simple LC Circuit 02 The impedance of the primary is
33:99, trigonometry, numeric, > 1 min.
239 (part 1 of 1) 10 points Z = XL = 2 π f L
What value of inductance should be used in = 2 π (60 Hz)(0.15 H) = 56.5487 Ω.
series with a capacitor of 1.5 pF to form an os-
cillating circuit that will radiate a wavelength Thus,
of 5.25 m? V 110 V
Correct answer: 5.17877 µH. I= = = 1.94523 A.
Z 56.5487 Ω
Explanation:
For λ = 5.25 m ,
242 (part 3 of 3) 10 points
What power is drawn by the primary when
c 2.99792 × 108 m/s
f0 = = the secondary is open?
λ 5.25 m Correct answer: 0 W.
= 5.71033 × 107 Hz. Explanation:
P = I V cos φ, but
From
1 R
f0 = √ , cos φ = = 0.
2π LC Z
Version One – Homework 39 – Ashley Smith 62

Therefore, P = 0 W. Explanation:
The total power consumed in the secondary is
StepDown Transformer 02
33:99, calculus, numeric, > 1 min. P2 = 8 × P = 320 W .
243 (part 1 of 3) 10 points
A transformer operating from 120 V (rms) We want to find R1 such that
supplies a 12 V lighting system for a garden.
Eight lights, each rated 40 W , are installed P2 = I12 R1 ,
parallel.
Find the equivalent resistance of the sys-
where I1 is the primary current when the
tem.
transformer is used. The primary current is
Correct answer: 0.45 Ω.
found from
Explanation:
Since the lights are installed parallel, the volt- I 2 V2
age is the same across each. For each light, I1 =
V1
V22 (26.6667 A) (12 V)
P = =
R 120 V
V22 = 2.66667 A .
R=
P
(12 V)2 The required resistance is
=
40 W
= 3.6 Ω . P2
R1 =
I12
For the 8 lights parallel,
320 W
1 8 =
= (2.66667 A)2
Req R = 45 Ω .
8
=
3.6 Ω
Req = 0.45 Ω . Voltage Drop Across Inductor
33:99, calculus, multiple choice, > 1 min.
246 (part 1 of 1) 10 points
244 (part 2 of 3) 10 points
A coil of wire of negligible resistance, acting
What is the current in the secondary circuit?
as an ”inductor”, is connected into a simple
Correct answer: 26.6667 A.
electrical circuit as shown.
Explanation:
The current in the secondary circuit is
V2 capacitors,
I2 = ? V inductor
Req resistors, etc
12 V
=
0.45 Ω G
= 26.6667 A . generator
The voltage drop (or emf) across the induc-
245 (part 3 of 3) 10 points tor depends on which of the following:
What single resistance, connected across the
120 V supply, would consume the same power 1. the current flowing through it
as when the transformer is used?
Correct answer: 45 Ω. 2. its capacitance
Version One – Homework 39 – Ashley Smith 63

3. the number of turns of wire in the coil


correct

4. the charge accumulated in the coil


Explanation:
∆I
E =L
∆t

Inductance L depends on the size, shape,


number of turns of wire, presence of a mag-
netic iron core. E depends on L and the time
rate of change of the current through the in-
ductor.
Version 095 – MT4 – chiu – (58655) 1
This print-out should have 22 questions.
Multiple-choice questions may continue on
the next column or page – find all choices
before answering.

001 10.0 points R


Two laser beams of equal intensity propagate
along parallel paths through a gas that makes
the beams visible to the eye. One laser has
a wavelength of λ1 = 638.19 nm and the sec-
ond has a wavelength of λ2 = 434.81 nm. You I
view these beams from the side, i.e. perpen- I
dicularly to the propagation path, so that all
you see is light scattered off the gas. Compute Find the magnitude of the displacement
the ratio of the intensity of the light of each current.
wavelength that reaches your eye, I2 /I1 . As-
sume the distance from your eye to each beam 1 2 dE

1. |ID | = πR
is the same. µ0 ǫ0 dt
1. 5.98614

2 dE

2. 4.46122 2. |ID | = I + ǫ0 πR
dt
3. 3.53722
dE
3. |ID | = µ0 ǫ0 πR2

4. 4.23341
5. 3.83883 dt

6. 4.83182 2 dE

4. |ID | = ǫ0 πR correct
7. 4.6409 dt
8. 5.21392

1 dE
9. 3.95471 5. |ID | = ǫ0 R
2 dt
10. 5.6154
Explanation:
The Ampere-Maxwell law can be under-
Correct answer: 4.6409.
stood in the following way:
~ • d~ℓ = µ0 I + µ0 ǫ0 dΦE
I
Explanation: B
The laser with the higher frequency is scat- dt
tered more. The scattered intensity varies like = µ0 (I + ID ) ,
ω4. where ID = ǫ0 dΦE /dt. Plugging in the given
values, we obtain

2 dE

I2 /I1 = (ω2 /ω1 )4 = (λ1 /λ2 )4 = 4.6409 . |ID | = ǫ0 πR .
dt

003 (part 2 of 2) 5.0 points


002 (part 1 of 2) 5.0 points Using the convention that clockwise around
N closely spaced turns of wire are wound in the ring is positive and counterclockwise is
the direction indicated on a hollow, plastic negative, determine B ~ at the location marked
ring of radius R, with circular cross section, ×.
as in the figure below. The coil carries a
current I with direction as shown in the figure. ~ =− µ 0 R dE
1. B N I − µ0 ǫ0
Assume the ring lies entirely within a region 2πR 2 dt

of uniform electric field E that points into the ~ = µ 0 R dE
2. B N I − µ0 ǫ0 correct
page and is decreasing: dE/dt = −|dE/dt|. 2πR 2 dt
Version 095 – MT4 – chiu – (58655) 2


~ µ0 R dE 3. E(h + 2vt)
3. B = − N I + µ0 ǫ0
2πR 2 dt

~ µ0 R dE Explanation:
4. B = N I + µ0 ǫ0
2πR 2 dt The emf about the loop must be equal to
Explanation: the line integral of the electric field around
First we choose the orientation of the area. the loop. In the neighborhood of segment
It is simpler to work with positive fluxes, so 23, B is not changing, so we may reason that
let the area vector point into the page; this E is zero along it. For segments 12 and 34,
orientation means the path of integration is note that E is independent of x, so E along
clockwise about the ring. these segments must either be zero or the sum
must cancel. The emf must be entirely along
By inspection, there are N wires piercing segment 41, so Eh is the correct choice.
the defined area with current flow in the same
direction as the area vector, so the enclosed 005 (part 2 of 2) 5.0 points
current is N I. The flux of E through the area
is positive, but it is decreasing, so the displace-
ment current is −ǫ0 πR2|dE/dt|. Plugging
these into the Ampere-Maxwell equation,

dE
I
~ • d~ℓ = µ0 N I − µ0 ǫ0 πR
B 2
dt

~ = µ 0 R dE
B N I − µ0 ǫ0
2πR 2 dt

004 (part 1 of 2) 5.0 points Now apply the Ampere-Maxwell law to the
In Professor Feynman’s lecture on the EM loop 12561. The magnitude of the LHS is
waves modeled in the figures below, a con- found to be Bb. What is the magnitude of the
stant, uniform B field points in the −z- RHS?
direction and propagates outward at a con-
stant speed v in the ±x-directions. The B
field originates from a current sheet located 1. ǫ0 µ0 Bvb
in the yz-plane.
2. ǫ0 µ0 Evb correct

3. ǫ0 µ0 Bvh

Explanation:
We are looking for a term proportional to
dE/dt. Since there is only one option con-
taining E, it is the only choice. Physically,
ΦE = Ebvt, so dΦ/dt = Ebv.
Applying Faraday’s law to loop 12341, one
finds that the the magnitude of the RHS of 006 (part 1 of 2) 5.0 points
Faraday’s law is given by Bhv. What is the A point source at O emits light isotropically.
magnitude of the LHS? Denote the power which the point source ra-
diates by P . A small flat surface is placed at
1. Bh D, which is a distance r from O. This sur-
face has an area A and is perpendicular to the
2. Eh correct radial vector OD.
Version 095 – MT4 – chiu – (58655) 3
4. 4.23717e-09
5. 2.70859e-08
6. 9.67757e-09
r D 7. 7.12857e-09
O 8. 4.08372e-08
9. 3.04025e-09
Find the intensity I of the incident light at
10. 3.31584e-09
D.
2P Correct answer: 4.18929 × 10−9 N/m2 .
1. I =
3A Explanation:
P Note: The time-averaged energy density is
2. I =
2A
I P
P ū = = .
3. I = c 4 c π r2
6 π r2
P Then the average pressure on the surface is
4. I =
2 π r2 1 2
P pressure = ū + 2 ū
5. I = 3 3
3A 5 5P
P = ū =
6. I = 3 12 π c r 2
A 5 (116 W)
P =
7. I = 12 π (2.99792 × 108 m/s) (3.5 m)2
3 π r2
= 4.18929 × 10−9 N/m2 .
P
8. I =
π r2
P 008 10.0 points
9. I =
4A
P

3.65 µm
10. I = correct
4 π r2
Explanation:
Notice that the emission is isotropic. So θ
the total power P is associated with the
entire spherical surface enclosing the point
source. In other words the average intensity Determine the maximum angle θ for which
of a spherical wave is given by the light rays incident on the end of the light
pipe shown in the figure above are subject
∆P P to total internal reflection along the walls of
I= = .
∆A 4 π r2 the pipe. The pipe of diameter 3.65 µm has
an index of refraction of 1.22 and the outside
007 (part 2 of 2) 5.0 points medium is air.
Consider the setup described in Part 1, where 1. 63.1211
1 2. 37.3997
the surface absorbs of the light.
3 3. 53.0347
If P = 116 W, r = 3.5 m, and A = 4 mm2 , 4. 65.0828
find the average pressure on the surface. 5. 28.8009
1. 4.18929e-09 6. 61.267
2. 7.84057e-09 7. 44.3353
3. 2.32016e-08 8. 71.9894
Version 095 – MT4 – chiu – (58655) 4
9. 74.8957 An electron is oscillating along the x-axis
10. 45.7392 about the origin, and its position is given by:

Correct answer: 44.3353◦. x(t) = x0 sin(ωt) ,


Explanation:
where x0 is a constant and ω is the angular
frequency of oscillation. What is the intensity
Given : nair = 1 , of emitted radiation at some point x on the
x-axis, |x| > x0 ?
npipe = 1.22 , and
r = 1.825 µm . 1. 0 correct
 2
µ0 kqωx0 cos(ωt)
φc 2.
φ c x
d 2
kqω 2 x0 sin(ωt)

1
3.
θ µ0 c6 x
2
kqω 2 x0 sin(ωt)

1
4. ǫ0
2 c2 x
The critical angle φc for total internal re- 2
kqω 2 x0 sin(ωt)

flection is given by 1
5.
nair µ0 c5 x
sin φc = .
npipe Explanation:
Radiation will only be emitted in direc-
  tions for which the perpendicular component
nair of the acceleration of the electron is nonzero.
φc = arcsin
npipe Since the electron is oscillating along the x-
 
1 axis, its acceleration will always be in the
= arcsin
1.22 ±x-directions. Since a⊥ = 0 for all points

= 55.052 . along the x-axis, 0 is the correct choice.

From the right triangle the angle φ at the 010 (part 1 of 2) 5.0 points
left-hand interface is An ideal circuit contains an inductor of induc-
tance 135 mH and a capacitor of capacitance
φ = 90◦ − φc
38.3 µF. At what frequency will this circuit
= 34.948◦ . oscillate? Answer in units of Hz.
1. 63.2419
Applying Snell’s Law, we have
2. 69.9929
nair sin θ = npipe sin φ 3. 75.77
4. 60.5288
  5. 57.0649
npipe sin φ 6. 103.47
θ = arcsin
nair 7. 119.787
1.22 sin 34.948◦ 8. 62.5258
 
= arcsin 9. 111.957
1
10. 56.2473
= 44.3353◦ .
Correct answer: 69.9929 Hz.
009 10.0 points Explanation:
Version 095 – MT4 – chiu – (58655) 5
In this case, we obtain:
Let : L = 135 mH = 0.135 H , and
C = 38.3 µF = 3.83 × 10−5 F . Q = 314.825 µC .
The natural frequency is given by
r
1 1
f=
2π LC 012 (part 1 of 3) 3.0 points
s
1 1 You spend the day at the beach. At noon
=
2 π (0.135 H) (3.83 × 10−5 F) you look at sky a little bit above the horizon
over the ocean. Describe the polarization of
= 69.9929 Hz . light that comes from that portion of the sky
to your eye. What is the best choice?
011 (part 2 of 2) 5.0 points
The capacitor is first charged to 360 µC and 1. It is predominantly unpolarized.
then connected to the inductor, forming an
ideal LC circuit. An ammeter measures cur- 2. It is predominantly polarized roughly ver-
rent flow in the circuit — when the current is tical to the horizon.
76.7853 mA, how much charge remains on the
capacitor? Answer in units of µC. 3. It is predominantly polarized roughly par-
1. 187.373 allel to the horizon. correct
2. 90.6688
3. 177.863 Explanation:
4. 350.09 For the case of blue noon sky, the incident
5. 328.668 sunlight is unpolarized (or randomly polar-
6. 314.825 ized in a plane perpendicular to its propaga-
7. 191.601 tion). However, since you are looking away
8. 144.043 from the Sun at the horizon the direct sunlight
9. 282.044 never reaches your eye. It is the rescattered
10. 308.049 light from the air molecules in that portion
of the sky that does reach your eye. The
Correct answer: 314.825. molecules reradiate in the same plane as the
Explanation: plane of the polarization of the sunlight. Near
the horizon, that plane is roughly parallel to
the horizon. From your view on the beach
Let : Q0 = 360 µC and you see light that is predominantly horizon-
I = 76.7853 mA . tally polarized.
Since there are no dissipative elements in the
circuit (i.e. resistors), energy is conserved 013 (part 2 of 3) 3.0 points
during the oscillation: Later in your day at the beach, the Sun sets
in the same portion of the sky a little bit
1 Q20 1 Q(t)2 1 2 above the horizon over the ocean. Describe
E= = + LI .
2 C 2 C 2 the polarization of light that comes from the
By solving this equation for Q(t), we can Sun to your eye. What is the best choice?
determine the charge on the capacitor for any
measured current I: 1. It is predominantly polarized roughly par-
s
 2 allel to the horizon.
2 I
Q(t) = Q0 − .
ω 2. It is predominantly unpolarized. correct
Version 095 – MT4 – chiu – (58655) 6

magnitude of the acceleration is 2 × 1018 m/s2


3. It is predominantly polarized roughly ver- (this large acceleration is possible because the
tical to the horizon. electron has a very small mass). We make
observations at location A at a horizontal dis-
Explanation: tance d = 10 m from the electron as in the
When sunlight is viewed head-on, one sees figure below.
mostly unpolarized light, since there is no pre-
ferred direction of polarization. Sunset light
partly consists of an uninterrupted light beam
coming directly from the sun, which is neces- θ
sarily unpolarized. However, some of it might A
also be rescattered light produced through Electron
scattering of the unpolarized sunlight by air d
molecules in the atmosphere. Even for this
component of light, there is no preferred az- Take the +x direction to be toward the
imuthal direction since the incident light ray right, the +y direction to be upward, and
(from the sun to the scattering molecule) and the +z direction to be out of the page (to-
the rescattered light ray (from the scattering ward you). Throughout this problem, use the
molecule to the eye) travel in the same direc- following constants:
tion. Hence even this rescattered component
will be unpolarized, to the extent that we as- c = 3 × 108 m/s
sume scattering has no azimuthal preference. e = 1.6 × 10−19 C
1
014 (part 3 of 3) 4.0 points = 9 × 109 N · m2 /C2 .
4πǫ0
Now consider the sky colors in the proximity
of the sun at sunset. One day it is red and When the radiative electric field reaches
another day it is orange. Which case corre- location A, what is the direction of that ra-
sponds to having a stronger scattering effect diative electric field at location A? Choose the
on the original white beam? unit vector corresponding to this direction.

1. Red correct 1. h0, 0, −1i

2. Neither — sunset color is not dependent 2. h1, 0, 0i


on scattering effects
3. h−1, 0, 0i
3. Orange
4. h0, −1, 0i
Explanation:
Let us assume the frequency composition 5. h0, 1, 0i correct
of original sunlight to be such that it is white
light. The more the scattering that has taken 6. h0, 0, 1i
place, the more the higher frequency compo-
nents of light will be removed from the orig- Explanation:
inal white beam and hence the redder it will The radiative electric field is proportional
be. to −q ~a⊥ = +e~a⊥ , which is upward. There
is an accompanying magnetic field out of the
015 (part 1 of 3) 3.0 points page because the direction of propagation E ~×
An electron is initially at rest. At time t = 0, ~ is to the right.
B
it is accelerated at an angle θ = 18◦ with re-
spect to the vertical for a very short time. The 016 (part 2 of 3) 3.0 points
Version 095 – MT4 – chiu – (58655) 7
What is the magnitude of this radiative elec- electric field. Since Erad is in the +y-direction
tric field? while the Poynting vector E ~ ×B ~ points in the
1. 3.49306e-09 +x-direction, it must be that Brad points in
2. 1.52266e-09 the +z-direction, out of the page; F = q~v × B ~
3. 1.94547e-09 tells us the force must be in the +x-direction
4. 3.04338e-09 (to the right).
5. 2.07846e-09
6. 1.26611e-09 018 10.0 points
7. 2.22524e-09 Two light pulses are emitted simultaneously
8. 1.39032e-09 from a source. The pulses take parallel paths
9. 2.3236e-09 to a detector 7.05 m away, but one moves
10. 3.15138e-09 through air and the other through plastic.
Determine the difference in the pulses’
Correct answer: 3.04338 × 10−9 N/C. times of arrival at the detector. (Assume
Explanation: the index of refraction of plastic is 1.271 and
The magnitude of the upward radiative that of air is 1.)
electric field is 1. 9.16107
2. 8.512
3. 6.30133

~ 1 e~a⊥
E = 4. 6.2272
rad 4πǫ0 c2 r 5. 6.3685
= 3.04338 × 10−9 N/C . 6. 8.9125
7. 12.0497
017 (part 3 of 3) 4.0 points 8. 7.8088
Just after the radiative field has reached 9. 5.3521
location A, what is the direction of the 10. 7.3656
magnetic f orce on a positive charge that had
been at rest before the radiative field arrived Correct answer: 6.3685 ns.
at location A? Note that the static electric
Explanation:
field is much smaller than the radiative elec-
tric field at this location so that only the
radiative electric field will play a role in deter-
mining your answer. Choose the unit vector Let : nplastic = 1.271 ,
corresponding to this direction. nair = 1 ,
ℓ = 7.05 m , and
1. h−1, 0, 0i c = 3 × 108 m/s .

2. h1, 0, 0i correct The time difference is


3. h0, 0, 1i ∆t = tplastic − tair
ℓ ℓ
4. h0, −1, 0i = − .
vplastic vair
5. h0, 1, 0i
Since the speed of light in a medium of
c
6. h0, 0, −1i refractive index n is v = ,
n
Explanation:
A positive charge is accelerated upward n
plastic nair 
(the +y direction) by the upward radiative ∆t = ℓ −
c c
Version 095 – MT4 – chiu – (58655) 8
7.05 m The first polarizer transmits light that is
= (1.271 − 1)
3 × 108 m/s polarized perpendicular to the long molecules.
 
1 ns
× 020 (part 2 of 2) 5.0 points
1 × 10−9 s
After passing through the second polarizer,
= 6.3685 ns . what is the polarization of the EM wave?

019 (part 1 of 2) 5.0 points 1. along the x-axis correct

2. along the y-axis

3. along sin θ x̂ + cos θ ŷ

4. none — no light gets through

5. along the z-axis

6. along cos θ x̂ + sin θ ŷ

Explanation:
The first polarizer transmits light that is
polarized perpendicular to the long molecules;
in this case, the polarization of the wave
An EM wave polarized along the x-axis and after passing through the first polarizer is
propagating in the +z direction is incident along sin θ x̂ + cos θ ŷ. The second polarizer
upon a pair of polarizers. The long molecules transmits light polarized along the x-axis, so
of the first polarizer make an angle θ with only the x-component of the EM wave passes
the +x-axis, while the long molecules of the through, leaving a wave that is once again po-
second polarizer are parallel to the y-axis, as larized along the x-axis (though reduced in
shown in the figure. intensity).
After passing through the first polarizer,
021 (part 1 of 2) 5.0 points
what is the polarization of the EM wave?
Consider the circuit shown in the figure
1. along the x-axis
where the switch has been initially in position
d for a long period of time so that a steady
2. along the z-axis
current has been achieved. Then, at time
t = 0 the switch is set to position c.
3. parallel to the direction of the long
molecules

4. none — no light gets through

5. along the y-axis


R
b
L aS c

E d
6. perpendicular to the direction of the long
molecules correct
Explanation:
Version 095 – MT4 – chiu – (58655) 9
What is the current through the inductor R = 97.6 Ω , and
at time t = 0? Let positive current flow be I0 = 3.3 A .
defined as from a to b.
After switching to c, the total stored energy
E is the magnetic energy at t = 0:
1. −
R
E
1
UL = L I02 ,

2. correct 2
R t=0

3. 0 which will be dissipated in the resistor be-


tween 0 < t < ∞ . Applying conservation of
Explanation:
energy,
The loop equation while S is at d is given
by 1
UR = UL = L I02

E − (Va − Vb ) − IR = 0 . t=0→∞ t=0 2
Immediately after the switch is thrown to 1
= (3 H) (3.3 A)2 = 16.335 J .
position c, the current remains at the max- 2
imum amplitude it had immediately before
the switch was thrown, due to the magnetic
inertia of the inductor. That amplitude is de-
termined by the value of I when dI/dt = 0,
or
E
E − IR = 0 → I = .
R
The current direction must remain unchanged
when the switch is thrown, so flow must be
positive, from a to b.

022 (part 2 of 2) 5.0 points


Consider an inductance of 3 H, resistance of
97.6 Ω, and initial current of 3.3 A at t = 0.
Find the total energy dissipated through
the resistor during the time interval t = 0 to
∞.
1. 46.08
2. 144.06
3. 0.9
4. 1.08
5. 74.06
6. 1.225
7. 1.62
8. 50.625
9. 16.335
10. 217.8

Correct answer: 16.335.


Explanation:

Let : L = 3 H ,
Version One – Homework 36 – Ashley Smith 1

This print-out should have 136 questions,


check that it is complete. Multiple-choice 002 (part 2 of 3) 10 points
questions may continue on the next column Compute the magnetic flux through each
or page: find all choices before making your turn.
selection. The due time is Central time. Correct answer: 3.33099 × 10−8 Wb.
Explanation:
Air Core Solenoid 02 The magnetic flux thru each turn is
32:01, calculus, numeric, > 1 min.
001 (part 1 of 3) 10 points Φ=BA ³π ´
A 40 mA current is carried by a uniformly =B d2
4
wound air-core solenoid as shown in the figure π
below. = (0.000188496 T) (0.015 m)2
4
12 cm −8
= 3.33099 × 10 Wb .
40 mA

15 mm 003 (part 3 of 3) 10 points


Compute the inductance of the solenoid.
Correct answer: 0.374737 mH.
Explanation:
The solenoid has 450 turns The inductance of the solenoid is

Compute the magnetic field inside the NΦ


L=
solenoid. I
Correct answer: 0.000188496 T. (450) (3.33099 × 10−8 Wb)
=
Explanation: (0.04 A)
= 0.374737 mH .
Let : N = 450 ,
` = 12 cm , Automobile Starter Motor
d = 15 mm , and 32:01, trigonometry, numeric, > 1 min.
I = 40 mA . 004 (part 1 of 1) 10 points
An automobile starter motor draws a current
` of 3.5 A from a 12 V battery when operating
at normal speed. A broken pulley locks the ar-
mature in position, and the current increases
to 18 A.
d What was the back emf of the motor when
operating normally?
I

Correct answer: 9.66667 V.


Explanation:
The solenoid has 450 turns
When not rotating, E = I R, and from this,
The magnetic field inside the solenoid is
E 12 V
R= = = 0.666667 Ω.
B = µ0 n I I 18 A
µ ¶
N When rotating, E − Eback = I 0 R, or
= µ0 I
`
Eback = E − I 0 R
µ ¶
450
= µ0 (0.04 A)
0.12 m = 12 V − (3.5 A)(0.666667 Ω)
= 0.000188496 T . = 9.66667 V.
Version One – Homework 36 – Ashley Smith 2

induces an emf E of 175 µV?


Current Changes Correct answer: 0.421045 A/s.
32:01, calculus, numeric, > 1 min. Explanation:
005 (part 1 of 2) 10 points Given: The number of turns in the solenoid
The current in a 90 mH inductor changes with N = 420 turns, the length of the solenoid
time as I = b t2 − a t. With a = 6 A/s and l = 16 cm, and the cross-sectional area of the
b = 1 A/s2 , find the magnitude of the induced solenoid A = 3 cm2 .
emf, |E|, at t = 1 s. The self-inductance of a solenoid is
Correct answer: 0.36 V. µ0 N 2 A
Explanation: L=
`
From Faraday’s Law, the induced emf E is
= 1.25664 × 10−6 N/A2
proportional to the rate of change of the mag-
netic flux, which in turn is proportional to (420 turns)2 × 0.0003 m2
×
the rate of change of the current. This is 0.16 m
expressed as = 0.000415633 H .
dI The induced emf E is given by
E =L
dt dI
d |E| = | − L |
= L (b t2 − a t) dt
dt dI −E
= L (2 b t − a) =| |
dt L
At t = 1 s , the magnitude of the induced emf −175 µV
=| |
is 0.000415633 H
= 0.421045 A/s .
|E| = (0.09 H) ¯ 2(1 A/s2 )(1 s) − (6 A/s) ¯
¯£ ¤¯

= 0.36 V .
Energy in an Inductor
32:01, trigonometry, numeric, > 1 min.
006 (part 2 of 2) 10 points 008 (part 1 of 1) 10 points
At what time is the emf zero, E = 0? A inductor of 300 turns has a radius of 5 cm
Correct answer: 3 s. and a length of 20 cm.
Explanation: Find the energy stored in it when the cur-
From Part 1, the induced emf is zero, E = 0, rent is 0.5 A.
when Correct answer: 0.000555165 J.
Explanation:
dI
=0 The inductance is
dt 2
2 b t0 − a = 0 2A 2 πr
a L = µ0 N = µ0 N
t0 = ` `
2b = 1.25664 × 10−6 N/A2
=3s 2
2 π (0.05 m)
× (300 turns)
(0.2 m)
Decreasing Current = 0.00444132 H .
32:01, calculus, numeric, > 1 min.
007 (part 1 of 1) 10 points The energy stored in an inductor is
An inductor in the form of an air-core solenoid 1
U= L I2
contains 420 turns, is of length 16 cm, and has 2
a cross-sectional area of 3 cm2 . 1
= (0.00444132 H)(0.5 A)2
What is the magnitude of the uniform rate 2
of change in current through the inductor = 0.000555165 J .
Version One – Homework 36 – Ashley Smith 3

Given: The number of turns in the solenoid


Flux Through a Coil 02 N = 120 turns, the diameter of the solenoid
32:01, calculus, numeric, > 1 min. D = 10 mm, and the length of the solenoid
009 (part 1 of 1) 10 points l = 9 cm.
Calculate the magnetic flux through a The self-inductance of a solenoid is given
300 turn, 7.2 mH coil when the current in by
the coil is 10 mA. NΦ
L1 = ,
Correct answer: 2.4 × 10−7 Wb. I
Explanation: where Φ is the total flux inside the solenoid
The inductance of a coil is related to the and I is the current in the wire wrapped
magnetic flux as around the solenoid.
From the text we can use Ampere’s Law to

L= find that the magnetic field in the solenoid is
I
LI µ0 N I
⇒ Φ= B= ,
N `
(0.0072 H) (0.01 A)
= where µ = µ0 is the magnetic permeability of
300 turn
the air core which is the same as free space.
= 2.4 × 10−7 Wb The magnetic flux in the solenoid is

B π D2
Inductance Coil Φ=BA= .
4
32:01, trigonometry, numeric, > 1 min.
010 (part 1 of 1) 10 points Using the above expressions for Φ and B,
A coil has an inductance of 3 mH, and the we obtain for the inductance of the solenoid
current through it changes from 0.2 A to 1.5 A NBA
in 0.2 s. L1 =
I
Find the magnitude of the average induced
µ0 N 2 π D 2
emf in the coil during this period. =
Correct answer: 19.5 mV. 4`
Explanation: = (1.25664 × 10−6 N/A2 ) (120)2
π (0.01 m)2
×
∆I 4 (0.09 m)
E =L = 1.57914 × 10−5 H .
∆t · ¸
(1.5 A) − (0.2 A)
= (0.003 H)
0.2 s
012 (part 2 of 2) 10 points
= 0.0195 V = 19.5 mV. The core is replaced with a soft iron rod that
has the same dimensions, but a magnetic per-
meability 800 µ0 .
Inductance of a Solenoid
What is the new inductance?
32:01, calculus, numeric, > 1 min.
Correct answer: 0.0126331 H.
011 (part 1 of 2) 10 points
Explanation:
A solenoid has 120 turns of wire uniformly
Let L2 be the inductance of the solenoid
wrapped around an air-filled core, which has
with the soft iron core. In the first part of
a diameter of 10 mm and a length of 9 cm.
the problem we just need to change µ0 by µ.
Calculate the self-inductance of the
Thus
solenoid.
Correct answer: 1.57914 × 10−5 H. µ N 2 π D2
Explanation: L2 =
4`
Version One – Homework 36 – Ashley Smith 4
µ
= L1
µ0 6. E is an induced electromotive force, units
= 800 (1.57914 × 10−5 H) newtons
= 0.0126331 H .
Explanation:
Inductance of a Solenoid 02 E is the self-induced electromotive force, mea-
dI
32:01, calculus, multiple choice, > 1 min. sured in Volts. L is the inductance and
013 (part 1 of 4) 10 points dt
gives the rate of change of the current.
The goal of this 4 part problem is to compute
the inductance of a solenoid with N turns, 015 (part 3 of 4) 10 points
length l, cross section area of A, in air. Gen- The inductance is measured in units of:
erally speaking, what equation defines induc-
tance? 1. Ohm
dI
1. L = E/ 2. Henry correct
dt
dI
2. L = −E/ correct 3. Tesla
dt
3. L = E/I 4. Weber

dE 5. Volt
4. L = −I/
dt
dE 6. Amp
5. L = I/
dt
7. Light-year
6. EL
Explanation:
Explanation:
Solution:
The unit of inductance is Henry.
The general formula defining inductance is
dI
E = −L
dt 016 (part 4 of 4) 10 points
How do we determine E for a solenoid of cross
014 (part 2 of 4) 10 points section A? Assume the solenoid has N total
In the above answer, what is E and what are N
turns and a length l, and n = .
the units? Choose the best answer. l
d Φm
1. E is an induced electromotive force, units 1. E = −n ,
dt
volts where Φm = (Einterior )A

2. E is a self-induced electromotive force,


d Φm
units newtons 2. E = N ,
dt
where Φm = (Binterior )A
3. E is an electromotive force, units volts

4. E is an electromotive force, units newtons d Φm


3. E = −n ,
dt
5. E is a self-induced electromotive force, where Φm = (Binterior )A
units volts correct
Version One – Homework 36 – Ashley Smith 5

the two solenoids. Thus the induced EMF in


d Φm the large solenoid will be greater if the iron
4. E = n ,
dt core is present than if it is not. By Ohm’s
where Φm = (Binterior )A
Law, we know then than the induced current
will also be greater if the iron core is present.
d Φm Since the galvanometer measures this current,
5. E = −N , the deflection of the galvanometer needle is
dt
where Φm = (Einterior )A greater than if the iron core were not present.

Long and Skinny


d Φm 32:01, calculus, numeric, > 1 min.
6. E = N ,
dt 018 (part 1 of 2) 10 points
where Φm = (Einterior )A Assume that the length of the solenoid is
much larger than the solenoid’s radius and
d Φm that the core of the solenoid is air. Calculate
7. E = −N , the inductance of a uniformly wound solenoid
dt
where Φm = (Binterior )A correct having 300 turns if the length of the solenoid
is 25 cm and its cross-sectional area is 4 cm2 .
Correct answer: 0.180956 mH.
d Φm Explanation:
8. E = n ,
dt In this case, we can take the interior magnetic
where Φm = (Einterior )A field to be uniform and given by the equation
N
Explanation: B = µ0 n I = µ 0 I,
d Φm `
E = −N for a solenoid. Φm is the
dt where n is the number of turns per unit length,
magnetic flux inside the solenoid, Φm =
N/`. The magnetic flux through each turn is
(Binterior )A.
NA
ΦB = B A = µ0 I,
`
Iron Core Solenoid
32:01, trigonometry, multiple choice, > 1 min. where A is the cross-sectional area of the
017 (part 1 of 1) 10 points solenoid. Using this expression and the equa-
An iron core is inserted into a solenoid which tion for the inductance,
is connected to a battery via a switch. A
N ΦB
larger solenoid, connected to a galvanometer, L= ,
is placed around it. When the switch is closed, I
the deflection of the galvanometer needle is we find that

1. greater than if the iron core were not µ0 N 2 A


L=
present correct `
= 1.25664 × 10−6 N/A2
2. smaller than if the iron core were not (300 turns)2 × 0.0004 m2
present ×
0.25 m
= 0.000180956 H
3. the same
= 0.180956 mH
Explanation:
Inserting an iron core into the small
solenoid increases the mutual inductance of 019 (part 2 of 2) 10 points
Version One – Homework 36 – Ashley Smith 6

Calculate the self-induced emf in the solenoid 32:01, trigonometry, numeric, > 1 min.
described in the first part if the current 021 (part 1 of 1) 10 points
through it is decreasing at the rate of 50 A/s. A 2 H inductor carries a steady current of
Correct answer: 9.04779 mV. 0.5 A. When the switch in the circuit is
Explanation: thrown open, the current disappears in 10 ms.
Using the equation What is the average induced emf in the
inductor during this time?
d ΦB Correct answer: 100 V.
E = −N ,
dt Explanation:
and given that
dI Given : L = 2 H,
= −50 A/s , I = 0.5 A , and
dt
t = 10 ms .
we get
From Faraday’s law, we know that the self-
dI induced emf is proportional to the time rate
E = −L
dt of change of the current. The proportionality
= −(0.000180956 H ) (−50 A/s ) constant L is the inductance of the inductor.
= 0.00904779 V = 9.04779 mV . Thus the self inductance is
∆I
|ε| = L
∆t
Serway CP 20 36 0.5 A
32:01, trigonometry, numeric, > 1 min. = (2 H )
0.01 s
020 (part 1 of 1) 10 points = 100 V
A coiled telephone cord has a spiral with
70 turns turns , a diameter of 1.3 cm, and
an unstretched length of 60 cm. Serway CP 20 39
Determine the self-inductance of the un- 32:01, trigonometry, numeric, > 1 min.
stretched cord. 022 (part 1 of 2) 10 points
Correct answer: 1.36217 × 10−6 H. A solenoid of radius 2.5 cm has 400 turns and
Explanation: a length of 20 cm.
Find its inductance.
Correct answer: 1.97392 mH.
Given : µ0 = 1.25664 × 10−6 n/A2 , Explanation:
N = 70 turns ,
r = 0.65 cm = 0.0065 m , and Given : r = 2.5 cm = 0.025 m ,
l = 60 cm = 0.6 m . N = 400 turns , and
l = 20 cm = 0.2 m .
The self-inductance is
A = π r 2 = π (0.025 m)2 = 0.0019635 m2 .
µ0 N2 A
L= The inductance is
l
= (1.25664 × 10−6 n/A2 ) µ0 N 2 A
L=
l
(70 turns)2 π (0.0065 m)2
× = (1.25664 × 10−6 n/a2 )
0.6 m
(400 turns)2 (0.0019635 m2 )
= 1.36217 × 10−6 H . ×
0.2 m
= 0.00197392 H
Serway CP 20 37 = 1.97392 mH .
Version One – Homework 36 – Ashley Smith 7

023 (part 2 of 2) 10 points Slinky With Inductance


Find the rate at which current must change 32:01, trigonometry, numeric, > 1 min.
through it to produce an emf of 75 mV. 025 (part 1 of 1) 10 points
Correct answer: 37.9954 A/s. A “slinky” toy spring has a radius of 4 cm and
Explanation: an inductance of 275 µH when extended to a
length of 1.5 m.
How many turns are in the spring?
Given : ² = 75 mV = 0.075 V .
Correct answer: 255.548 turns.
∆I
From ² = L , we get Explanation:
∆t µ0 N 2 A
∆I ² From L = , we find
= l
∆t L s
0.075 V Ll
= N=
0.00197392 H µ0 A
= 37.9954 A/s .
s
(0.000275 H)(1.5 m)
=
(1.25664 × 10−6 N/A2 ) π (0.04 m)2
Serway CP 20 40 = 255.548 turns .
32:01, trigonometry, numeric, > 1 min.
024 (part 1 of 1) 10 points
An emf of 24 mV is induced in a 500-turn Small Air Core Solenoid
coil when the current is changing at a rate of 32:01, calculus, numeric, > 1 min.
10 A/s. 026 (part 1 of 1) 10 points
What is the magnetic flux through each A small air-core solenoid has a length of 4 cm
turn of the coil at an instant when the current and a radius of 0.25 cm. If the inductance is to
is 4 A? be 0.06 mH, how many turns per centimeter
Correct answer: 1.92 × 10−5 Wb. are required?(Please round off your answer to
Explanation: two significant figures.)
Correct answer: 78 turns/cm.
Given : ε = 24 mV0.024 V , Explanation:
Let n be the number of turns per centime-
N = 500 ,
ter. The inductance of the air-core solenoid
I = 4 A , and is
dI
= 10 A/s . L = µ 0 n2 A l
dt
From Faraday’s Law A = π r2
dΦ dI = π 0.0025 m2
ε = −N = −L
dt dt = 1.9635 × 10−5 m2
dI L
|ε| = L ⇒ n2 =
dt µ0 A l
|ε| 0.024 V
=⇒ L = dI = = 0.0024 H
dt
10 A/s 1
=
At the instant where the current is 4 A , the 1.25664 × 10−6 N/A2
flux thru each turn is 6 × 10−5 H
×
LI (0.0024 H) (4 A) 1.9635 × 10−5 m2 × 0.04 m
Φ= = = 6.07927 × 107 (turns/m)2
N 500
−5
= 1.92 × 10 Wb .
Version One – Homework 36 – Ashley Smith 8

Therefore, Solution:
The Slinky is essentially a solenoid (coiled
q loops of wire). The inductance of a solenoid
n = 6.07927 × 107 (turns/m)2 depends on l and r, and N (the number of
loops of the coil), but does not depend on the
= 7800 turns/m
age of the child.
= 78 turns/cm
µ0 N 2 A µ0 N 2 (π r2 )
L= =
l l
Spring as an Inductor so
32:01, calculus, numeric, > 1 min. Ll
N2 =
027 (part 1 of 1) 10 points µ0 π r 2
A spring has a radius of 4 cm and an induc- s
tance of 125 µH when extended to a length of Ll
2 m. N=
µ0 π r 2
Find an approximate value for the total s
number of turns in the spring. (0.000125 H) (2 m)
=
Correct answer: 199 turns. (4π × 10−7 N/A2 ) π (0.04 m)2
Explanation: = 198.915
The spring can be considered as a solenoid.
The inductance of a solenoid is as N should be an integer, the answer is 199 .

µ0 N 2 A
L=
l Tightly Wound Circular Coil
A = πr 2 32:01, trigonometry, numeric, > 1 min.
= π (0.04 m)2 029 (part 1 of 1) 10 points
A tightly wound circular coil has 50 turns,
= 0.00502655 m2
s each of radius 0.2 m. A uniform magnetic
Ll field is introduced perpendicular to the plane
⇒ N=
µ0 A of the coil.
s If the field increases in strength from 20 T
(0.000125 H) (2 m) to 20.3 T in 0.4 s, what average emf is induced
=
µ0 (0.00502655 m2 ) in the windings of the coil?
= 199 turns Correct answer: 4.71238 V.
Explanation:
∆Φ
We use E = N , with
Testing a Slinky ∆t
32:01, calculus, numeric, > 1 min. A = π r 2 = π (0.2 m)2 = 0.125664 m2 ,
028 (part 1 of 1) 10 points
A child (approximately 4 years old) takes her ∆Φ = ∆B A
metal “Slinky Toy” (a flexible coiled metal
= (20.3 T − 20 T) (0.125664 m2 )
spring) and does various tests to determine
that the Slinky has an inductance 125 µH, = 0.037699 T m2 .
when it has been stretched to a length of 2 m. Thus,
If a Slinky has a radius of 4 cm, what is the
∆Φ
total number of turns in the Slinky? E =N
Correct answer: 199 . ∆t
Explanation: (50 turns) (0.037699 T m2 )
=
Basic Concepts: Inductance of a (0.4 s)
solenoid. = 4.71238 V .
Version One – Homework 36 – Ashley Smith 9

Turns in a Solenoid R
32:01, calculus, numeric, > 1 min.
030 (part 1 of 1) 10 points
A solenoid inductor is 20 cm long and has R P L
a cross-sectional area of 5 cm2 . When the
current through the solenoid decreases at a E
rate of 0.625 A/s, the induced emf is 200 µV. S
Find the number of turns per meter of the
solenoid.
Correct answer: 1595.77 m−1 . What is the instantaneous current at point
Explanation: P immediately after the switch is closed?
The induced emf in the solenoid is
1. IP (0) = 0 correct
dI
ε = −L . E
dt 2. IP (0) =
R
The self-inductance of a solenoid is E
3. IP (0) =
2R
µ0 N 2 A E
L= . 4. IP (0) =
l RL
EL
Using the above expression, we obtain 5. IP (0) =
2R
2E
µ0 N 2 A dI 6. IP (0) =
ε=− R
l dt
ε l 3E
N2 = − 7. IP (0) =
dI R
µ0 A 4E
dt 8. IP (0) =
0.0002 V × 0.2 m R
=−
1.25664 × 10−6 N/A2 8E
9. IP (0) =
1 R
×
0.0005 m2 × (−0.625 A/s) 16E
10. IP (0) =
= 101859 R
Explanation:
N = 319.154 . The current in L has to change gradually.
So immediately after the switch is closed,
So, the number of turns per meter is there is no current going through point P.

N 032 (part 2 of 3) 10 points


n= When the switch has been closed for a long
l
= 1595.77 m−1 . time, what is the energy stored in the induc-
tor?

L E2
AP EM 1993 MC 59 61 1. UL = correct
2 R2
32:02, calculus, multiple choice, > 1 min. LE
031 (part 1 of 3) 10 points 2. UL =
Consider the circuit shown. 2R
Version One – Homework 36 – Ashley Smith 10

Current, I [A]
L E2 5
3. UL = 4
4 R2
3
L R2 2
4. UL = 1
2 E2 2. 0
E 2 R2 0 1 2 3 4 5 6 7 8 910
5. UL =
4L Time, t [s]
LE

Current, I [A]
6. UL = 5
3R 4
LE 3
7. UL = 2
4R 1
3. 0
LE
8. UL = 0 1 2 3 4 5 6 7 8 910
8R
Time, t [s]
LE
9. UL =

Current, I [A]
16 R 5
LE 4
10. UL = 3
32 R 2
1
4. 0
Explanation: 0 1 2 3 4 5 6 7 8 910
After the switch has been closed for a Time, t [s]
long time, the current in L does not change

Current, I [A]
any more. So no voltage increase or de- 5
crease across L. Therefore, the current going 4
E 3
through L is I = , which gives the energy 2
R 1
stored in L as 5. 0
0 1 2 3 4 5 6 7 8 910
Time, t [s]
1 L E2
Current, I [A]

UL = L I2 = . 5
2 2 R2 4
3
2
1
6. 0
033 (part 3 of 3) 10 points 0 1 2 3 4 5 6 7 8 910
After the switch has been closed for a long Time, t [s]
time, it is opened at time t = 0.
Current, I [A]

Which of the following graphs best repre- 5


sents the subsequent current i at point P as a 4
3
function of time t? 2
1
7. 0
Current, I [A]

5 0 1 2 3 4 5 6 7 8 910
4
3 Time, t [s]
2
1
1. 0 cor-
0 1 2 3 4 5 6 7 8 910
Time, t [s]
rect
Version One – Homework 36 – Ashley Smith 11

Current, I [A]
5 e) the charge accumulated in the coil
4 f) presence of a magnetic iron core inside
3 the coil
2
1 g) size and shape of the coil
8. 0
0 1 2 3 4 5 6 7 8 910 1. (a),(b),(c),(d) only
Time, t [s]
2. (b),(c),(e),(g) only
Explanation:
At time t = 0, the current at P is not zero.
3. (a),(c),(f),(g) only
Because the current going through L can not
be changed abruptly, so at time t = 0, the
4. all of them
current is non-zero. Also from the Kirchhoff’s
rule, we have
5. (a),(d),(f),(g) only correct
dI Explanation:
L + I R + I R = 0, dI
dt The voltage drop across the inductor −L
dt
which will give the expression for the current depends, as we can see, on L and on the rate of
2Rt
I as I = I0 e− L . dI
current change . L, in its turn depends on
The correct graph is dt
size and shape of the coil, the number of turns
Current, I [A]

5 in the coil, and on presence of a magnetic iron


4
3 core inside the coil. This gives us (a), (d), (f),
2 and (g) options only.
1
0
0 1 2 3 4 5 6 7 8 910 Current Increase
Time, t [s] 32:02, calculus, numeric, > 1 min.
035 (part 1 of 2) 10 points
Coil of Wire in a Circuit At t = 0, a 12 V battery is connected to a
32:02, trigonometry, multiple choice, > 1 min. series circuit containing a 10 Ω resistor and a
034 (part 1 of 1) 10 points 2 H inductor.
A coil of wire of negligible resis- What will be the current in the circuit, a
tance, acting as an inductor, is connected long time after the circuit is established?
into a simple electrical circuit as shown. Correct answer: 1.2 A.
Explanation:
After a long time, we have a simple circuit
of a battery with E = 12 V and a resistor
core of R = 10 Ω. Then, E = I R and thus the
current is
E 12 V
I= = = 1.2 A .
S R 10 Ω

036 (part 2 of 2) 10 points


The voltage drop (or emf) across the induc- How long will it take the current to reach
tor depends on which of the following: 50 percent of its final value?
a) the number of turns of wire in the coil Correct answer: 0.138629 s.
b) the current flowing through it Explanation:
c) its capacitance The current in an RL circuit is given by
d) how rapidly current is changing in the ³
−t/τ
´
coil I = Is 1 − e ,
Version One – Homework 36 – Ashley Smith 12
E which can be solved forµt as ¶
where Is is the steady state current and τ
R I0
is the time constant of the circuit. Solving the t = τ ln .
If
above equation for t, we obtain
µ ¶ Increase and Decay of Current
I
t = −τ ln 1 − . 32:02, calculus, numeric, > 1 min.
Is 038 (part 1 of 3) 10 points
The time constant is An inductor and a resistor are connected with
a double pole switch to a battery as shown in
L the figure.
τ=
R
2H
=
10 Ω 4.9 Ω 140 mH S b
= 0.2 s .
a
Therefore, the time at which I = 0.5 Is is 6V

t1 = −(0.2 s) ln (1 − 0.5)
= 0.138629 s . If the switch is thrown from position b
to position a (connecting the battery), how
much time elapses before the current reaches
Decaying Current 220 mA?
32:02, trigonometry, numeric, > 1 min. Correct answer: 5.65842 ms.
037 (part 1 of 1) 10 points Explanation:
A certain circuit consists of an inductor of
15 mH in series with a resistor of 120 Ω. At
a moment when the current in the circuit is Let : R = 4.9 Ω ,
12 A, a switch in the circuit is opened. L = 140 mH , and
How long will it take for the current to fall E = 6 V.
to 4.44 A?
Correct answer: 0.000125 s.
Explanation: R L
The time constant τ of a circuit gives the S b
time required for the current to fall to 0.37 a
times its initial value. For an RL-circuit, E

L
τ= .
R
The time constant of an R L circuit is
If L
Here you can easily check that = 0.37, so τ=
I0 R
that the decay time to If is given by 0.14 H
=
4.9 Ω
0.015 H = 0.0285714 s .
τ= = 0.000125 s.
120 Ω The final current reached in the circuit is
If you did not notice this, then note that in E
I0 =
general the current at time t in such a circuit R
is given by 6V
=
4.9 Ω
I(t) = I0 e−t R/L = I0 e−t/τ , = 1.22449 A .
Version One – Homework 36 – Ashley Smith 13

The switch is in position a in an R L circuit The time t3 that elapses for the current to fall
connected to a battery at t = 0 when I = 0. to I3 = 0.16 A is
Then the current vs. time is µ ¶
I3
t3 = −τ ln
³ ´
I = I0 1 − e−t / τ . I0
= −(0.0285714 s)
Solving the above expression for t, when I = µ
0.16 A

I1 gives × ln
1.22449 A
µ ¶
I1 = 58.1459 ms .
t1 = −τ ln 1 −
I0
= −(0.0285714 s)
µ ¶ Increase of the Current
0.22 A 32:02, calculus, numeric, > 1 min.
× ln 1 −
1.22449 A 041 (part 1 of 2) 10 points
= 5.65842 ms . An inductor that has an inductance of 15 H
and a resistance of 30 Ω is connected across a
100 V battery. What is the rate of increase of
039 (part 2 of 3) 10 points the current at 0 s.
What is the maximum current in the inductor Correct answer: 6.66667 A/s.
a long time after the switch is in position a? Explanation:
Correct answer: 1.22449 A. The current in an RL circuit is
Explanation:
ε ³ ´
After a long time compared to τ , we have a I= 1 − e−R t / L .
d.c. circuit with a battery supplying an emf E, R
which is equal to the voltage drop I R across The rate of change of the current is
the resistor. Thus
dI d ³ε ε ´
E = − e−R t / L
I= dt dt R R
R ε −R
6V = − e−R t / L ×
= R L
4.9 Ω ε −R t / L
= 1.22449 A . = e .
L
At t = 0 s , the rate is
040 (part 3 of 3) 10 points
Now the switch is quickly thrown from a to b, dI 100 V −(30 Ω) (0 s) / 15 H
= e
shorting the inductor and resistor. dt 15 H
How much time elapses before the current = 6.66667 A/s .
falls to 160 mA?
Correct answer: 58.1459 ms.
Explanation: 042 (part 2 of 2) 10 points
The current decay in an R L circuit when What is the rate of increase of the current at
there is no voltage source present, and the 1.5 s.
initial current is I0 , is Correct answer: 0.331914 A/s.
Explanation:
I = I0 e−t /τ . Using the same formula derived in Part 1, the
rate of change of the current at t = 1.5 s is
Solving the above equation for t we obtain
dI 100 V −(30 Ω) (1.5 s) / 15 H
µ ¶
I = e
t = −τ ln . dt 15 H
I0 = 0.331914 A/s .
Version One – Homework 36 – Ashley Smith 14

Consider the set up shown in the figure where


Increasing Current 02 a solenoid has a steadily increasing magnetic
32:02, calculus, numeric, > 1 min. flux which generates identical induced emf’s
043 (part 1 of 2) 10 points for the two cases illustrated.
In an RL circuit, the electromotive force is i
6 V, the inductance is 24 mH, and the resis-
tance is 10 Ω. Find the current 0.5 ms after
the switch is thrown closed (1) #2 #1
B A
Correct answer: 0.112838 A.
Explanation:
The time constant of the circuit is Case 1: Two identical light bulbs are in
L series. The corresponding electrical power
τ= consumed by bulb 1 and bulb 2 are P1 and P2 ,
R
0.024 H respectively.
= C
10 Ω
= 0.0024 s .
(2) D #2 #1
The maximum current in the circuit is the fi- B A
nal current reached. This is a steady state F
current, to which the inductor has no re-
sponse. Therefore, the maximum current is Case 2: Bulb 2 is shorted by a wire which
simply given by Ohm’s Law as is connected between the two points C and
F . The corresponding electrical power con-
ε sumed by bulb 1 and bulb 2 are P10 and P20 ,
I0 =
R respectively.
6V Hint: It may be helpful to first write down
=
10 Ω the loop equation for ACDF A and ACBF A.
= 0.6 A . P10
The ratio of is given by
P1
The current in an RL circuit connected to a
battery at t = 0 is P0
1. 1 = 4 correct
³ ´ P1
I = I0 1 − e−t / τ P0
2. 1 = 8
³ ´ P1
−0.0005 s / 0.0024 s
= 0.6 A 1 − e P0
3. 1 = 3
= 0.112838 A . P1
P0
4. 1 = 2
P1
044 (part 2 of 2) 10 points
Find the maximum value of the current. P0
5. 1 = 1
Correct answer: 0.6 A. P1
Explanation: P0
6. 1 = 0
As explained in Part 1, the maximum current P1
is P0 1
I0 = 0.6 A . 7. 1 =
P1 2
P 0 1
Induced EMF 02 8. 1 =
P1 3
32:02, calculus, multiple choice, > 1 min.
045 (part 1 of 4) 10 points
Version One – Homework 36 – Ashley Smith 15
P10 1
9. = P20
P1 4 4. =2
P 0 1 P2
10. 1 = P20
P1 8 5. =1
Explanation: P2
Basic Concepts: Induced EMF P20
6. = 0 correct
Solution: Let E and R be the induced emf P2
and resistance of the light bulbs, respectively. P20 1
For case 1, since the two bulbs are in series, the 7. =
P2 2
equivalent resistance is simply Req = R + R = P20 1
2 R and the current through the bulbs is 8. =
P2 3
E P20 1
I= 9. =
2R P2 4
Hence, for case 1, the power consumed by P20 1
10. =
bulb 1 is P2 8
µ ¶2
E Explanation:
P1 = R For case 2, bulb 2 is shorted, i.e., P20 = 0.
2R
Hence the ratio is 0.
E2
=
4R
047 (part 3 of 4) 10 points
For case 2, since bulb 2 is shorted, the current Consider the set up shown in the figure where
through bulb 1 is now a solenoid has a steadily increasing magnetic
flux which generates identical induced emf’s
E for the two cases illustrated.
I0 =
R i
and the power, consumed by bulb 1 is
µ ¶2 (1) #2 #1
0 E
P1 = R B A
R
E2
= Case 1: Two identical light bulbs are in
R
series. The corresponding electrical power
Hence the ratio is consumed by bulb 1 and bulb 2 are P1 and P2 ,
P10 respectively.
= 4. C
P1

046 (part 2 of 4) 10 points o


(3) #2 #1
P0
The ratio of 2 is given by B A
P2
P 0
1. 2 = 4 i2 D i1
P2
Case 3: Let the points C and D be on the
P0 symmetry line of the diagram. Connect points
2. 2 = 8
P2 C and D by a wire, which equally divides the
P0 magnetic flux. The corresponding electrical
3. 2 = 3
P2 power consumed by bulb 1 and bulb 2 are P100
Version One – Homework 36 – Ashley Smith 16

and P200 , respectively.


Hint: It may be helpful to first write down P200
1. =4
the loop equation for ACODA) P100
P100 P200
The ratio of is given by 2. =8
P1 P100
P 00 P200
1. 1 = 1 correct 3. =3
P1 P100
P 00 P200
2. 1 = 8 4. =2
P1 P100
P 00 P200
3. 1 = 3 5. =0
P1 P100
P 00 P200
4. 1 = 2 6. = 1 correct
P1 P100
P 00 P200 1
5. 1 = 4 7. =
P1 P100 2
P 00 P200 1
6. 1 = 0 8. =
P1 P100 3
P 00 1 P200 1
7. 1 = 9. =
P1 2 P100 4
00
P 1 P200 1
8. 1 = 10. =
P1 3 P100 8
P 00 1
9. 1 = Explanation:
P1 4 The loop equation for CBDOC is
P100
=1 E
10. P 1 − I2 R = 0 .
8 2
Explanation:
For case 3, the loop equation for ACODA Solving for I2 yields
is
E E
− I1 R = 0 . I2 = .
2 2R
Solving for I1 yields
Comparing with part 3, we cans see that
E
I1 = .
2R I1 = I 2 .
Hence the power dissipated by bulb 2 for case
3 is Hence,
P200 I22 R
¶2
E2
µ
00 E = = 1.
P1 = R= .
2R 4R P100 I12 R
Using the result from part 1,
Inductor in a Circuit 02
P100 32:02, calculus, numeric, > 1 min.
= 1.
P1 049 (part 1 of 1) 10 points
An inductor that has a resistance of 50 kΩ
048 (part 4 of 4) 10 points is connected to an ideal battery of 120 V.
P 00 1.5 ms seconds after the switch is thrown the
The ratio of 200 is given by
P1 current in the circuit is 0.6 mA. Calculate the
Version One – Homework 36 – Ashley Smith 17

inductance. We will first apply Kirchoff’s Laws with the


Correct answer: 260.704 H. currents labeled as follows: I1 will flow around
Explanation: the bottom loop, through the battery; I2 will
Basic Concepts: RL circuits The magni- flow through the middle leg of the circuit, and
tude of the current in the circuit as a function I3 will flow around the top loop, through the
of time is inductor, as shown in the figure above.
E Kirchoff’s Laws then give
I(t) = (1 − e−tR/L ).
R
Knowing that at t1 = 1.5 ms the current is I1 = I 2 + I 3 , (1)
I1 = 0.6 mA , we can solve for L. First,
I1 R and from the bottom loop
e−tR/L = 1 −
E
and finally, V − I1 R − I2 R = 0 , (2)
−tR
L= = 260.704 H .
ln(1 − I1ER ) and from the outer loop

Nontrivial RL Circuit d I3
V − I1 R − I3 (2 R) − L = 0. (3)
32:02, calculus, numeric, > 1 min. dt
050 (part 1 of 2) 10 points
Prior to t = 0, the switch in the circuit is Using Eq. 1 to eliminate I2 in Eq. 2, we obtain
open.
4Ω V − I1 R − I1 R + I3 R = 0

=⇒ I1 (2 R) = I3 R + V . (20 )
8Ω 2.2 H
Substituting this into Eq. 3 gives
15 V
S 4Ω R d I3
V − (I3 R + V ) − I3 (2 R) − L = 0.
2R dt
(30 )
If the switch is closed at t = 0, find the which simplifies to this differential equation:
current in the inductor at t = 0.088 s.
Correct answer: 0.24726 A. d I3
Explanation: 2L + 5 R I3 = V .
dt

Let : R = 4 Ω , This is the differential equation for a simple


LR circuit with inductance 2 L and resistance
2R = 8 Ω,
5 R . The the time constant for this circuit is
L = 2.2 H , and 2L
τ = , and the current through the induc-
E = 15 V . 5R
tor is
R I2
V h i
Iind = I3 (t) = 1 − e(−t/τ )
2R L I3 ·5 R
(15 V)
= 1
5 (4 Ω)
E µ ¶¸
S I1 R 5 (4 Ω) (0.088 s)
− exp −
2 (2.2 H)
= 0.24726 A .
Version One – Homework 36 – Ashley Smith 18

r
051 (part 2 of 2) 10 points C
7. Imax = E correct
Find the current in the switch at that same L
time. E
r
C
Correct answer: 1.99863 A. 8. Imax =
Explanation: R L
The current through the switch is the same Explanation:
as the current through the battery, I1 . Once Basic Concepts
we have I3 , we can use equation (20 ), which R C L circuits
gives us 1 2 1 2
L Imax = q
2 2 C max
1 V 1
I1 = I3 + = C E2
2 2R 2
1 (15 V) r
C
= (0.24726 A) + ⇒ Imax = E.
2 2 (4 Ω) L
= 1.99863 A .
053 (part 2 of 3) 10 points
Which has the dimension of time in an LC
RC and LC Circuits
circuit?
32:02, trigonometry, multiple choice, > 1 min.
052 (part 1 of 3) 10 points [L]
Consider the following circuit. After leaving 1. [T ] =
[C]
the switch at the position “a” for a long time,
move the switch from “a” to “b”. There will 2. [T ] = [L] [C]
be current oscillations.
[C]
3. [T ] =
L [L]
1
4. [T ] =
C [L] [C]
S b µ ¶1
[L] 2
5. [T ] =
a [C]
E
R 6. [T ] = ([L] [C]) 2 correct
1

µ ¶ 12
[C]
The maximum current will be given by 7. [T ] =
[L]
E √ µ
1
¶ 21
1. Imax = LC 8. [T ] =
R [L] [C]
r
L Explanation:
2. Imax =E
C Let the square bracket be used to denote
√ the dimension of the enclosed quantity. Recall
3. Imax = E LC
the time constant for a RC circuit, we have
[T] = [R][C]; while for LC circuit, we have [T]
r
1
4. Imax =E = [L]/[R], so the dimension of time in a LC
LC
circuit
E
5. Imax = 1 1
[T ] = (([R][C])([L]/[R])) 2 = ([L] [C]) 2 .
R
r
E L
6. Imax = 054 (part 3 of 3) 10 points
R C
Version One – Homework 36 – Ashley Smith 19

Which has the dimension of time in an RL The maximum current will be given by
circuit?
E √
1. Imax = LC
1. [T ] = [R] [L] R
r
L
[R] 2. Imax = E
2. [T ] = C
[L] √
1 3. Imax = E L C
3. [T ] =
[R] [L] r
1
[L] 4. Imax = E
4. [T ] = correct LC
[R]
E
5. [T ] = ([R] [L])2 5. Imax =
R
r
µ
[R]
¶2 E L
6. [T ] = 6. Imax =
[L] R C
r
µ
1
¶2 C
7. [T ] = 7. Imax = E correct
[R] [L] L
r
µ
[L]
¶2 E C
8. [T ] = 8. Imax =
[R] R L
Explanation:
Explanation: Basic Concepts
One may deduce the dimension of the quan- R C L circuits
tity involving R and C from the respective 1 1 2
2
potential differences. Here we have L Imax = q
2 2 C max
· ¸
dI 1
[E] = [I][R] = [L] = C E2
dt 2
r
[L] [I] C
⇒ = · ¸ = [T ] . ⇒ Imax = E.
[R] dI L
dt RC and LC Circuits 02
32:02, trigonometry, numeric, > 1 min.
RC and LC Circuits 01 056 (part 1 of 2) 10 points
32:02, calculus, multiple choice, > 1 min. Consider the circuit below. After leaving the
055 (part 1 of 1) 10 points switch at the position “a” for a long time,
Consider the following circuit. After leaving move the switch from “a” to “b”. There will
the switch at the position “a” for a long time, be current oscillations.
move the switch from “a” to “b”. There will
be current oscillations. 50 mH

L 50 µF
S b
C
S b a
50 V
50 Ω
a
E
R
What is the maximum current?
Correct answer: 1.58114 A.
Version One – Homework 36 – Ashley Smith 20

Explanation: C

Let : L = 50 mH ,
R = 50 Ω , L S b
C = 50 µF , and a
E = 50 V . E
R

L
After leaving the switch at the position “a”
for a long time, the current reaches its steady
C state and this current is given by
S b
a 1. I = 0 .
E
R
2. I = L R .

Basic Concepts R C and L C circuits. 3. I = E R .


Solution:
R
4. I = .
1 2 1 2 1 E
L Imax = qmax = C E 2
2 2C 2 E
r 5. I = . correct
C R
⇒ Imax = E.
R
rL 6. I = .
5 × 10−5 F L
= (50 V)
0.05 H 7. I = E L .
= 1.58114 A
L
8. I = .
E
057 (part 2 of 2) 10 points r
C
What is the time constant in the RC circuit? 9. I = E .
Correct answer: 0.0025 s. L
r
Explanation: L
10. I = E .
One may deduce the dimension of the quan- C
tity involving R and C from the respectiveh qpo-
i Explanation:
tential differences. Here we have [I R] = , Basic Concepts RL circuit and LC Cir-
hq i C
cuit
or [R C] = = [T ]. The time constant is
I The loop ABDEA gives the equation,
τRC = R C dI
E −IR−L = 0.
= (50 Ω)(5 × 10−5 F) dt
= 0.0025 s . dI E
At steady state, = 0, or I =
dt R
RL and LC Circuits 01 059 (part 2 of 3) 10 points
32:02, calculus, multiple choice, > 1 min. Now the switch is moved from position “a”
058 (part 1 of 3) 10 points to “b”. There will be the usual LC circuit
Consider the circuit below. oscillations.
Version One – Homework 36 – Ashley Smith 21

The maximum voltage on the capacitor C


will be given by T
4. t =
4
R T
1. VCmax = √ . 5. t =
E LC 3
r
max E C T
2. VC = . 6. t =
R L 2
E √ Explanation:
3. VCmax = LC .
R
E
4. VCmax = √ . q
R LC VC C
r = qmax
max R L VCmax
5. VC = . C
E C
r = sin ω t
max R C 1
6. VC = . = .
E L 2
R√
7. VCmax = LC . π
E The least time in question occurs at ω t = ,
r
or 6
E L
8. VCmax = . correct π T T
R C t= = .
6 2π 12
Explanation:
For an LC circuit,
RL and LC Circuits 02
UCmax = ULmax = UL + UC = const. 32:02, calculus, numeric, > 1 min.
061 (part 1 of 4) 10 points
so,
1 1 Consider the circuit shown in the figure. Let
2 2
C VCmax = L ICmax . the switch be placed in the position b for a
2 2
long time. Then at time t = 0, the switch is
E
With the present setup, Imax = . Solving set to position a.
R
for VCmax gives, C
r r
L E L
VCmax = Imax = .
C R C
c L S b
060 (part 3 of 3) 10 points
a
Let the time, when the switch is turned from E
R
“a” to “b”, be at t = 0.
Find the least time when the voltage across
the capacitor is at one-half of the maximum
The time-dependent expression of current
voltage, where T is the period of the oscilla-
I in the left loop for t > 0 is given by
tions.
· µ ¶¸
T E 1
1. t = 1. t = 1 − exp − t
6 R RC
· µ ¶¸
T E R
2. t = 2. t = exp − t
10 R L
· µ ¶¸
T E R
3. t = correct 3. t = 1 − exp − t correct
12 R L
Version One – Homework 36 – Ashley Smith 22
µ ¶2
· µ ¶¸ 1 E
E 1 8. U = L correct
4. t = exp − t 2 R
R RC
E
· µ
L
¶¸ 9. U = 0
5. t = 1 − exp − t
R R µ ¶2
L
10. U = E
· µ ¶¸
E L
6. t = exp − t R
R R
· µ ¶¸ Explanation:
E 1
7. t = 1 − exp − t The energy stored in the inductor after a
R LC long time is given by
· µ ¶¸
E 1
8. t = exp − t 1
R LC U = L I2 .
· µ ¶¸ 2
E C
9. t = 1 − exp − t
R R After a long time, I = E/R . Thus
· µ ¶¸
E C µ ¶2
10. t = exp − t 1 E
R R U= L .
2 R
Explanation:
Just prior to shifting the switch from b 063 (part 3 of 4) 10 points
to a, there is zero current flowing. When Given: L = 2 H, E = 4 V, R = 5 Ω and
the switch is set to a, the left sided loop C = 1 F.
is a RL circuit, where the initial current is Now the switch changes from a to b, which
zero. According to the text, the current build- produces an LC-circuit in the right sided loop.
up is given by I = I0 [1 − exp(− t/τ )] , with Reset the clock so that t = 0 corresponds to
I0 = E/R , the current that flows in a dc the moment when the switch changes from a
circuit, and τ = L/R, the time scale to reach to b.
a current comparable to the dc level. Find the magnitude of potential difference
across the inductor, at t = 0.1 T, where T =
062 (part 2 of 4) 10 points 2π √
= 2π L C, which is the natural period of
What is the energy stored in the inductor ω
after a long time? oscillation of the LC circuit.
Correct answer: 0.665003 V.
µ ¶2 Explanation:
R
1. U = L The initial current in the LC circuit is
E
µ ¶ I0 = E/R , and there is zero change on the ca-
1 E pacitor. The general solution for the change
2. U = L
2 R and current through the capacitor is
µ ¶2
1 R
3. U = L Q = Qmax sin(ω t + δ) ,
2 E
1 2π
µ ¶
1 R
4. U = L with ω = √ = ,
2 E LC T
µ ¶2
1 L dQ
5. U = E I= = ω Qmax cos(ω t + δ)
2 R dt
µ ¶2
E
6. U = L = Imax cos(ω t + δ)
R
µ ¶
E In order for Q(t = 0) = 0, we must have
7. U = L δ = 0. Then, Imax = E/R = I0 is the initial
R current. The current entering the capacitor is
Version One – Homework 36 – Ashley Smith 23

also the current through the inductor. Hence 2π


t= (0.1 T), the current does not change
the current in the inductor is T
µ ¶ direction. During this time, the current flows
E E 2πt from b to c. Since the current is decreasing
I(t) = cos(ω t) = cos .
R R T from b to c, the back emf due to the inductor
wants to oppose this current change. Hence,
Then the magnitude of the potential differ- Vc > V b .
ence across the inductor at t = 0.1 T is
dI RL and LC Circuits 03
|VL | = L | | 32:02, calculus, multiple choice, > 1 min.
dt µ
t

1 065 (part 1 of 4) 10 points
= [Imax L]sin √ Consider the figure shown below. The switch
L C (LC)1/2 is initially set at position b. There is no charge
r ¯ µ ¶¯
E L ¯¯ 2 π ¯¯ nor current around the right loop while at
= sin t ¯
R C ¯ T position b. At t = 0 the switch is set to
√ position a.
(usingT = 2π L C)
s L
4 V (2 H)
= sin (2 π 0.1 T)
5 Ω (1 F) C
c S b
= 0.665003 V .
a
E
064 (part 4 of 4) 10 points R
The relationship between Vb and Vc and the
direction of the current flow at t = 0.1 T are
given respectively by What is the potential difference Va − Vc at
t = 0?
1. Vb > Vc , b to c
1. Vca = E e−1
2. Vc > Vb , c to b
2. Vca = 0
3. Vc > Vb , b to c correct
3. Vca = ∞
4. Vb > Vc , c to b
4. Vca = −E
5. Vb = Vc , c to b
5. Vca = E correct
6. Vb = Vc , b to c
6. Vca = E (1 − e−1 )
7. Vc > Vb , no current flow
7. Vca = −E e−1
8. Vb > Vc , no current flow
8. Vca = −E (1 − e−1 )
9. Vb = Vc , no current flow
9. Vca = −∞
Explanation:
T 2πt 10. Vca = ∞ − ∞
Note that since t is less than , then
4 T
π
is less than . This implies that from t = 0 to Explanation:
2
Version One – Homework 36 – Ashley Smith 24

Since at t = 0 the current is zero, we can


consider the potential difference Va −Vc across 067 (part 3 of 4) 10 points
the battery, which immediately yields After a long time at position a, the switch is
set back to position b.
Va − V c = E What is maximum charge on the capacitor
while the switch is at position b?
Another way of approaching this problem is
by using the formula
r
E 2R
1. Qmax =
¯
dI ¯¯ R L
Vac = Vc − Va = −L . E 1
dt ¯t=0 2. Qmax = √
R 2LC

066 (part 2 of 4) 10 points 3. Qmax = E R 2 L C
The switch is still in position a.
1
What is the power consumption through 4. Qmax = E R √
1 LC
the resistor R at t = τL , where τL is the E 1
2 5. Qmax = √
time constant of the LR loop? R LC
µ ¶¸2 r
E2
·
1 E R
1. Vca = 1 − exp − correct 6. Qmax =
R 2 R L
µ ¶¸2 r
E2
·
1 E L
2. Vca = 1 − exp + 7. Qmax =
R 2 R R
¶¸2 r
E2
· µ
1 R
3. Vca = 1 + exp − 8. Qmax = E R
R 2 L
µ ¶¸2 r
E 2 ·
1 L
4. Vca = 1 + exp + 9. Qmax = E R
R 2 2R
2 · µ ¶¸2 E √
E 1 10. Qmax = L C correct
5. Vca = exp − R
R 2
¶¸2 Explanation:
E2
· µ
1 By conservation of energy we have
6. Vca = exp +
R 2
¸2
E2 Qmax 2
· µ ¶
1 1
7. Vca = exp − −1 = L Imax 2 .
R 2 2C 2
E2
8. Vca = Solving for Qmax yields
R
Explanation: √ E √
The equation for the power dissipated is Qmax = Imax LC = LC .
R
given by
¶¸2
E2
· µ
2 t 068 (part 4 of 4) 10 points
P (t) = I(t) R = 1 − exp − 3
R τL At t = T , where T is the period of the LC
4
τL circuit, the relationship between Vb and Vc ,
At t = , we then have
2 and the direction of the current through the
µ ¶¸2 inductor are given by
E2
·
1
P = 1 − exp − .
R 2 1. Vb = Vc , from b through L to c
Version One – Homework 36 – Ashley Smith 25
EL −1
1. I0 = e
2. Vb > Vc , from b through L to c R
E
3. Vb > Vc , from c through L to b 2. I0 =
R
EL
4. Vb = Vc , no current flow 3. I0 =
R
5. Vb > Vc , no current flow correct 4. I0 = E R
E ¡
6. Vb = Vc , from c through L to b 1 − e−1
¢
5. I0 =
R
7. Vc > Vb , no current flow E
6. I0 = e−1
R
8. Vc > Vb , from b through L to c E
7. I0 = √
2R
9. Vc > Vb , from c through L to b
8. I0 = ∞
Explanation:
Let’s track the oscillation. At t = 0 the EL ¡
1 − e−1
¢
9. I0 =
current is at its maximum value flowing down R
through the inductor; the capacitor is un-
10. I0 = 0 correct
charged. At one quarter of the period (i.e.,
T Explanation:
t = ), the bottom plate is fully charged. At
4 The differential equation for an LC circuit
T is given by
t= the current is flowing up through the
2
3T dI
inductor while at t = , the upper plate of E −IR−L = 0,
4 dt
the capacitor is fully charged.
3 whose solution is
Hence at t = T , I = 0 and Vb > Vc .
4 · µ ¶¸
E t
RL and LC Circuits 05 I(t) = 1 − exp − .
R τL
32:02, calculus, multiple choice, > 1 min.
069 (part 1 of 4) 10 points Hence at t = 0 we have I = 0. (The back emf
Consider the figure shown below. The switch across the inductor achieves its greatest value
is initially set at position b. There is no charge at t = 0).
nor current around the right loop while at
position b. At t = 0 the switch is set to 070 (part 2 of 4) 10 points
position a. What is the potential difference Vca = Va − Vc
C at t = 0?

1. Vca = Ee−1
L S b
2. Vca = 0
a
E
R 3. Vca = ∞

4. Vca = −E
What is the current at t = 0?
5. Vca = E correct
Version One – Homework 36 – Ashley Smith 26

6. Vca = E(1 − e−1 ) E √


10. Qmax = L C correct
R
7. Vca = −E e−1 Explanation:
By conservation of energy we have
8. Vca = −E(1 − e−1 )
Qmax 2 1
= L Imax 2 .
9. Vca = −∞ 2C 2
Solving for Qmax yields
10. Vca = ∞ − ∞
√ E √
Explanation: Qmax = Imax LC = LC .
Since at t = 0 the current is zero, we can R
consider the potential difference Vba = Va − Vb
across the battery, which immediately yields 072 (part 4 of 4) 10 points
3
Va − V b = E . At t = T , where T is the period of the LC
4
circuit, the relationship between Vb and Vc ,
Another way of approaching this problem is and the direction of the current through the
by using the formula inductor are given by
dI ¯¯
Vb − Va = −L ¯ . 1. Vb = Vc , from b through L to c
dt t=0
2. Vb > Vc , from b through L to c
071 (part 3 of 4) 10 points
After a long time at position a, the switch 3. Vb > Vc , from c through L to b
is set back to position b. What is maximum
charge on the capacitor while the switch is at 4. Vb = Vc , no current flow
position b?
r 5. Vb > Vc , no current flow correct
E 2R
1. Qmax =
R L 6. Vb = Vc , from c through L to b
E 1
2. Qmax = √ 7. Vc > Vb , no current flow
R 2LC

3. Qmax = E R 2 L C 8. Vc > Vb , from b through L to c
1
4. Qmax = E R √ 9. Vc > Vb , from c through L to b
LC
E 1 Explanation:
5. Qmax = √ Let’s track the oscillation. At t = 0 the
R LC
r current is at its maximum value flowing down
E R through the inductor; the capacitor is un-
6. Qmax =
R L charged. At one quarter of the period (i.e.,
1
r
E L t = T ), the bottom plate is fully charged.
7. Qmax = 4
R R
r At t = 1/2 T the current is flowing up through
R 3
8. Qmax =ER the inductor while at t = T , the upper plate
L 4
r of the capacitor is fully charged.
L 3
9. Qmax =ER Hence at t = T , I = 0 and Vb > Vc .
2R 4
Version One – Homework 36 – Ashley Smith 27

Hence at t = 0 we have I = 0. (The back emf


RL and LC Circuits 06 across the inductor achieves its greatest value
32:02, calculus, multiple choice, > 1 min. at t = 0).
073 (part 1 of 2) 10 points
Consider the figure shown below. The switch 074 (part 2 of 2) 10 points
is initially set at position b. There is no charge 3
At t = T , where T is the period of the LC
nor current around the right loop while at 4
position b. At t = 0 the switch is set to circuit, the relationship between Vb and Vc ,
position a. and the direction of the current through the
C inductor are given by

1. Vb = Vc , from b through L to c
L S b 2. Vb > Vc , from b through L to c
a
E 3. Vb > Vc , from c through L to b
R
4. Vb = Vc , no current flow
What is the current at t = 0? 5. Vb > Vc , no current flow correct
EL −1
1. I0 = e 6. Vb = Vc , from c through L to b
R
E
2. I0 = 7. Vc > Vb , no current flow
R
EL 8. Vc > Vb , from b through L to c
3. I0 =
R
4. I0 = E R 9. Vc > Vb , from c through L to b

E ¡
1 − e−1
¢
5. I0 =
R
E
6. I0 = e−1 Explanation:
R
Let’s track the oscillation. At t = 0 the
E
7. I0 = √ current is at its maximum value flowing down
2R through the inductor; the capacitor is un-
8. I0 = ∞ charged. At one quarter of the period (i.e.,
1
EL ¡ t = T ), the bottom plate is fully charged.
9. I0 = 1 − e−1
¢ 4
R At t = 1/2 T the current is flowing up through
3
10. I0 = 0 correct the inductor while at t = T , the upper plate
4
Explanation: of the capacitor is fully charged.
The differential equation for an LC circuit 3
Hence at t = T , I = 0 and Vb > Vc .
is given by 4
dI RL and LC Circuits 10
E −IR−L = 0,
dt 32:02, calculus, multiple choice, > 1 min.
whose solution is · µ ¶¸ 075 (part 1 of 2) 10 points
E t Consider the figure shown below. The switch
I(t) = 1 − exp − .
R τL is initially set at position b. There is no charge
Version One – Homework 36 – Ashley Smith 28

nor current in the top loop while at position magnetic field in L continues to be zero tem-
b. At t = 0 the switch is set to position a. porarily i.e. i continues to be 0 temporarily.
C
076 (part 2 of 2) 10 points
3
At t = T , where T is the period of the LC
4
L S b circuit, the relationship between Vb and Vc ,
c
and the direction of the current through the
a inductor are given by
E
R
1. Vb = Vc , from b through L to c

What is the current at t = 0? 2. Vb > Vc , from b through L to c


EL −1 3. Vb > Vc , from c through L to b
1. I0 = e
R
E 4. Vb = Vc , no current flow
2. I0 =
R
EL 5. Vb > Vc , no current flow
3. I0 =
R
6. Vb = Vc , from c through L to b
4. I0 = E R
E ¡ 7. Vc > Vb , no current flow correct
1 − e−1
¢
5. I0 =
R
8. Vc > Vb , from b through L to c
E
6. I0 = e−1
R 9. Vc > Vb , from c through L to b
E
7. I0 = √
2R Explanation:
8. I0 = ∞ Let’s track the simple harmonic oscillation.
At t = 0 the current is at its maximum value
EL ¡ flowing down through the inductor; the ca-
1 − e−1
¢
9. I0 = pacitor is uncharged. At one quarter of the
R
1
10. I0 = 0 correct period (i.e., t = T ), the right plate is fully
4
Explanation: charged and the current is zero. At t = 1/2 T
The differential equation for an LC circuit the right plate has zero charge and the current
is given by flowing up through the inductor is maximum.
3
At t = T , the left plate of the capacitor is
dI 4
E −IR−L = 0, fully charged and the current is zero.
dt 3
Hence at t = T , I = 0 and Vc > Vb .
whose solution is 4
RL Circuit
· µ ¶¸
E t
I(t) = 1 − exp − . 32:02, trigonometry, numeric, > 1 min.
R τL
077 (part 1 of 1) 10 points
Hence at t = 0 we have I = 0. The inductor A circuit consists of a 14 Ω resistor and a
has a magnetic inertia. When S is at posi- 300 mH inductor connected in series to a 10 v
tion b, i = 0 and there is no magnetic field in battery.
L. Immediately after S is switched to a, the What is the value of the current when the
Version One – Homework 36 – Ashley Smith 29

current is increasing at the rate of 3 A/s? the circuit we are at “t = 0+ ”, a mathemati-


Correct answer: 0.65 A. cal notation meaning a very short time ² after
Explanation: t = 0. (Nothing happens in the circuit at
In an RL circuit t = 0, only immediately after when the switch
is, indeed, closed. However, this is just a
∆i mathematical detail.) There are two loops in
V =L + iR
∆t the problem, one with E, R1 , R2 and one with
10 v = (0.3 H)(3 A/s) + i(14 Ω) E, R1 , L. So at t = 0+ , the battery “wants
i = 0.65 A to” drive a current through both loops. The
first loop presents no problem; since there is
no inductance “working against us”, a current
RL Circuit 02 will immediately be set up. The second loop,
32:02, calculus, multiple choice, > 1 min. however, has an inductor which tries to pre-
078 (part 1 of 5) 10 points vent any change in the current going through
A circuit is set up as shown in the figure. it, and so goes up smoothly from I = 0, as can
be seen in the given solution (just put t = 0
I L to find I = 0). Therefore, at this instant, the
inductor L carries no current, and we can ne-
glect it when we find the current through R2 .
I2 R2 The equivalent resistance is Req = R1 + R2
so, from E = Req I we find
E
S R1
E
I2 =
I1 R1 + R 2
The switch is closed at t = 0. The current
I through the inductor takes the form 079 (part 2 of 5) 10 points
Find I2 immediately after the circuit is closed.
E ³ ´
I= 1 − e−t/τx E
Rx 1. I2 =
R2
where Rx and τx are to be determined. E
Find I immediately after the circuit is 2. I2 =
R1
closed.
E
3. I2 = correct
1. I = 0 correct R1 + R 2
4. I2 = 0
E
2. I =
R1 Explanation:
E See previous explanation.
3. I =
R2
E 080 (part 3 of 5) 10 points
4. I = Find I after the circuit is closed for a long
R1 + R 2
time.
Explanation:
Basic Concepts Inductors and Ohm’s E
1. I = correct
Law R1
V = RI E
2. I =
Solution: Before the circuit is closed, no R2
current is flowing. When we have just closed 3. I = 0
Version One – Homework 36 – Ashley Smith 30

E
µ ¶
1 1
4. I = 3. L + correct
R1 + R 2 R1 R2
Explanation: L
4.
When we wait a long time, the current I R2
dI L
though the inductor levels out, i.e., → 0. 5.
dt R1 + R 2
The voltage over the inductor is
6. L R1
dI
VL = L
dt 7. L R2
at all times. Thus VL → 0 as t → ∞, and
8. L(R1 + R2 )
we can replace the inductor in the figure by a
straight wire. When the current now comes Explanation:
from R1 to the junction where I1 splits into To find τx , we set up the loop equation
I2 and I, there is no resistance in the “I” through L
path but a nonzero resistance R2 in the other. dI
Naturally, the current takes the path with no E − R 1 I1 − L =0
dt
resistance. Since we do pass through R1 in
any case, the equivalent resistance is now R1 . and the loop through R2
At t = ∞
E E − R1 I1 − R2 (I1 − I) = 0
I=
R1 since I2 = I1 − I. We see from the second
I2 = 0 equation that
E + I R2
I1 =
081 (part 4 of 5) 10 points R1 + R 2
Find I2 after the circuit is closed for a long If we substitute this into the first equation,
time. we find
E
µ ¶
E + I R2 dI
1. I2 = E − R1 −L =0
R1 + R 2 R1 + R 2 dt
E or
2. I2 =
R1 µ ¶ µ ¶
E R2 R1 R2 dI
3. I2 = E − I −L =0
R2 R1 + R 2 R1 + R 2 dt

4. I2 = 0 correct meaning the circuit is equivalent to a one-loop


circuit with a battery
Explanation: µ ¶
See previous explanation. 0 R2
E = E
R1 + R 2
082 (part 5 of 5) 10 points
and a resistance
Determine the time constant τx . µ ¶
0 R1 R2
L R =
1. R1 + R 2
R1
L With these notations we see
2.
1 1 dI
+ E 0 − R0 I − L =0
R1 R2 dt
Version One – Homework 36 – Ashley Smith 31

so the time constant is What is the current I through the circuit


L L at time t = 0 ?
τx = 0 =
R R1 R2
L
R1 + R 2 1. I =
or µ ¶ R
1 1 E
τx = L + 2. I =
R1 R2 R
Alternative solution to Part 3 E
3. I =
The mathematically minded might realize RL
that an equation of the form 4. I = R L
dI
L −RI −E = 0
dt 5. I = 0 correct
has a solution which is composed of a ho-
mogeneous solution and a particular solution. E
6. I =
The particular solution corresponds to the L
first term in the I in the problem statement, Explanation:
the homogeneous corresponds to the second With switch S at position “a”, we have a
term. The homogeneous solution is therefore single loop circuit. Applying the loop rule
the only one of relevance to the time con- to the circuit, beginning at the battery and
stant τx , and we can disregard any batteries moving clockwise, the sum of the potential
E. This provides a simple solution to Part 3: changes is
Just ignore the battery and find the equiva-
lent resistance connected to L, in this case
1 1 1 dI
= + E −IR−L =0 (1)
Req R1 R2 dt
so the time constant is µ
The solution to this differential equation for I

L 1 1
τ= =L + is
Req R1 R2
as before. E h i
I= 1 − e−R t/L (2)
Nobody seems to have proven that ignoring R
the batteries always works, so the previous
solution is more reliable. At time t = 0, the exponential term is unity,
therefore Eq. (2) is
RL Circuit 03
32:02, calculus, multiple choice, > 1 min. I(0) = 0 .
083 (part 1 of 6) 10 points
In the circuit shown in the figure, the switch
S is thrown to position “a” at time t = 0.
084 (part 2 of 6) 10 points
What is the current through the circuit after
a long period of time t → ∞ ?

R L S b 1. I = 0
a E
E 2. I = correct
R
E
3. I =
RL
Version One – Homework 36 – Ashley Smith 32

4. I = R L E
4. VL =
R
L
5. I = 5. VL = E L
R
E 6. VL = R L
6. I =
L
Explanation: Explanation:
At time t −→ ∞, the exponential term in At time t −→ ∞, the exponential term in
Eq. (2) approaches zero as Eq. (2) approaches zero as

E E
I(∞) = I(∞) =
R R
From
From
I(0) = 0 ⇒ VL (0) = E
I(0) = 0 ⇒ VL (0) = E
and
E and
I(∞) = ⇒ VL (∞) = 0 . E
R I(∞) = ⇒ VL (∞) = 0 .
R
085 (part 3 of 6) 10 points Intuitive reasoning: at t=∞, I is constant;
What is the voltage drop VL across the induc- dI dI
tor at t = 0 ? i.e., = 0, or VL = L = 0.
dt dt
1. VL = EL 087 (part 5 of 6) 10 points
After the switch S has been at position “a”
2. VL = 0 for a long time (a time long relative to the
time constant of the circuit), it is switched to
E position “b”. Let this instant become t = 0.
3. VL =
L What is the current I at t = 0 ?
E
4. VL = L
R 1. I =
R
5. VL = E correct
2. I = 0
6. VL = R L E
3. I =
Explanation: RL
Rewriting the loop equation in term of VL
4. I = R L
E − I R − VL = 0 .
E
5. I = correct
R
086 (part 4 of 6) 10 points E
What is the voltage drop VL across the induc- 6. I =
L
tance after a long period of time t → ∞ ?
Explanation:
At this time, S is switched from position
1. VL = E
“a” to position “b”, thus removing the battery
from the circuit. This time is now called
2. VL = 0 correct
t = 0. The energy stored in the magnetic field
E of the induction can be dissipated through the
3. VL = resistor. Immediately at the new t = 0, the
L
Version One – Homework 36 – Ashley Smith 33

current is the same as the previous t −→ ∞, µ ¶


so R1
3. ε exp(−t(R1 + R2 )/L)
E R2
I(0) = .
R 4. ε exp(−tR1 R2 /((R1 + R2 )L))
After some time t −→ ∞, all of the energy of µ ¶
the inductor will be drained because of Ohmic R1
5. ε exp(−tR2 /L) correct
heating in the resistor. R2
µ ¶
R1
6. ε exp(−t(R1 + R2 )/L)
088 (part 6 of 6) 10 points R1 + R 2
What is the current I after a long period of
µ ¶
R1
time t → ∞? 7. ε exp(−tR1 R2 /((R1 + R2 )L))
R2
µ ¶ µ ¶
E R1 −tR1 R2
1. I = 8. ε exp
R R1 + R 2 (R1 + R2 )L
2. I = 0 correct Explanation:

E
3. I =
RL V1 = ε1 exp(−t/τ )
4. I = R L L L
τ= =
Rtotal R1 + R 2
L ε
5. I = I0 =
R R2
E εR1
6. I = ε1 =
L R
µ2 ¶
Explanation: R1
V = ε exp(−t(R1 + R2 )/L)
At that time R2

I(∞) = 0 .
Rotating Conducting Disk
32:02, calculus, numeric, > 1 min.
090 (part 1 of 2) 10 points
RL Circuit 11 A metal disk with a radius of r = 15 cm
32:02, calculus, multiple choice, > 1 min. rotates with a frequency of 60 rev/s. A mag-
089 (part 1 of 1) 10 points netic field of B = 6 T is perpendicular to the
S
disk. A resistor of R = 45 Ω is connected
between the center and the edge of the disk.
i2 L
ε i1 R1
R
R2
i

The switch S in the circuit shown has been r


closed for a long time. At t = 0, S is opened.
B
What is the voltage across resistor R1 as a
function of time t?

1. ε exp(−tR2 /L)

2. ε exp(−t(R1 + R2 )/L)
ω
Version One – Homework 36 – Ashley Smith 34

How much current will run through the where


resistor?
Correct answer: 0.565487 A. 1
A= rs
Explanation: 2
Basic Concepts: 1
= r2 θ
2
dΦ 1
E = vB = −
dt
(1) = r2 θ
2
1
V =IR (2) = r2 ω t ,
2
P =IV (3)
P = F v = τ ω. (4) and where the arc length s = r θ and the area
A = r s is approximated to be a triangle of
Solution: Consider an infinitesimal ring of height r and base s.
radius r and thickness dr. It moves with
velocity ω r. Thus the motional E induced 091 (part 2 of 2) 10 points
across this ring is What torque is required to keep the disk spin-
ning at the same rate?
dV = B v dr = B ω r dr.
Correct answer: 0.0381703 N m.
The total E induced from the center to the Explanation:
edge of the disk is then The power dissipated by the resistor is
Z Z r
V = dV = B ω r dr P = I2 R
0

1 and the power supplied to a rotating system


= B ω r2 . (5) by a torque τ is
2
Given Ohm’s Law V = IR, we find
P = τ ω.
V
I=
R Thus to keep the disk rotating at constant
B ω r2 speed, the power added must equal the power
= dissipated, so
2R
(6 T) (376.991 s−1 ) (0.15 m)2
= I2 R
2 (45 Ω) τ=
= 0.565487 A. ω
¶2
B ω r2
µ
R
Alternate Solution: Using Eqs. (1) and (2), =
2R ω
we have
B 2 r4 ω
E =
|I| = 4R
R (6 T)2 (0.15 m)4 (376.991 s−1 )
1 dΦ =
= 4 (45 Ω)
R µdt ¶ = 0.0381703 N m
B dA
=
R dt
µ ¶
B 1 2 Series RC Circuit
= r ω
R 2 32:02, trigonometry, numeric, > 1 min.
B r2 ω 092 (part 1 of 1) 10 points
= , A(n) 6 V battery is connected in series with
2R
Version One – Homework 36 – Ashley Smith 35

a resistor and an inductor. The series cir-


cuit has a time constant of 600 µs, and the Series RL Circuit 02
maximum current is 300 mA. 32:02, trigonometry, numeric, > 1 min.
What is the value of the inductance? 095 (part 1 of 3) 10 points
Correct answer: 12 mH. An inductor of 50 mH with a resistance of
Explanation: 5 Ω is connected to a power supply with a
maximum voltage of 80 V and a frequency of
V 6V 50 Hz. Find the current in the circuit.
Imax = = = 300 mA
R R Correct answer: 4.85303 A.
yields Explanation:
6V You can consider this circuit as a RLC se-
R= = 20 Ω .
300 mA ries circuit with XC = 0. XL = 2 π f L =
L 15.708 Ω, so
τ= = 600 µs = 0.0006 s.
R q
Therefore, Z= 2 + X 2 = 16.4845 Ω
RL L and
L = τ R = (0.0006 s)(20 Ω)
= 0.012 H = 12 mH. V
I= = 4.85303 A .
Z

Series RL Circuit 096 (part 2 of 3) 10 points


32:02, trigonometry, numeric, > 1 min. Find the phase angle between the current and
093 (part 1 of 2) 10 points applied voltage.
The switch in a series RL circuit with a resis- Correct answer: 72.3432 ◦ .
tance of 6 Ω , inductance of 3.5 H, and voltage Explanation:
of 24 V is closed at t = 0.2 s.
What is the maximum current in the cir-
cuit?
Correct answer: 4 A. Z
Explanation:
φ
² 24 V
Imax = = = 4 A.
R 6Ω R2
094 (part 2 of 2) 10 points
RL
What is the current when t = 0.7 s? cos φ =
Correct answer: 2.30251 A. Z
Explanation: φ = 72.3432◦ .
The time constant of the circuit is
L 3.5 H
τ= = = 0.583333 s. 097 (part 3 of 3) 10 points
R 6Ω
Find the power loss in the inductor.
Thus, at t = 0.7 s s the current has been Correct answer: 117.76 W.
flowing for ∆t = 0.5 s and is Explanation:
· µ ¶¸ The power which is converted into joule
∆t
I = 1 − exp − Imax heat is P = I 2 RL = 117.76 W.
τ
· µ ¶¸
0.5 s Series RL Circuit 03
= 1 − exp − (4 A)
0.583333 s 32:02, calculus, numeric, > 1 min.
= 2.30251 A. 098 (part 1 of 3) 10 points
Version One – Homework 36 – Ashley Smith 36

In an RL series circuit, the maximum current


is 0.5 A. After the switch is thrown, a current 100 (part 3 of 3) 10 points
of 0.25 A is reached in 0.15 s, and then the Calculate the electromotive force for this cir-
voltage across the resistor is 20 V. Calculate cuit.
the resistance for this circuit. Correct answer: 40 V.
Correct answer: 80 Ω. Explanation:
Explanation: As shown in Part 2, the maximum current is
Using Ohm’s Law, the resistance is
ε
Is = .
VR R
R=
I
20 V It follows that
=
0.25 A ε = RI
= 80 Ω .
= (80 Ω) (0.25 A)
= 40 V .
099 (part 2 of 3) 10 points
Calculate the inductance for this circuit.
Correct answer: 17.3123 H. Serway CP 20 42
Explanation: 32:02, trigonometry, numeric, > 1 min.
The current in an RL circuit is 101 (part 1 of 2) 10 points
ε ³ ´ An RL circuit with L = 3 H and an RC circuit
I= 1 − e−R t / L . with c = 3 µF have the same time constant.
R
If the two circuits have the same time con-
From the above equation, the maximum cur-
stant, what is the value of the resistance?
rent, at t = ∞, is
Correct answer: 1000 Ω.
ε Explanation:
Is = .
R
This maximum current is the steady state
current of the circuit. Now, we express the Given : L = 3 H , and
inductance in terms of the parameters of the C = 3 µF = 3 × 10−6 F .
circuit as follows :
L
³ ´ Using τ = R C = , we get
I = Is 1 − e−R t / L R
I r
L
r
3H
= 1 − e−R t / L R= =
Is C 3 × 10−6 F
I = 1000 Ω .
e−R t / L = 1 − .
Is
Taking the natural log of both sides in the
above equation, we obtain 102 (part 2 of 2) 10 points
µ ¶ What is this common time constant?
−R t I Correct answer: 3 ms.
= ln 1 −
L Is Explanation:
−R t
L= ³ ´
ln 1 − IIs L
τ=
R
−(80 Ω) (0.15 s) 3H
= = = 0.003 s
ln 1 − 0.25 A
¡ ¢
0.5 A 1000 Ω
= 17.3123 H . = 3 ms .
Version One – Homework 36 – Ashley Smith 37

At times prior to t = 0, the switch is open.


Simple LC Circuit The switch is closed at t = 0.
32:02, trigonometry, numeric, > 1 min.
103 (part 1 of 3) 10 points 4 kΩ 12 mH
A 40 µF capacitor is connected in series with
a 10 mH inductance and a switch. The ca-
pacitor is first charged to a voltage of 120 V. 240 V
The charging battery is then removed, and S
the switch is closed. I
What is the maximum charge on the capac-
itor?
Correct answer: 0.0048 C. When I = 15 mA, what is the potential
Explanation: difference across the inductor?
For the LC circuit, Correct answer: 180 V.
Explanation:
Qmax = C V
= (4 × 10−5 F) (120 V)
= 0.0048 C . Let : R = 4 kΩ ,
L = 12 mH , and
104 (part 2 of 3) 10 points E = 240 V .
What is the maximum current in the circuit?
Correct answer: 7.58947 A.
Explanation:
R L
Q
Imax = ω Q = √
LC E
0.0048 C S
=p
(0.01 H)(4 × 10−5 F) I
= 7.58947 A .
Basic Concepts: LR Circuits.
105 (part 3 of 3) 10 points The simplest method of solution is simply
What is the resonant frequency of the cir- to set the sum of the voltages around the
cuit? circuit to 0. Then
Correct answer: 251.646 Hz.
Explanation: VL = E − V R
r = (240 V) − (15 mA) (4 kΩ)
1 1 = 180 V .
f=
2π LC
s
1 1
=
2π (0.01 H)(4 × 10−5 F) Solenoid Around a Solenoid
= 251.646 Hz . 32:02, calculus, multiple choice, < 1 min.
107 (part 1 of 1) 10 points
A large solenoid is connected to a battery via
Simple LR Circuit a switch. A smaller solenoid coaxial with the
32:02, calculus, numeric, > 1 min. larger solenoid is connected to a galvanome-
106 (part 1 of 1) 10 points ter, as shown in the figure below.
Version One – Homework 36 – Ashley Smith 38

creased, this leads to a greater induced emf


in the smaller coil, thus a greater current (de-
flection in the galvanometer).

Solenoid Length
32:02, calculus, numeric, > 1 min.
G galvanometer 108 (part 1 of 1) 10 points
S
A 0.388 mH inductor has a length that is four
times its diameter. If it is wound with 22 cm−1
turns per centimeter, what is its length?
When the switch is thrown from open to
Correct answer: 0.10913 m.
closed the galvanometer needle registers a de-
Explanation:
flection.
The inductance of a solenoid is
An iron core is placed coaxially in the
solenoids, as shown below. N2 A
L = µ0 .
l
The area is
iron core π 2
A= D
4
µ ¶2
π l
= .
S G galvanometer 4 4
Substituting the above expression in L, we
obtain
When the switch is thrown from open to µ0 N 2 π l 2
closed the galvanometer needle registers a de- L= .
64 l
flection which is
Noting that N = n l, the above equation be-
1. smaller than if the iron core were not comes
µ0 n 2 l 3 π
present. L= .
64
2. greater than if the iron core were not So, the length is
present. correct µ ¶ 13
64 L
l=
3. the same as if the iron core were not π µ0 n 2
present. µ ¶ 31
64 (0.000388 H)
=
Explanation: π (1.25664 × 10−6 N/A2 ) (2200 m−1 )2
An iron core, when inserted a solenoid = 0.10913 m .
will greatly increase the magentic field in
the solenoid (for the given current in the
solenoid’s windings). This occurs because of Telephone Self Inductance
the high value of the magnetic permeability 32:02, calculus, numeric, > 1 min.
of Ferromagnetic materials. 109 (part 1 of 1) 10 points
Hence (with the iron core placed inside the A coiled telephone cord has 70 turns, a cross-
solenoids) the rate of change in the mag- sectional diameter of 1.3 cm, and an un-
netic flux through both solenoids is greatly stretched length of 60 cm. Determine an
increased when the switch is thrown. approximate value for the self-inductance of
More specifically, since the rate change of the unstretched cord.
the magnetic flux in the smaller coil is in- Correct answer: 1.36217 × 10−6 H.
Version One – Homework 36 – Ashley Smith 39

Explanation: for the current as a function of time (with t


Treating the telephone cord as a solenoid, and τ in milliseconds), we find that at t =
we have: 2 ms the current in the circuit is
d E
· µ
t
¶¸
A = π( )2 I= 1 − exp −
2 R τ
1.3 cm 2
= π( )
· µ ¶¸
12 V 2 ms
2 = 1 − exp −
= 1.32732 cm2 6 ΩΩ 5 ms
= 0.65936 A .
= 0.000132732 m2
µ0 N 2 A
⇒ L=
l
Twice the Turns
µ0 (70)2 (0.000132732 m2 )
= 32:02, calculus, multiple choice, < 1 min.
0.6 m 112 (part 1 of 1) 10 points
= 1.36217 × 10−6 H In the circuit below a steady current I0 is
flowing when the switch is in position “a”. If
the switch is then thrown to position “b”, the
Time Constant and Current current will decrease to zero after some time.
32:02, calculus, numeric, > 1 min.
110 (part 1 of 2) 10 points I(t)
The switch in the figure is closed at t = 0.

L R L S b
ε R
a
E

S
Find the time constant of the circuit if L = Suppose we replace the inductor with one of
30 mH, E = 12 V, R = 6 Ω. the same area and length, but twice as many
Correct answer: 5 ms. turns.
Explanation: The time required for current to drop to
The time constant τ is given by 1
one-half its initial value, I(t) = I0
2
L
τ= 1. increases four-fold. correct
R
0.03 H
= 2. doubles.
6 ΩΩ
= 0.005 s = 5 ms .
3. halves.

111 (part 2 of 2) 10 points 4. decreases by ln(2).


Calculate the current in the circuit at t =
2 ms. 5. increases by ln(2).
Correct answer: 0.65936 A.
Explanation: 6. is unchanged.
Using the equation
· µ ¶¸ 7. decreases by one-fourth
E t
I(t) = 1 − exp − ,
R τ 8. decreases by exp(2).
Version One – Homework 36 – Ashley Smith 40

R1
9. increases by exp(2).
Explanation:
R2 L S b
If the switch in the RL circuit is opened at
t = 0 then the current fades away as a
E
R
I(t) = I0 e− L t ,

where I0 is the current at t = 0. So the time


When the switch is at “a”, the circuit com-
required for the current to drop to 1/2I
prises the battery, the inductor L, and the
L resistor R2 . A long time after the switch has
τ= ln2 been in position “a”, the current is steady.
R
This means that, the inductor has no response
is proportional to L. When we replace the to the current. In other words, the circuit can
inductor of inductance L with one of the same be considered as consisting of E and R2 only,
area and length, but twice as many turns, the with the inductor reduced to a wire. In this
magnetic field inside it will double, and the case, the current is simply found by Ohm’s
total magnetic flux will quadruple. Therefore Law
the inductance L1 of the new inductor will be E
four times that of the original one, causing τ I0 =
R2
to quadruple.
12 V
=
Varying Voltage 12 Ω
= 1 A.
32:02, calculus, numeric, > 1 min.
113 (part 1 of 3) 10 points
One application of an RL circuit is the gen- 114 (part 2 of 3) 10 points
eration of time-varying high-voltage from a Now the switch is thrown quickly from “a” to
low-voltage source, as shown in the figure. “b”.
1200 Ω Compute the initial voltage across the in-
ductor.
Correct answer: 1212 V.
12 Ω 2H S b Explanation:
When the switch is thrown from “a” to
a
12 V “b”, the current in the circuit is the current
passing thru R2 , which was found in Part 1
to be 1 A . From Kirchhoff’s Loop Law, the
initial voltage across the inductor is equal to
What is the current in the circuit a long the initial voltage across R1 and R2 . So, we
time after the switch has been in position have
“a”?
Correct answer: 1 A. VL = V R 1 + V R 2
Explanation: = I 0 R1 + I 0 R2
= (1 A) (1200 Ω) + (1 A) (12 Ω)
Let : R1 = 1200 Ω , = 1212 V .
R2 = 12 Ω ,
L = 2 H , and 115 (part 3 of 3) 10 points
E = 12 V . How much time elapses before the voltage
Version One – Homework 36 – Ashley Smith 41

across the inductor drops to 12 V? An air-core solenoid consists of 200 turns of


Correct answer: 7.61571 ms. wire wound on a form that is ` = 40 cm long
Explanation: and has a inner diameter of d = 1 cm. If
The voltage across an inductor is a current of I = 13 A is established in this
solenoid: Given µ0 = 4π × 10−7 T m.
dI What is its self-inductance?
VL = −L .
dt Correct answer: 0.0098696 mH.
When the switch is at “b”, we are dealing with Explanation:
an RL circuit with an initial current I0 that The self-inductance of a solenoid is
decays as µ0 N 2 A
L=
I = I0 e−t / τ . l
2
Therefore, the voltage across the inductor is (1.25664 × 10−6 N/A2 )(200)2 (π 0.014 m )
=
0.4 m
d = 0.0098696 mH
VL = −L I0 e−t / τ
dt
L I0 −t / τ
= e . 117 (part 2 of 4) 10 points
τ
What is the magnetic field at its center?
Solving for t, we obtain Correct answer: 0.00816814 T.
µ ¶ Explanation:
VL τ
t = −τ ln . The magnetic field of a solenoid is
L I0
B = µ0 n I
The time constant is 200
= (1.25664 × 10−6 N/A2 )( )(13 A)
L 0.4 m
τ= = 0.00816814 T
Rt
L
= 118 (part 3 of 4) 10 points
R1 + R 2
2H How much energy is stored in it?
= Correct answer: 0.000833982 J.
1200 Ω + 12 Ω
= 0.00165017 s . Explanation:
The energy stored in a solenoid is
Therefore, the time that elapses before VL = 1
12 V is W = L I2
2
·
VL τ
¸ 1
t = −τ ln = (9.8696 × 10−6 H)(13 A)2
L I0 2
= 0.000833982 J
= −(0.00165017 s)
· ¸
(12 V) (0.00165017 s)
× ln 119 (part 4 of 4) 10 points
(2 H) (1 A)
What is the energy density in the solenoid?
= −(0.00165017 s) Correct answer: 26.5465 J/m3 .
× (−4.61512) Explanation:
= (0.00761571 s) (1000 ms/s) The energy density in the solenoid is
= 7.61571 ms . B2
w=
2 µ0
0.00816814 T2
Air Core Solenoid =
32:03, trigonometry, numeric, > 1 min. 2(1.25664 × 10−6 N/A2 )
116 (part 1 of 4) 10 points = 26.5465 J/m3
Version One – Homework 36 – Ashley Smith 42

Explanation:
Air Core Solenoid 01 The self-inductance of a solenoid is
32:03, trigonometry, numeric, > 1 min.
120 (part 1 of 4) 10 points N2 A
Given: µ0 = 4 π × 10−7 T m/A. L = µ0
`
An air-core circular solenoid is shown in the = (1.25664 × 10−6 N/A2 )
figure below. A current of 13 A is established
(222)2 π (0.01 m)2
in the wire which makes up this solenoid. ×
0.4 m
40 cm = 0.0486414 mH ,

1 cm
where A = π r 2 .
13 A

122 (part 3 of 4) 10 points


How much energy is stored in the solenoid?
Correct answer: 0.00411019 J.
The solenoid has 222 turns Explanation:
What is the magnetic field at its center? The energy stored in a solenoid is
Correct answer: 0.00906664 T.
Explanation: 1
U= L I2
2
1
= (4.86414 × 10−5 H) (13 A)2
Let : N = 222 , 2
` = 40 cm , = 0.00411019 J .
r = 1 cm , and
I = 13 A .
123 (part 4 of 4) 10 points
` A tight circular loop whose diameter is less
than the diameter of the solenoid (as shown
r in the figure below) is concentric with the
solenoid and is placed midway along the
length of the solenoid.
Starting from t = 0, the initial current in-
I

creases linearly in time until the current dou-


bles in a time t = 4 s, and then the current
The solenoid has 222 turns
remains constant at 26 A.
The magnetic field of a solenoid is
µ0 N I
B=
` 1 mm
I(t)

= (1.25664 × 10−6 N/A2 )


µ ¶
222
× (13 A)
0.4 m Inside solenoid has 33 turns
= 0.00906664 T .
Find the magnitude of the emf |E| in the
circular loop, at a time, 0 s < t < 4 s, during
121 (part 2 of 4) 10 points the increase of current.
What is its self-inductance? Correct answer: 5.87476 × 10−8 V.
Correct answer: 0.0486414 mH. Explanation:
Version One – Homework 36 – Ashley Smith 43

The self-inductance of a solenoid is

Let : T = 4 s , µ0 N 2 A
L=
d` = 0.001 m , and l
2

N` = 33 . (1.25664 × 10−6 N/A2 )(200)2 (π 0.014 m )


=
0.4 m
The magnetic flux in the loop is ΦB = B A, = 0.0098696 mH
d2
where A = π ` is the cross-sectional area of The magnetic field of a solenoid is
4
µ0 N I(t)
the loop, B = , as already stated. B = µ0 nI
` 200
The magnitude of the emf |E| around the = (1.25664 × 10−6 N/A2 )( )(13 A)
solenoid in a coil with N` turns is 0.4 m
= 0.00816814 T

|E| = N` The energy stored in a solenoid is
dt
dB 1
= N` A W = LI 2
dt 2
µ 0 N d I π d2 1
= N`
` dt 4
. = (9.8696 × 10−6 H)(13 A)2
2
= 0.000833981 J
Since I varies linearly in time until the current
doubles and the energy density is
dI ∆I 2I − I I
= = = . B2
dt ∆t T T w=
2µ0
N 222 0.00816814 T2
Thus, with n = = = 555 m−1 , =
` 40 cm 2(1.25664 × 10−6 N/A2 )
µ0 n N ` I π d 2 = 26.5465 J/m3
|E| =
4T
µ0 (555 m−1 ) (33) (13 A) 125 (part 2 of 4) 10 points
=
(4 s) What is the magnetic field at its center?
π (0.001 m)2 Correct answer: 0.00816814 T.
× Explanation:
4
= 5.87476 × 10−8 V .
126 (part 3 of 4) 10 points
How much energy is stored in it?
Air Core Solenoid 03 Correct answer: 0.000833981 J.
32:03, trigonometry, numeric, > 1 min. Explanation:
124 (part 1 of 4) 10 points
An air-core solenoid consists of 200 turns of 127 (part 4 of 4) 10 points
wire wound on a form that is 40 cm long and What is the energy density in the solenoid?
has a inner diameter of 1 cm. A current of Correct answer: 26.5465 J/m3 .
13 A is established in this solenoid. Explanation:
What is its self-inductance? Use µ0 = 4π ×
−7
10 T m. Energy and an RL Circuit
Correct answer: 0.0098696 mH. 32:03, calculus, numeric, > 1 min.
Explanation: 128 (part 1 of 4) 10 points
Version One – Homework 36 – Ashley Smith 44

The switch in the circuit shown is closed at


time t =0. 129 (part 2 of 4) 10 points
Calculate the rate at which energy is being
8Ω 4H stored in the inductor at this time.
Correct answer: 16.7432 W.
Explanation:
24 V Now we know that the energy stored in the
S inductor is given by
I
1
UL = L I2 ,
2
At what rate is energy being dissipated as
joule heat in the resistor after an elapsed time with I(t) given in the previous question.
equal to the time constant of the circuit? To find the time rate of change we take the
Correct answer: 28.7695 W. derivative with respect to time, which gives us
Explanation: the rate energy is being stored in the inductor

d UL
Let : R = 8 Ω, PL =
dt
L = 4 H , and dI
= LI
E = 24 V . dt
E −t/τ
= LI e

R L E
= LI ,
Rτ e
E because t = τ .
S L
Recall τ = , so
R
I
E R1
PL = L I
The current as a function of time in an RL R L e
circuit that has zero initial current is given by 1
=IE
e
E ³ −t/τ
´
1
I= 1−e = (1.89636 A)(24 V)
R e
= 16.7432 W .
which for the case where t = τ gives
E ¡ Alternative solution: From energy con-
1 − e−1
¢
I= servation, it follows that the power being sup-
R
plied by the battery, IE, is the sum of the
µ ¶
(24 V) 1
= 1− power dissipated by the resistor at a rate I 2 R,
(8 Ω) e
= 1.89636 A . and the energy buildup rate in the inductor
PL . Thus
The power dissipated in the resistor is given
by I E = I 2 R + PL , or

P = I2 R
= (1.89636 A)2 (8 Ω) PL = I E − I 2 R
= 28.7695 W . = 16.7432 W .
Version One – Homework 36 – Ashley Smith 45

while the switch is in the process of being


130 (part 3 of 4) 10 points thrown until the switch is thrown all the way
What is the total energy stored in the inductor to position b.
at this time?
Correct answer: 7.19238 J. I(t)
Explanation:
Again, the energy stored in an inductor is
given by R L S b
1 a
UL = L I2 E
2
1
= (4 H) (1.89636 A)2
2
= 7.19238 J . If I0 = I(t1 ) is the current in the inductor
at time t = t1 , what is the current, I(t), as a
function of time?
131 (part 4 of 4) 10 points
How long a time does it take the current to
1. I(t) = I0 (1 − exp[−R (t − t1 )/L])
reach 70 percent of its maximum value?
Correct answer: 0.601986 s.
2. I(t) = I0 exp(−R t/L)
Explanation:
Since our current is
³ ´ 3. I(t) = I0 exp(−L t/R)
I = Imax 1 − e−t/τ ,
4. I(t) = I0 (1 − exp[−L (t − t1 )/R])
Using τ = L/R, we can solve for t
µ ¶ µ ¶ 5. I(t) = I0 exp[−R (t − t1 )/L] correct
L I
t=− ln 1 −
R Imax 6. I(t) = I0 exp[−L (t − t1 )/R]
µ ¶
4H
=− ln(1 − 0.7) 7. I(t) = I0 [1 − exp(−L t/R)]
8Ω
= 0.601986 s ,
8. I(t) = I0 [1 − exp(−R t/L)]
I
where we used = 0.7 . Explanation:
Imax Basic Concepts:
Energy in an LR Circuit LR circuits.
32:03, calculus, multiple choice, > 1 min. Magnetic energy stored in an inductor:
132 (part 1 of 3) 10 points 1
UM = L I 2 .
The switch in the circuit in the figure below 2
Power dissipated in a resistor:
is initially in position a. After a very long
P = I V = I 2 R = V 2 /R .
time, the switch is thrown from position a
We know from the derivation in your textbook
to position b at time t1 . The switch in the
that for an L R-circuit, the current after the
circuit is a special kind of switch. When the
battery is disconnected is given by
switch is at position a, there is no connection
to position b. When the switch is in position
b, there is no connection to position a. But, I(t) = I0 exp[−(t − t1 )/τ ] ,
while the switch is moving from position a
to position b, both positions are connected to where τ = L/R, and I0 is the current after
the switch. This allows the current through the circuit has been connected to the battery
the inductor to be maintained by the battery for a long time. I0 = E/R . Go back to your
Version One – Homework 36 – Ashley Smith 46

textbook and review how this is found by us-


ing Kirchoff’s loop rule to derive a differential R
6. Pdiss (t) = I(t)
equation, solving the differential equation by L
separation of variables, then evaluating the dI
7. Pdiss (t) = 2 L I(t)
constants of integration by applying the ap- dt
propriate initial conditions. dI
If we define t0 = t − t1 , we know from that the 8. Pdiss (t) = R I(t)
dt
current is given by Explanation:
I(t) = I0 exp(−t0 /τ ) , The Simple Way:
so the derivative From the conservation of energy, all of the en-
µ is ¶
dI 1 ergy which is converted from magnetic energy
= I0 − exp(−t0 /τ ) . to electrical energy is dissipated in the resis-
dt τ
tor. There is nothing in the circuit to store
Note that we have made use of the fact that the electrical energy (i.e., there are no capac-
dt0 /dt = 1 since t0 = t − t1 . Putting these itors). This means that the rate at which
results back into our equation for the power magnetic energy is converted into electrical
transferred from the inductor to the rest of energy is equal to the rate at which electri-
the circuit, we find cal energy is converted into heat energy. The
µ ¶
d UM 2 1 rate of change of energy is power. This means
= L I0 − exp(−2 t0 /τ )
dt τ that the rate at which magnetic energy is con-
µ
1
¶ verted into electrical energy is equal to the
2
= −L I0 exp(−2t0 /τ ) power dissipated in the resistor. Thus,
L/R
= −I 2 (t) R . d UM
= Pdiss (t) = I 2 (t) R .
Thus, we get the same results as we found dt
using the simple argument from the conserva- The Complicated Way: The energy
tion of energy. The minus sign tells us that stored in the magnetic field of the inductor
energy is being lost from the magnetic field as is given by
magnetic energy is converted first into electri-
cal energy and then into heat energy in the 1
UM (t) = L I 2 (t) .
resistor. 2
The rate at which this energy is converted
133 (part 2 of 3) 10 points
into electrical energy is then
What is the instantaneous rate at which the
magnetic energy stored in the inductor’s mag- d UM
·
d 1
¸
dI
2
netic field is converted into electrical energy = L I (t) = L I .
dt dt 2 dt
as a function of time after the switch is thrown
from position a to position b?
134 (part 3 of 3) 10 points
2
1. Pdiss (t) = I (t) R correct What is the total amount of electrical energy
dissipated in the resistor after the switch is
1 thrown to position b?
2. Pdiss (t) = L I 2 (t)
2
dI L
3. Pdiss (t) = L 1. ∆Ediss = I0
dt R

4. Pdiss (t) = L I(t) 2. ∆Ediss = I02 R


¯
L L d I ¯¯
5. Pdiss (t) = I(t) 3. ∆Ediss =
R dt ¯t=t1
Version One – Homework 36 – Ashley Smith 47

We can evaluate this integral by making the


4. ∆Ediss = L I0 substitution, t0 = t − t1 , so dt0 = dt. The
integral becomes
1
5. ∆Ediss = L I02 correct Z ∞
2 2
∆Ediss = I0 R exp(−2 t0 /τ ) dt0
R
6. ∆Ediss = I0 ³0 τ ´ £
L ¤∞
= I02 R − exp(−2 t0 /τ ) 0
2µ ¶
¯
d I ¯¯
7. ∆Ediss = 2 L I0 1 2 L
dt ¯t=t1 = − I0 R [0 − 1]
¯
d I ¯¯ 2 R
8. ∆Ediss = R I0 1
dt ¯t=t1 = L I02 .
2
Explanation:
The Simple Way: Once again, we use the This is the same result we got using the con-
principle of the conservation of energy. The servation of energy directly.
initial energy stored in the magnetic field of
the inductor is
1 Energy Stored in an Inductor
UM i = L I02 .
2 32:03, calculus, numeric, > 1 min.
The final energy stored in the inductor is 135 (part 1 of 1) 10 points
1 1 In an RL series circuit, an inductor of 4 H
UM f = L If2 = L × 0 = 0 . and a resistor of 8 Ω are connected to a 24 V
2 2
battery. The switch of the circuit is initially
The change in the magnetic energy is thus,
open. What energy is stored in the inductor
1
∆UM = UM f − UM i = − L I02 . when the current reaches its final equilibrium
2 value after the switch is closed?
The minus sign again tells us that magnetic Correct answer: 18 J.
energy is going down as it is converted into Explanation:
electrical energy. From the conservation of The current in an RL circuit is
energy, all of this energy is dissipated in the
ε ³ ´
resistor, so the amount of energy dissipated in I= 1 − e−L t / R .
the resistor is R
1 From the above equation, we see that, the
Ediss = L I02 .
2 final equilibrium value of the current, which
The Complicated Way: The power dis- occurs as t → ∞ , is
sipated in the resistor is Pdiss (t) = I 2 (t) R .
From the definition of power, Pdiss = ε
I0 =
d Ediss /dt . We can solve for the energy dis- R
sipated by integrating this with respect to 24 V
=
time; 8Ω
Z ∞ =3A.
∆Ediss = Pdiss (t) dt
t1 The energy of an in inductor carrying a cur-
Z ∞
=R I 2 (t) dt rent 3 A is
t1
Z ∞ 1
U= L I2
2
= I0 R (exp [−(t − t1 )/τ ])2 dt 2
Zt1∞ 1
= (4 H) (3 A)2
= I02 R exp [−2 (t − t1 )/τ ] dt . 2
t1 = 18 J .
Version One – Homework 36 – Ashley Smith 48

Energy Stored in a Solenoid


32:03, calculus, numeric, > 1 min.
136 (part 1 of 1) 10 points
A solenoid 50 cm long is 7 cm in radius and
has 2000.
Find the magnetic energy of the solenoid
when the current in the windings is 2 A.
(Hint: First find the self-inductance.)
Correct answer: 0.309511 J.
Explanation:
The energy stored in a solenoid is given by
1
E= L I2
2
where L is the inductance of the solenoid.
Since the inductance in a solenoid is

L = µ 0 n2 π r 2 l

where n is the density of turns per unit length


we have
µ ¶2
1 N
E = µ0 π r2 I 2 l
2 l
µ0 N 2 π r 2 I 2
=
2l
1.25664 × 10−6 N/A2 × (2000)2
=
2 × 0.5 m
× 3.1415926536 × (0.07 m)2 × (2 A)2
= 0.309511 J .

Вам также может понравиться